You are on page 1of 36

1. PESIGAN VS.

ANGELES (LIMITATIONS)
129 SCRA 174 (1994)
Nature: Petition to review the order of the RTC of Caloocan
City, Angeles.
Facts: Anselmo and Marcelino Pesigan, both carabao dealers,
transported 26 carabaos and a calf from Sipocot Camarines Sur
to Batangas on April 2, 1982 with the necessary permits. In spite
of the permits, the carabaos were confiscated by Lt. Zenarosa
and Dr. Mirancda while passing Basud, Camarines Norte. The
confiscation was based on EO No. 626-A which provided that
no carabaos shall be transported from one province to another
and violation would result to confiscation of said animals and
distribution to deserving farmers. The carabaos were distributed
accordingly. The Pesigans filed for replevin and damages but
was dismissed by Judge Angeles for lack of merit.
Issue: W o N the Pesigans can be held liable for violating EO
626-A which was published on June 14, 1982.

At that time, appellant was taking his rest, but when he heard
that the walls of his house were being chiselled, he arose and
there he saw the fencing going on. If the fencing would go on,
appellant would be prevented from getting into his house and the
bodega of his ricemill. So he addressed the group, saying
-'Pare,if possible you stop destroying my house and if possible
we will talk it over - what is good,' addressing the deceased
Rubia, who is appellant's compadre. The deceased Fleischer,
however, answered: 'No, gademit, proceed, go ahead.' Appellant
apparently lost his equilibrium and he got his gun and shot
Fleischer, hitting him. As Fleischer fell down, Rubia ran towards
the jeep, and knowing there is a gun on the jeep, appellant fired
at Rubia, likewise hitting him. Both Fleischer and Rubia died as
a result of the. It appears, however, that this incident is
intertwined with the long drawn out legal battle between the
Fleischer and Co., Inc. of which deceased Fleischer was the
secretary-treasurer and deceased Rubia the assistant manager, on
the one hand, and the land settlers of Cotabato, among whom
was the appellant.
Issue: W o N Narvaez defense of property can be appreciated.

Held: The trial courts order of dismissal, reversed and set aside.
The executive order should not be enforced against the Pesigans
on April 2, 1982 because it is a penal regulation punished more
than two months later in the Official Gazette dated June 14,
1982. It became effective only 15 days thereafter.
The word laws in Art. 2 of Civil Code includes circulars and
regulations which prescribe penalties. Publication is necessary to
apprise the public of the contents of the regulations and make
the penalties binding on the persons affected by it.
2. TANADA VS. TUVERA (LIMITATIONS)
136 SCRA 27 (1985)
Nature and Facts: This is a case where the petitioners, Lorenzo
Taada, et al., seek a writ of mandamus to compel respondents,
Juan Tuvera (in his capacity as Exec Asst to the Pres), et al., to
publish and/or cause the publication in the official gazette of
various presidential decrees, letters of instructions, general
orders, proclamation, executive orders, letter of implementation
and administrative orders
Issue: Whether or not these presidential decrees are already
enforceable and binding even before publication (contained
special provisions as to the date they are to take effect)
Held: The government, as a matter of policy, refrains form
prosecuting violations of criminal laws until the same shall have
been published in the official gazette or in some other
publication, even though some criminal laws provide that they
shall take effect immediately.
Without publication, the people have no means of knowing what
presidential decrees have actually been promulgated. Without
such notice and publication, there would be no basis for the
application of the maxim ignorantia legis non excusat.
3. PEOPLE VS. NARVAEZ (PROSPECTIVITY OF
CRIMINAL LAW AND SELF DEFENSE/DEFENSE OF
PROPERTY)
121 SCRA 389 (1983)
Nature: Appeal from the decision of the Court of First Instance
of South Cotabato, Branch I convicting the accused of murder.
Facts: At about 2:30 in the afternoon of August 22, 1968,
Graciano Juan, Jesus Verano and Cesar Ibaez, together with the
two deceased Davis Fleischer and Flaviano Rubia, were fencing
the land of George Fleischer, father of deceased Davis Fleischer.
This is located in the municipality of Maitum, South Cotabato.
At the place of the fencing is the house and rice drier of
appellant Mamerto Narvaez.

Held: Narvaez that he did so in defense of his person and of his


rights, and therefore he should be exempt from criminal liability.
Defense of one's person or rights is treated as a justifying
circumstance under Art. 11, par. 1 of the Revised Penal Code,
but in order for it to be appreciated, the following requisites
must occur: "First. Unlawful aggression; "Second. Reasonable
necessity of the means employed to prevent or repel it; "Third.
Lack of sufficient provocation on the part of the person
defending himself".
The aggression referred to by appellant is the angry utterance by
deceased Fleischer. There is no question, therefore, that there
was aggression on the part of the victims: Fleischer was
ordering, and Rubia was actually participating in the fencing.
This was indeed aggression, not on the person of appellant, but
on his property rights.
The third element of defense of property is present, i.e., lack of
sufficient provocation on the part of appellant who was
defending his property. As a matter of fact, there was no
provocation at all on his part, since he was asleep at first and
was only awakened by the noise produced by the victims and
their laborers. His plea for the deceased and their men to stop
and talk things over with him was no provocation at all. Be that
as it may, appellant's act in killing the deceased was not
justifiable, since not all the elements for justification are
present, particularly the reasonable necessity of the means
employed to prevent or repel such attack. He should therefore
be held responsible for the death of his victims, but he could be
credited with the special mitigating circumstance of incomplete
defense, pursuant to paragraph 6, Article 13 of the Revised
Penal Code.
The SC finds the aggravating (qualifying) circumstance of
evident premeditation not sufficiently established. Since in the
case at bar, there was no direct evidence of the planning or
preparation to kill the victims nor that the accused premeditated
the killing, and clung to his premeditated act, the trial court's
conclusion as to the presence of such circumstance may not be
endorsed. Evident premeditation is further negated by appellant
pleading with the victims to stop the fencing and destroying his
house and to talk things over just before the shooting. But the
trial court has properly appreciated the presence of the
mitigating circumstance of voluntary surrender, it appearing that
appellant surrendered to the authorities soon after the shooting.
Likewise, SC also finds that passion and obfuscation attended
the commission of the crime. The appellant awoke to find his
house being damaged and its accessibility to the highway as well
as of his rice mill bodega being closed.

4. PEOPLE VS. BERNARDO (NULLUM CRIMEN NULLA


POENA SINE LEGE)
123 SCRA 365 (1983)

head. The accused, together with another, barged into the room,
demanding the firearms of the victim. Before fleeing with the
victims .38 caliber gun, TEMBLOR filed tow more shots.

Nature: Petition for certiorari of the decision of CFI Bulacan,


Br. VI convicting the petitioners of violating PD 772 (AntiSquatting Act), sentencing them to pay P 2, 500 each with
subsidiary imprisonment in case of insolvency.

On Aug. 1981, during the mass surrender of dissidents,


TEMBLOR surrendered to Mayor Dick Carmona. On Nov. 26,
1981, he was arrested by the Buenavista Police at the public
market and detained at the municipal jail where he was seen by
VICTORIA CAGAMPANG, the victims widow and was
positively identified.

Facts: ISIDRO BERNARDO, tenant of Leda Sta. Rosa in her


Riceland in PLARIDEL, BULACAN from Oct. 1972- Aug.
1974, constructed a house for their dwelling. Without the
knowledge of Sta. Rosa, Isidro left and transferred to San
Nicolas, transferring his tenancy rights to his son, CAYETANO.
Through Dr. Patricio Cruz, Sta. Rosa took possession of the
whole land and filed forcible entry against the petitioners.
Petitioners lost in the CFI and lower courts but still failed to
vacate the land. On Aug. 22, 1974, a criminal complaint for the
violation of PD 772 was filed against the Bernardos and hence
were found guilty by CFI of Bulacan.
Issue: Whether or not PD772 applies to pasture lands.
Held: Petition granted, judgment of conviction set aside;
criminal case dismissed.
PD 772 is intended to apply only to urban communities,
particularly illegal constructions. No person shall be brought
within the terms of a penal statute who is not clearly within
them; nor should any act be pronounced criminal which is not
clearly made so by the statute. (There is no crime when there is
no law punishing it.)
5. PASCUAL vs. BOARD OF MEDICAL EXAMINERS
(STRICT CONSTRUCTION)
28 SCRA 344 (1969)

TEMBLORs defense was an alibi: on the said date, he and his


father had been in the house of SILVERIO PEROL in Brgy.
Camagong.
Issue: W o N motive is essential in convicting Temblor
Held: Decision of lower court affirmed; civil indemnity
increased to P30,000.
Motive is not essential when culprit has been identified. Fact
that accused has knowledge of the deceaseds firearm is
sufficient enough for motive. The fleeing of the accused after
killing and taking CAGAMPANGs firearm implies admission
of guilt.
Accuseds alibi cannot prevail over the positive identification of
the witness who had no base motive to accuse him of the crime.
In order for alibi to be acceptable as a defense, it is not enough
for the appellant to be elsewhere when the crime was
committed, but it must be proven beyond reasonable doubt that
it was physically impossible for him to be at the scene of the
crime. Nasipit is accessible to Talo-ao by jeep or tricycle for 15
to 20 minutes.
7. PEOPLE vs. HASSAN
157 SCRA 261 (1988)

Nature: Appeal from a decision of the CFI of Manila for the


writ of prohibition rendered on Aug. 2, 1965
Facts: At the initial hearing for an administrative case for
alleged immorality, counsel for complainants called as his first
witness, the appellee, Arsenio Pascual Jr., who was the one
charged with malpractice.
Issue/s: W o N the Board was right to call and coerce Pascual to
take the witness stand against himself?
W o N the right against self-incrimination can be availed of in
an administrative hearing.
Held: Decision of the lower court affirmed.
The constitutional guarantee against self-incrimination extends
to administrative hearings which possess a criminal or penal act.
The Board cannot compel the person to take the witness stand
without his consent. A proceeding for malpractice possesses a
criminal or penal aspect in the sense that the respondent would
suffer revocation of his license as a medical practitioner.
The right against self-incrimination extends not only to the right
to refuse to answer questions put to the accused while on the
witness stand, but also to forego testimony and remain silent and
refuse to take the witness stand. That while crimes should not go
unpunished, objectives should not be accomplished by means
offensive to the high sense of respect accorded to the human
personality.
6. PEOPLE vs. TEMBLON (DOLO)
161 SCRA 623 (1988)
Nature: Appeal from the judgment of CFI of Agusan del Norte
and Butuan City convicting VICENTE TEMBLOR alias
Ronald for murder sentencing him to the penalty of reclusion
perpetua.
Facts: On Dec. 30, 1980, at around 7:30 pm in Brgy. Talo-ao,
Buenavista, Agusan del Sur, TEMBLOR bought a half-pack of
Hope cigarette from the store of JULIUS CAGAMPANG.
While the latter was opening the pack, a gun shot was heard and
CAGAMPANG fell to the floor with a gunshot wound in the

Nature: Appeal from a decision of the RTC of Zamboanga City


finding the accused guilty beyond reasonable doubt of the crime
of murder and sentenced to reclusion peretua.
Facts: July 23, 1981, at around 7pm JOSE SAMSON
was
a backrider on the motorcycle of RAMON PICHEL JR. when
they went to buy mangoes at the Fruit Paradise near the Barter
Trade Zone in Zamboanga City. SAMSON saw a person stab
PICHEL only once while he was parked 2-3 meters away. After
stabbing, the suspect fled to PNB. SAMSON rushed the victim
to the General Hospital where the latter died. In the hospital, the
witness was interrogated as to the suspects description, who
according to him was had semi-long hair, wearing white polo
short-sleeved shirt, maong pants, standing 55 and with a dark
complexion. According to SAMSON, he only knows the suspect
by face and not by name. At Funeraria La Merced, police
brought the accused alone, for identification where the witness
positively identified him as the killer.
Held: Decision reversed; accused is acquitted.
Evidence of the prosecution does not satisfy quantum of proof
beyond reasonable doubt. Value judgment must not be separated
from the constitutionally guaranteed presumption of innocence.
Prosecutions evidence is weak and unconvincing. Expert
testimony of the medico-legal officer (DR.VALENTIN
BERNALEZ) contradicted on material points of the lone
witness. He found two stab wounds, (chest and at the left arm
posterior), the nature of the wounds indicating they were
inflicted while the suspect was in front of the victim.
The investigation conducted by the police was not satisfactory.
The lone presentation of HASSAN to SAMSON at the funeral
parlor violated the accused rights to counsel in all the stages of
investigation into the commission of a crime.
Motive is essential when there is doubt as to the identity of the
culprit.
8. PEOPLE vs. AH CHONG (MISTAKE OF FACT)
15 PHIL 488 (1910)

Nature: Appeal from the judgment of CFI of Rizal convicting


the appellant of the crime of simple homicide with extenuating
circumstances sentenced to 6 years 1 day of presidio mayor.
Facts: Ah Chong was employed as a cook at Fort McKinley. He
was roommates with the deceased, PASCUAL GUALBERTO
at officers quarters #27, about 40 meters away form the nearest
building, without a lock and had only 1 door opening to the
porch and 1 window. As a safety precaution, Ah Chong and
Gualberto had an understanding that if either returned late at
night, he should knock and acquaint the other as to his identity.
On Aug. 14, 1908 at around 10 pm, he was awakened by
someone trying to force open the door. He called out twice,
Who is there?, to which no answer was given. Because of the
vines covering the porch, the room was very dark. While calling
out a threat to the invader, he was struck above the knee by the
edge of the chair which was thrown towards his direction when
the door was opened forcibly. Getting the common knife under
his pillow and struck wildly at the intruder which turned out to
be GUALBERTO. Seeing it was his roommate, Ah Chong ran
back to his room to secure bandages and called to his employees
in Room #28 for help.
Prior to the incident, there had been several instances of
robberies inside the port.
Issue/s: W o N one can be held criminally liable for doing an act
that would be exempt form criminal liability had there been no
mistake of fact.
W o N malice or criminal intent is an essential element or
ingredient of the crimes of homicide and assassination as
defined and penalized in the penal code.
Held: Decision reversed; accused is acquitted.

man killed turned out not to be Balagtas, but one SERAPIO


TECSON.
Issue: W o N the killing of Tecson was a mistake of fact to
exempt Oanis and Galanta from criminal liability.
Held: Defendants are guilty of murder with the mitigating
circumstance of incomplete requisites of performance of duty
appreciated in their favor and sentenced to reclusion temporal.
Ignorance of fact can be appreciated only when the mistake is
committed without fault of carelessness. Defendants had ample
time and opportunity to ascertain the victims identity since he
was asleep. Mistake in the identity of the intended victim cannot
be considered reckless imprudence.
10. PEOPLE VS. BUAN (CULPA)
22 SCRA 1383 (1968)
Nature: Appeal from an order of CFI Bulacan in its Criminal
Case #5423, overruling a motion to quash on the ground of
double jeopardy.
Facts: JOSE BUAN was driving a passenger bus of the La
Mallorca Company on July 23, 1962 along McArthur Highway,
Guiguinto, Bulacan when it collided with the passenger jeep of
Sergio Lumidao injuring 9 passengers. (6 suffered slight
physical injuries, 3 serious physical injuries and damaging the
jeep to the extent of P 1, 395.00)
On December 19, 1963, he was acquitted in the Justice of Peace
Court for slight physical injuries through reckless imprudence.
However, prior the acquittal, the provincial fiscal of Bulacan
filed in the CFI a case for serious physical injuries and damage
through property through reckless imprudence.

The definitions of crimes and offenses as set out in the penal


code rarely contain provisions expressly declaring that malice or
criminal intent is an essential ingredient of the crime.
Nevertheless, the provisions of Art. 1 indicate malice and
criminal intent as an essential requisite.

Issue: W o N the 2nd case places the appellant twice in jeopardy


for the same offense and is barred by the previous acquittal.

Voluntary act is a free, intelligent and intentional act, and which,


without intention there can be no crime. Voluntary implies and
includes the words con malice or with malice. When the act
which was actually intended to be done was in itself a lawful
one, and in the absence of negligence or imprudence, in general
without intention, there can be no crime.

Once acquitted or convicted of a specific act of reckless


imprudence, the accused may not be prosecuted again for the
same act. The gravity of the consequences is only taken into
account to determine the penalty, it does not qualify the
substance of the offense.

*Requisites of Mistake of fact:


The act would have been lawful had the facts be how the
accused believed them to be;
The intention would have been lawful;
The mistake was not attended by any fault or negligence on the
part of the accused.
There must be no reasonable opportunity to ascertain the facts
Excessive force negates mistake of fact (OANIS)
9. PEOPLE VS. OANIS
74 PHIL 257 (1943)
Nature: Appeal from the judgment of CFI of Nueva Ecija
finding the accused guilty of homicide through reckless
imprudence.
Facts: On December 24, 1938, Provincial Inspector, CAPT.
GODOFREDO MONSOD received a telegram from MAYOR
GUIDO ordering the arrest (whether dead or alive) of one
ANSELMO BALAGTAS, who was an escaped convict. The
said convict was informed to be with a bailarina named IRENE.
MONSOD then informed the defendants and instructed them to
arrest Balagtas, and if overpowered, they were to follow the
instructions in the telegram.
OANIS, knowing a certain IRENE, accompanied GALANTA
and went to the location of IRENE at Rizal St. Upon reaching
the place, they asked BRIGADA MALLARE to point where
IRENEs room was. They were also informed that Irene was
sleeping with her paramour. When they reached the room,
seeing a man with his back towards the door, they
simultaneously and successively fired at him killing him. The

Held: The order of the CFI is reversed and is ordered to quash


and dismiss the charges.

11. PEOPLE VS. DIZON (NEGLIGENCE) 158 SCRA 127


(1988)
Nature: An administrative complaint against respondent
Baltazar R. Dizon for rendering a manifestly erroneous decision
due to gross incompetence and gross ignorance of the law
Facts: The case in which the respondent rendered a decision of
acquittal involved a tourist, Lo Chi Fai, who was caught by a
Customs guard at the Manila International Airport while
attempting to smuggle foreign currency and foreign exchange
instruments out of the country. At the time of his apprehension,
he was found carrying with him foreign currency and foreign
exchange
instruments
(380
pieces)
amounting
to
US$355,349.57, in various currency denominations without
authority as provided by law.
At the time the accused was apprehended, he was able
to exhibit two currency declarations, which he was supposed to
have accomplished upon his arrival in Manila in previous trips.
Information was filed against Lo Chi Fai with the
RTC of Pasay City for violation of Sec. 6, Central Bank Circular
No. 960.
The respondent judge, in his decision acquitting the
accused, stated: The factual issue for this Court to determine is
whether or not the accused willfully violated Sec. 6 of Circular
No. 960. The fact that the accused had in his possession the
foreign currencies when he was about to depart from the
Philippines did not by that act alone make him liable for
violation of Sec. 6. What is imperative is the purpose for which
the act of bringing foreign currencies out of the country was
done the very intention.

Issue: Whether or not the respondent judge was guilty of gross


incompetence or gross ignorance of the law in rendering the
decision in question
Held: The Court found the respondent judge guilty of gross
incompetence, gross ignorance of the law and grave and serious
misconduct.
The respondent judge has shown gross incompetence
or gross ignorance of the law in holding that to convict the
accused for violation of Central Bank Circular No. 960; the
prosecution must establish that the accused had the criminal
intent to violate the law. The respondent ought to know that
proof of malice or deliberate intent (mens rea) is not essential in
offenses punished by special laws, which are mala prohibita.
12. PEOPLE VS. VALDEZ (PROPOSAL
CONSPIRACY) 159 SCRA 153 (1988)

AND

Nature:This case is before us on automatic review of the


decision of the Regional Trial Court, First Judicial Region,
Branch 26, San Fernando, La Union, convicting the accusedappellants Danilo Valdez and Simplicio Orodio of the crime of
murder and sentencing each of them to death.
Facts: The house of the Maquiling family stands on the slope of
a mountain in Barangay Ambagat, Santol, La Union. At about
8:00 o'clock in the evening of 7 June 1977, the victim Eleno
Maquiling, his sisters Leticia and Thelma, his mother Esmenia,
and his father Juanito were all in the yard of their house.
Esmenia and Juanito were under the awning of their house
facing north, engaged in stringing together tobacco leaves. The
victim's brother Dionisio was eating his dinner in the wall-less
kitchen located on the ground floor of the house. The victim
Eleno was seated with his back toward the north and plucking a
guitar. The place and its surroundings were lighted by a 300
candle power petromax lamp hanging under the northern end of
the awning of the house.
While the Maquilings were thus seated in their yard, a relative of
the family, one Carolina, arrived and asked Esmenia to
accompany her to a prayer meeting. Esmenia demurred and
instead asked Eleno to accompany Carolina. The victim was
then just about two (2) meters away from his parents and about
to stand up when suddenly a very loud gun shot rang out from
the northern side of the yard and Eleno fell to the ground, crying
out to his father for help. Juanito rushed to his fallen son and
carried him into their house; Eleno, however, died immediately
thereafter.
The victim's mother Esmenia was about to succor Eleno when
she instinctively looked toward the direction from whence the
gunshot came and saw the two(2) accused, Danilo Valdez and
Simplicio Orodio, running down the hill away from the bamboo
groves on the northern side of the house. Dionisio Maquiling,
brother of the victim, also testified that he too had seen Danilo
with a gun and Simplicio both running away in a westernly
direction. Danilo stated that he was then about seven (7) meters
away from the accused-appellants. Danilo Valdez was a
neighbor and a relative of the Maquilings, while Simplicio
Orodio was their old acquaintance residing in Sitio Villaga,
Barangay Corooy of the same town; thus, both were well-known
to Esmenia and Dionisio Maquiling. On 8 June 1977, the
Municipal Health, Officer of Balaoan, Dr. Monico O. Morales,
conducted an autopsy which showed that the victim Eleno had
sustained eight (8) gunshot (pellet) wounds on his back.
The morning after the shooting, on 8 June 1977, Sgt. Segundo
Tuvera of the Integrated National Police, Santol, La Union, went
to the house of the Maquilings to investigate the death of Eleno.
He saw a petromax lamp hanging from the awning of the
northern end of the house, as well as footprints near the bamboo
groves near the northern side of the house. During his
investigation, neither Esmenia nor Dionisio informed Sgt.
Tuvera of what they had seen.

On 10 June 1977, Juanito Maquiling, the victim's father,


executed a sworn statement before the police in the Santol
Police Substation. Juanito admitted in his statement that he had
not seen the accused-appellants on the night of the shooting. He
did relate, however, that three(3) days prior to the shooting of
Eleno, Eleno had informed him that in case something untoward
happened to him (Eleno), the accused-appellants Danilo Valdez
and Simplicio Orodio should be held responsible, since he
(Eleno) had quarreled with them concerning their stealing and
robbing. Juanito, further, stated that the accused Danilo has had
a personal grudge against Eleno; Danilo had mortgaged to
Eleno's brother a stolen spading fork, a circumstance that Eleno
discovered when the real owner of the spading fork came to talk
to him. Esmenia, Eleno's mother, gave no sworn statement on
that day. Ten (10) days later, on 20 June 1977, however, she
made a sworn statement to the Philippine Constabulary in San
Fernando, La Union. Shortly thereafter, on 23 June 1977,
Dionisio Maquiling, Eleno's brother, gave his own separate
sworn statement also to the Philippine Constabulary. Both
Esmenia and Dionisio identified Danilo Orodio as Eleno's
killers. At the trial, Esmenia Maquiling was firm and categorical
in identifying the appellants as the men she saw running from
the bamboo groves immediately after the shooting..
Held: There is nothing in the record to show that the
prosecution witnesses were moved by any improper motive to
accuse falsely the accused-appellants one a relative and the
other an old acquaintance of so grave a crime as murder.
The prosecution's evidence was more than adequate to sustain
the finding of the trial court of a conspiracy between Danilo
Valdez and Simplicio Orodio. Conspiracy being present, it does
not matter that the prosecution had failed to show who as
between the two actually pulled the trigger of the shotgun that
killed Eleno Maquiling. 17 Both Danilo Valdez and Simplicio
Orodio are liable as co-conspirators since any act of a coconspirator becomes the act of the other regardless of the precise
degree of participation in the act.
The trial court correctly appreciated the presence of treachery
and evident premeditation. The accused had purposely sought
nocturnity and hid themselves behind the bamboo groves located
close by the victim's house and had fired at Eleno Maquiling
suddenly, without any warning, from behind obviously to ensure
the success of their deadly purpose without any risk to
themselves and without any possibility of retaliation.
Since both treachery and evident premeditation were present,
and only one qualifying circumstance is necessary to constitute
homicide into murder, evident premeditation may be considered
as a generic aggravating circumstance. The circumstance of
nighttime is, however, absorbed by treachery. A second
aggravating circumstance that the victim who had given no
provocation was slain in his dwelling was also found by the
trial court.
13. PEOPLE VS. NACIONAL
248 SCRA 122 (1995)
Nature: Appeal from the decision of the Regional Trial Court of
Legazpi City
Facts: The six accused, including the accused-appellant, were all
civilian members of the barangay organization of the CPP-NPA
at Daraga, Albay. Their organization had a conference for the
purpose of identifying suspected informers of the military whom
they perceived as posing a threat to the NPAs operations within
the vicinity. They identified Quirino Lagason and Joel Lagason
as military informers and were targeted for liquidation.
According to Crisanto Miranda, a neighbor who
accompanied the accused that day, Walter Nacional approached
Quirino and said something to him. Walter then pulled out a gun
from his waist and shot Quirino in the face, hitting him between
the eyebrows. Quirino fell to the ground and died instantly. A
few seconds later, Absalon Millamina shot Joel Lagason on the
head. The group then fled towards the direction of the RCPI

Relay Station. Joels mother, who was at the scene of the crime,
rushed him to the hospital where he died a few hours later.
The defense set up by the accused consisted of denials.
Accused-appellant Javier Mirabete insisted on his claim that he
was merely watching a volleyball game when the shooting
happened. He denied being a member of the NPA or any rebel
organization. He likewise denied the existence of a plot and a
conspiracy to kill the Lagasons. Accused-appellants claimed that
he was a mere farmer, already 69 years old and had barely
finished third grade in school. According to him, his advanced
age made it impossible for him to join the NPA at the time of the
incident. He contended that the testimonies of the witnesses
identifying him with the group that killed the Lagasons were
unreliable and hearsay because both witnesses never knew him.
Issue: Whether or not accused-appellants contention is with
merit
Held: The decision appealed from was affirmed insofar as the
criminal liability of accused-appellant was concerned.
Evidence proved beyond doubt that accused-appellant
was a civilian member of the CPP-NPA and was part of the
group that deliberately planned the killing of the Lagasons. The
events that led to the victims deaths also showed that the group
members deliberately planned, plotted and premeditated their
victims deaths. Evident premeditation exists when the
execution of the criminal act was preceded by cool thought and
reflection upon the resolution to carry out the criminal intent.
There must be, between the reflection and execution of the
crimes, a space of time sufficient for the offender to arrive at a
calm judgment.
It was also held that the prosecution had clearly and
convincingly established the existence of a conspiracy in the
planning and execution of the crimes. Conspiracy arises at the
very instant the plotters agree, expressly or impliedly, to commit
the felony and forthwith to actually pursue it. It hardly matters
that the accused-appellant was not actually present at the
specific place of the shooting. He was at the waiting shed but
this was for the purpose of providing security to those who
carried out the shooting. The waiting shed was located along the
way to the Lagasons house, strategically at the entrance to and
exit from it. A conspiracy, once established, makes each of the
conspirators liable for the acts of the others. All conspirators are
liable as co-principals regardless of the extent of their
participation because in the contemplation of law, the act of one
is the act of all.
14. PEOPLE VS. TRINIDAD
169 SCRA 51 (1989)
Nature: APPEAL from the judgment of the Regional Trial Court
of Bayugan, Agusan del Sur, Br. 7, convicting the accused of
two counts of murder and frustrated murder.
Facts: The deceased victim, Lolito Soriano, was a fish dealer
based in Davao City. His helpers were Ricardo Tan, a driver,
and the other deceased victim Marcial Laroa. On 19 January
1983, using a Ford Fiera, they arrived at Butuan City to sell fish.
In the morning of 20 January 1983 Soriano together with Laroa
and a helper of one Samuel Comendador left for Buenavista.
Tan was left behind in Butuan but followed to later in the
morning.
While at Buenavista, accused Emeliano Trinidad, a member of
the Integrated National Police, assigned at Nasipit Police
Station, and residing at Baan, Butuan City, asked for a ride to
Bayugan, Agusan del Sur, which is on the way to Davao City.
He was in uniform and had two firearms, a carbine, and the
other, a side-arm .38 caliber revolver. The four left Butuan on 20
January 1983 at about 5:20 P.M. bound for Davao City. Tan was
driving, seated to his right were Soriano, then Laroa and the
accused. When they reached the stretch between El Rio and
Afga, the police advised them to drive slowly because,
according to him, the place was dangerous. All of a sudden, Tan
heard two gunshots. Soriano and Laroa slumped dead. He did
not actually see the shooting of Laroa but he witnessed the
shooting of Soriano, having been alerted by the sound of the first
gunfire. Both were hit on the head. The accused had used his
carbine in killing the two victims.
Ricardo hurriedly got off the Fiera, ran towards the direction of
Butuan City and hid himself in the bushes. He heard a shot

emanating from the Fiera while he was hiding in the bushes,


then a passenger jeep passed by, he hailed it and rode on the
front seat. However, after sometime, he noticed that the accusedappellant was seated at the back. Apparently noticing him as
well, the accused ordered him to get out and to approach him.
Instead of following, Tan moved backward and ran around the
jeep followed by the appellant. When the jeep started to drive
away, Ricardo clung to its side. The accused fired two shots, one
of which hit Tan on his right thigh. As another passenger jeep
passed by, he jumped from the first jeep and ran to the second.
However, the passengers in the latter jeep told him to get out not
wanting to get involved in the affray. Pushed out, Ricardo
crawled until a member of the P.C. chanced upon him and
helped him board a bus for Butuan City. Trinidad was convicted
for the murder of Laroa and Soriano and for the frustrated
murder of Ricardo.
Issue: W o N the accused the attack on Tan constitutes frustrated
or attempted murder.
Held: Decision modified. Trinidad was found guilty of the two
murders and attempted murder.
The defense is correct in contending that in the Frustrated
Murder case, TRINIDAD can only be convicted of Attempted
Murder. He had commenced the commission of the felony
directly by overt acts but was unable to perform all the acts of
execution which would have produced it by reason of causes
other than his spontaneous desistance, such as, that the jeep to
which TAN was clinging was in motion, and there was a spare
tire which shielded the other parts of his body. Moreover, the
wound on his thigh was not fatal and the doctrinal rule is that
where the wound inflicted on the victim is not sufficient to cause
his death, the crime is only attempted Murder, the accused not
having performed all the acts of execution that would have
brought about death.
15. PEOPLE VS. VELASCO
73 SCRA 574 (1976)
Nature: Appeal from the decision of the Regional Trial Court
Facts: On Nov. 2, 1967, the offended party, Estelita Lopez, five
years old, accompanied by her cousin Nenita Lopez, was at the
North Cemetery, Manila. The defendant, Ricardo Velasco,
called them, gave Nenita a five-centavo coin and asked her to
buy cigarettes for him. After she left, the accused held Estelita
by the hand and brought her to an alley. Once in a hidden place
between the tombs, he kissed her on the lips, took off her panties
and placed himself on top of the girl while she was lying down
on the ground face up and tried to insert his sexual organ into
that of the victim. The girl shouted in pain, Aray, aray!
Arsenio Perez, who happened to see the accused
holding the hand of the girl while walking along 24 th street in the
cemetery as well as when they turned into the alley and who at
first thought that the accused was a relative of the girl, upon
hearing the shouts of the girl and because of the shouts believed
that something bad was being done to the girl, proceeded to the
place where the shouts came and upon standing on top of one of
the tombs he saw, a short distance away, the accused on top of
the girl, with his pants and drawers lowered down to his knees,
and the dress of the girl raised up and the buttocks of the
accused making upward and downward movements. He tried to
seek for help and upon seeing Jose Castro on the 24th street told
him, Mang Pepe, Mang Pepe, the daughter of Mang Pidiong is
being raped! Castro proceeded to the place pointed by Perez
with the latter following him, and while standing on top of a
tomb, a short distance away, Castro saw the defendant on top of
the girl.
The accused then stood up and raised his pants while
the girl rose from the ground crying. Castro approached the
defendant and the girl and asked him what happened, and he
said the girl lost her way and was crying. The girl was bleeding
at the same time and he noticed that she even wiped off with her
dress the blood on the front part of her thighs.
Issue: Whether or not the accused is guilty of consummated
rape

Ruling: The decision of the lower court finding the accused


guilty of the crime of consummated rape was affirmed.
There was no question that rape was the crime
committed, but because of the tender age of the victim,
penetration was impossible due to the infantile character of the
vagina. However, considering the anatomical position of the
labia majora and minora, that these two external parts of the
female sexual organ cover the hymen and the vaginal opening
and, therefore, in order to rupture the hymen and produce the
medico-legal finding that the vaginal opening was painful and
sensitive to touch. The conclusion was inevitable that the
sexual organ of the accused must have entered and had passed
the labia majora.
16. URBANO VS. IAC
157 SCRA 1 (1998)
Nature: This is a petition to review the decision of the then
Intermediate Appellate Court which affirmed the decision of the
Circuit Criminal Court of Dagupan City finding petitioner
Filomeno Urbano guilty beyond reasonable doubt of the crime
of homicide.
Issue: Whether or not there was an efficient intervening cause
from the time Javier was wounded until his death which would
exculpate Urbano from any liability for Javiers death.
Facts: (1) At about 8:00oclock in the morning of October 23,
1980, petitioner Filomeno Urbano went to his rice field at
Barangay Anonang, San Fabian, Pangasinan located at 100
meters from the tobacco seedbed of Marcelo Javier . He found
the place where he stored his palay flooded with water coming
from the irrigation canal nearby which had overflowed. Urbano
went to the elevated portion of the canal to see what happened
and there he saw Marcelo Javier and Emilio Erfe cutting grass.
He asked them who was responsible for the opening of the
irrigation canal and Javier admitted that he was the one. Urbano
then got angry and demanded that Javier pay for his soaked
palay. A quarrel between them ensued. Urbano unsheathed his
bolo (about 2 feet long, including the handle, by 2 inches wide)
and hacked Javier hitting him on the right palm of his hand,
which was used in parrying the bolo hack. Javier who was then
unarmed ran away from Urbano but was overtaken by Urbano
who hacked him again hitting Javier on the left leg with the back
portion of said bolo, causing a swelling on said leg. When
Urbano tried to hack and inflict further injury, his daughter
embraced and prevented him fro hacking Javier.
(2) Upon the intercession of Councilman Solis, Urbano and
Javier agreed to settle their differences. Urbano promised to pay
Php 700.00 for the medical expenses of Javier. Hence, on
October 27, 1980, the two accompanied by Solis appeared
before the San Fabian Police to formalize their amicable
settlement.
(3) However, at about 1:30 a.m. on November 11, 1980 Javier as
rushed to the Nazareth General Hospital in very serious
condition. When admitted to the hospital Javier lockjaw and was
having convulsion Dr. Edmundo Exconde who personally
attended to Javier found the latters serious condition was
caused by tetanus toxin. Eh noticed the presence of a hearing
wound in Javiers palm which could have been infected by
tetanus.
(4) In an information dated April 10, 1981 Filomeno Urbano
was charged with crime of homicide before the then Circuit
Court of Dagupan City, Third Judicial District. Upon agreement,
Urbano pleaded not guilty. After trial, the trial court found
Urbano guilty as charged. We was sentenced to suffer an
indeterminate prison term of twelve (12) years of prision mayor,
as minimum to seventeen (17) years., four (4) months and one
(1) day of reclusion temporal, as maximum, together with the
accessories of the law to indemnify the heirs of the victim,
Marcelo Javier, in the amount of Php 12,000.00 without
subsidiary imprisonment in case of insolvency, and to pay the
costs. He was ordered confined at the New Bilibid Prison, in
Muntinlupa, Rizal upon finality of the decision, in view of the
nature of his penalty. The then Intermediate Appellate Court
affirmed the conviction of Urbano on appeal but raised the
award of indemnity to the heirs of the deceased to Php
30,000.00 with cost against the appellant. The appellant filed a
motion for reconsideration and/or new trial. The motion for new

trial was based on an affidavit of Barangay Captain Menardo


Soliven.
(5) The motion was denied. Hence, this petition.
Ruling: The instant petition is granted, the questioned decision
of the then Intermediate Appellate Court, now Court of Appeal,
is reversed and set aside. The petitioner is acquitted of the crime
of homicide. Costs de oficio.
Rationale: (a) The evidence on record does not clearly show
that the wound inflicted by Urbano was infected with tetanus at
the time of the infliction of the wound. The evidence merely
confirms that the wound, which was already healing at the time
Javier suffered the symptoms of the fatal ailment, somehow got
infected with tetanus. However, as to when the wound was
infected is not clear from the record. (b) Doubts are present.
There is likelihood that the wound was but the remote cause and
its subsequent infection, for failure to take necessary
precautions, with tenants may have been the proximate cause of
Javiers death with which the petitioner had nothing to do.
17. PEOPLE VS. ABARCA
153 SCRA 735 (1987)
Nature: This is an appeal from the decision of the Regional
Trial Court of Palo, Leyte, sentencing the accused-appellant
Francisco Abarca to death for the complex crime of murder with
double frustrated murder.
Issue: The accused-appellant assigns the following errors by the
court a quo: (i) in convicting the accused for the crime charged
instead of entering a judgment of conviction under article 247 of
the Revised Penal Code, and (ii) in finding that the killing was
attended by the qualifying circumstance of treachery.
Facts: (1) Khingsley Paul Koh and the wife of accused
Francisco Abarca. Jenny had illicit relationship. The illicit
relationship apparently began while the accused was in Manila
reviewing for the 1983 bar examinations.
(2) On July 15, 1984, the accused was in his residence in
Tacloban, Leyte. On the morning of that date he went to the bus
station to go to Dolores, Easter Samar, to fetch his daughter.
However, he was not able to catch the first trip (in the morning).
He went back to the station in the afternoon to take the 2:00
oclock trip but the bus engine trouble and could not leave. The
accused then proceeded to the residence of his father after which
he went home. He arrived at his residence at the V & G
Subdivision in Tacloban City at around 6:00 oclock in the
afternoon.
(3) Upon reaching home the accused found his wife, Jenny and
Khingsley in the act of sexual intercourse. When the wife and
Koh noticed the accused, the wife pushed her paramour who got
his revolver. The accused who was then peeping above the builtin cabinet in their room jumped and ran away.
(4) The accused went to look for a firearm at Tacloban City. He
went to the house of PC soldier, C2C Arturo Talbo, arriving
there at around 6:30 p.m. He got Talbos firearm, an M-16 rifle
and went back to his house at V & G Subdivision. He was not
able to find his wife and Koh there. He proceeded to the
mahjong session as it was the hangout of Kingsley Koh.
The accused found Koh playing mahjong. He fired at Kingsley
Koh three times with his rifle. Koh was hit. Arnold and Lina
Amparado who were occupying a room adjacent to the room
where Koh was playing mahjong were also hit by shots fired by
the accused Kingsley Koh died instantaneously of
cardiorespiratory arrest due to shock and hemorrhage as a result
of multiple gunshot wounds in the head, trunk and abdomen.
Arnold Amparado was hospitalized and operated on in the
kidney to remove a bullet. His wife, Lina Amparado, was also
treated in the hospital as she was hit by bullet fragments.
Ruling: The decision appealed from is modified. The accusedappellant is sentenced to four months and 21 days to six months
of arresto mayor. The period within which he has been in
confinement shall be credited in the service of these penalties.
He is furthermore ordered to indemnify Arnold and Lina
Amparado in the Sum of Php 16,000.00 as and for Arnold
Amparados loss of earning capacity. No special pronouncement
as to costs.

Rationale: (a) Art. 247 of the RPC apply in the instant case.
There is no question that the accused surprised his wife and her
paramour, the vacuum in this case, in the act of illicit copulation,
as, a result of which he went out to kill the deceased in a fit of
passionate outburst. Article 247 prescribes the following
elements: (1) that a legally married person surprises his spouse
in the act of committing sexual intercourse with another person,
and (2) that he kills any of them or both of them in the act or
immediately thereafter. These elements are present in this case.
The trail court, in convicting the accused-appellant of murder,
therefore erred.
(b) Article 247, or the exceptional
circumstances mentioned therein, amount to an exempting
circumstances mentioned therein, amount to an exempting
circumstance, for even where death or serious physical injuries
is inflicted, the penalty is so greatly lowered as to result to no
punishment at all. (c) Article 247 of the Revised Penal code does
not define and provide for specific crime, but grants a privilege
or benefit to the accused for the killing of another or the
infliction of serious physical injuries under the circumstances.
Punishment consequently is not inflicted upon the accused. He is
banished, but that is intended for his protection. (d) The next
question refers to the liability of the accused-appellant for the
physical injuries suffered by Lina Amparado and Arnold
Amparado who were caught in the crossfire as the accusedappellant shot the victim. The Solicitor General recommends a
finding of double frustrated murder against the accusedappellant and being the more severe offense, proposes the
imposition of reclusion perpetua in its maximum periods
pursuant to Article 48 of the Revised Penal Code. This where we
disagree. The accused-appellant did not have the intent to kill
the Amparado couple. Although as a rule, one committing as
offense is liable for all the consequences of his act, that rule
presupposes that the act done amounts to a felony. Here, the
accused-appellant was not committing murder when he
discharges his rifle upon the deceased. Inflicting death under
exceptional circumstances is not murder.
18. PEOPLE VS. ABAGON
161 SCRA 255 (1988)
Nature: This is an appeal from the decision of the RTC, which
found the accused guilty of the crime murder.
Issue: Accused-appellant, thru their counsel, raise the following
assignments of error: (i) The Honorable Regional Trial Court
erred in finding that the prosecutions evidence consisting of the
testimony of its witnesses sufficiently established the guilt
beyond reasonable doubt of the accused for the killing of ones
Celis Lupango, which killing was qualified to murder by
treachery, (ii) The Honorable Regional Trial Court erred in
holding that the defense of alibi on the part of the accused Mateo
Abagon and of self-defense on the part of Abner Ongonion will
no lie, (iii) The Honorable Trial court erred in declaring that
conspiracy had been sufficiently established, and (iv) The
Honorable Regional Trial Court erred in finding both accused
guilty of the crime of murder considering that their guilt were
not established beyond reasonable doubt.
Facts: (1) About 6:00 oclock in the afternoon of April 17,
1981, at Barangay Pinamarubuhan, Mobo, Masbate, while the
herein victim Celis Lupango and companions Isabelo Radaza,
Jr., Benjamin Bergado and Nilo Lalaguna were inside the store
of Corazon Cana to celebrate the birthday of Isabelo Radaza, Jr.,
two persons, later identified as Mateo Abagon and Abner
Ongonion, entered and stabbed Celis Lupango. First to enter was
Abner Ongonion, followed closely behind Mateo Abagon, and
with a six-inches double bladed knife stabbed Celis Lupango
three or four times. Mateo Abagon, in turn, with a seven-inch
knife also stabbed Celis Lupango several times.
(2) After Celis Lupango fell to the ground the two accused left.
At this point Terisito Lupango, brother of Celis Lupango,
arrived and he carried Celis Lupango, with the help of Benjamin
Bergado and Nilo Lalaguna whom he found inside the store,
outside the store intending to bring him to the hospital. Outside
the store, the waiting Abner Ongonion, who was with Mateo
Abagon, Julio Ongonion, Alejandro Ongonion, Romulo
Barruga, Antonio Danao and Arnel Onarosa, he drew his firearm
and fire two shots at them. Upon being fire at, Benjamin

Bergado and Nilo Lalaguna ran away while Teresito Lupango


sought cover. Abner Ongonion and his companions approached
and they took turns in stabbing the prostate body of Celis
Lupango with bolos and knives. Terisito eventually able to
report the incident.
(3) The cause of death was established to be shock, secondary
to massive external. Hemorrhage caused by multiple stabs and
incised wounds. Testifying, Dr. Quemi admitted the possibility
that the wounds were afflicted the possibility that the wounds
were inflicted by one or two assailants.
(4) The defense presented both accused to deny the charges.
For his defense, accused Mateo Abagon claims that at the time
of the incident he was in his house at the seashore of
Pinamarubuhan about 100 meters away from the scene of the
incident. He went out of the house only he learned of the
stabbing incident when he saw many persons running towards
the scene. After seeing the lifeless body of the victim, he
returned home immediately. He did not see his co-accused
Abner Ongonion at the scene. In fact he did not see any other
person there. On the other hand, accused Abner Ongonion
claims that at that particular time he left his house to fetch his
mother at the Tugbo River where she washed clothes. On his
way he passed by the store of Corazon Cana to buy cigarettes.
At the store he was pulled inside by Celis Lupango, where the
latter was drinking with others, among who was June Radaza.
He was asked to drink but he refused because of a headache.
Celis then asked him why are you brave and then he pulled his
knife, but at as he did so the knife bumped the edge of the table
and fell to the ground. As Celis recovered the knife from the
ground, Ongonion was able to get hold of Celis hand and they
grappled for the knife. While grappling he succeeded in
thrusting the knife to the left breast of Celis and again thrust it to
the stomach. After he was able to get possession of the knife he
kept on stabbing Celis, being by then angry. In the meantime, as
they grappled, the companions of Celis Lupango jumped out of
the window, while June Radaza who was there watching ran
away when he saw Celis was stabbed. Eh then went out and
proceeded to the PC Headquarters at Masbate and surrendered.
Benjamin Bergado and Teresito Lupango were not seen by him
in the store. He then stated that his co-accused Mateo Abagon
was in the store.
Ruling: Wherefore the judgment appealed from is affirmed
except that the penalties are modified. Appellant Mateo Abagon
is sentenced to suffer an indeterminate penalty of imprisonment
from twelve years and one day of prision mayor as minimum to
eighteen years, eight months, and one day of reclusion temporal
as maximum. Appellant Abner Ongonion is sentenced to suffer
an indeterminate penalty of imprisonment from ten years and
one day of prision mayor as minimum to seventeen years, four
moths and one day of reclusion temporal as maximum. The two
accused-appellants shall pay jointly the amount of, thirty
thousand pesos to the heirs of Celis Lupango as indemnity
Rationale: (i) Appellant Ongonions theory of self-defense is
untenable. According to the testimonies of Bergado, Radaza,
and Lupango, the attack by the assailant was unprovoked. The
sudden attack on the victim with knives drawn indicates that the
stabbing was unintentional. No other conclusion can likewise be
surmised from their gunshots fired by the assailants at those who
tried to bring Celis to the hospital, while the victims body lay
helpless i=on the street, the appellants kept on stabbing the
victim, thereby ensuring his death. (ii) Having admitted the
killing Ongonion must clearly establish that he acted in selfdefense, the burden of proof is now shifted to him, he must,
therefore, rely on the strength of his own evidence and not on
the weakness of the prosecution (People vs. Sandie, 149 SCRA
240; and People vs. Regulacion) for even if the latters evidence
is weak, it could not be disbelieved after the appellant admitted
the killing. The number and nature of the stab wounds inflicted
by more than one person beloved Ongonions theory of selfdefense. These and the testimonies of two eyewitnesses and one
peace officer further serve to destroy Ongonions statement.
Moreover it is a well settled rule that the findings of the fact of
the trial court on the credibility of witnesses are generally
accorded the highest respect by the appellee court (People vs.
Traya, 147 SCRA 381) for these courts have the privileges of
examining the deportment and demeanor of witnesses, and
therefore, can discern if such witnesses, and therefore, can

discern if such witnesses are telling the truth or not (People vs.
Ramilo, 147 SCRA 102). (iii) Ongonions claim of self-defense
is likewise negated by the physical evidence and other
circumstances, such as his failure to present the knife upon
surrender, his failure to tell the police authorities that he killed
the deceased in self-defense and the absence of any injury on the
body of Ongonion while the deceased suffered eleven wounds
when, according to the appellant, there was supposedly a
struggle that tool place. For self-defense to prosper, the
following elements should have been proven by appellant: (a)
unlawful aggression, (b) reasonable necessity of the means
employed to prevent or repel it; and (c) lack of provocation on
the part of the one defending himself. On the contrary, all the
evidence on record shows that not one of the elements of selfdefense is present. (iv) Appellant Abagaons defense on the
other hand, is alibi, an inherently weak defense especially when
it can be proved that it was not physically impossible for him to
be at the scene of the crime. In order to be given full faith and
credit, alibi must not leave any room for doubt as to its
plausibility and veracity. The appellant at the time of the crime
was allegedly in a place which approximately only 100 meters
away from the scene of the crime, renders his defense of alibi
not credible. (v) More important, Abagon and his companion
were positively identifies by eyewitnesses Bergado and
Lalaguna. The records show that the appellants took turns at
stabbing the victim inside and outside the store. The presence
and location of the eleven stab wounds, as testified by Dr.
Quemi also indicate that the same were inflicted by more than
one person. (vi) The assailants acted in concerted efforts with
community of criminal purpose to ensure the death of the victim
is indicative of conspiracy between them. Conspiracy is
established by concerted action It may be noted that even if
conspiracy had not been established, the liability of the two
appellants would not change for each inflicted on his own,
multiple stabbing blows on the victim resulting in mortal
injuries. They acted as principals by direct participation. (vii)
Treachery was likewise proven by the evidence presented. The
attack was immediate, sudden and unexpected. Treachery exists
when the offender commits any crime against person, employing
means, methods or forms in the execution, without risk to him
arising from any defense which the offended party might make.
19. PEOPLE VS. IGNACIO
G.R. NO. 134563 (FEB. 2000)
Nature: Appeal on the May 18, 1998 decision of the Regional
Trial Court of Masbate, Masbate, convicting Eulogio Ignacio of
murder
Facts: On January 11, 1997 (morning) in Barangay Divisoria,
Municipality of Dimasalang, Masbate, Jessie Lacson and Edwin
Velasco were gathering shells from the seashore. They got
thirsty, went to the fishpond and get coconuts or butong. The
fishpond is owned by Cielo Cortes alias Malagring. Eulogio
Ignacio, Loloy, is the caretaker, which stays at the house
inside the fishpond. Jessie then got 1 coconut, walked ahead of
Edwin in going to the dike, to break open the coconut. Eulogio
saw Jessie as Jessie reached the dike, he did not see Edwin
walking behind Jessie. |Eulogio asked Jessie to put down the
young coconut. Jessie did. Eulogio fired his homemade shotgun
at Jessie hitting the left portion of his breast. Eulogio was meters
away fro, Jessie. Edwin was meters away. Eulogio cranked his
homemade shotgun aimed it at Edwin but did not fire. Edwin
went to Jessies parents, then to Carlit Alcove, the Baraga
Tanned. Carlit asked Eulogio to surrender which he did.
Version of the Defense: Eulogio acted on the defense of
property. On January 9, 1997, he was informed by his neighbor,
Gil Aristotles regarding a theft incident in the fishpond. On
January 11, 1997, he saw Jessie and Edwin coming out of his
house carrying a basket his house was filled with 28 crabs.
Upon seeing Eulogio, Jessie and Edwin fled. He ordered them to
stop; they did not, so he fired his gun (without intention to kill
them). He then reported incident to Kalawao Gil Aristotles.
Ruling: There was no legal reason for him to shoot the victim,
an unarmed minor; killing was qualified as murder because of
the presence of treachery.
Issue:

1st issue: Evidence of appellants guilt


For jstifyi8ng circumstances to be appreciated, the
accused has the burden of proving unlawful aggression on the
part the victim. Eulogio was not attacked by Jessie.
2nd issue: Treachery
A killing is qualified as treachery when the accused
employs means without risk to him arising from the defense
which the offended party might make.
Victim and companion stopped after
Eulogio shouted at them
Jessie was already facing him when he
shot him
There was no proof that he
deliberately and consciously adopted
any means to kill did not act on
mere impulse
3rd issue: Mitigating Circumstance
Mitigating circumstance of
because he allegedly gave himself up?

voluntary

surrender

Mitigating Circumstance:
1. offender has not been actually arrested
2. offender surrender himself to a person
of authority
3. surrender is voluntary
=No mitigating circumstance he forced to give himself up
Main issue: Whether Eulogio acted in unlawful defense of the
landowners property.
Held: Appeal is denied, assailed decision is affirmed.
20. PEOPLE VS. RICOHERMOSO
56 SCRA 431 (1974)
Nature: Severo Padernal and Juan Padernal appealed from the
decision of the Circuit Criminal Court at Lucena City,
convicting them of murder, sentencing each of them to reclusion
perpetua and ordering them to pay solitarily the sum of twelve
thousand pesos to the heirs pf Geminiano de Leon ad to pay the
costs (Criminal Case No. CCC-IX-37- Quezon or 1922-CFIGumaca).
In the same decision they were convicted of lesions levees. Each
one was sentences to suffer the penalty of fifteen (15 days of
arresto manor and to pay the costs. Rosendo Perpean, Rito
Monterey and Macario Monterey were acquitted (Criminal Case
No. CCC-IX-38- Quezon or 1923-CFI-Gumaca).
Issue: The only issue in this appeal, which concerns Juan
Padernal, is whether he conspired with Ricohermoso and Severo
Padernal to kill Geminiano de Leon.
Facts:
1.

2.

3.

At about nine oclock in the morning of


January 30, 1965 Geminiano de Leon, together
with his thirty-three-year old common-law
wife Fabian Rosales, his twenty year old son
Marian to de Leon and one Rizal Rosales,
encountered Poi Ricohermoso in Barrio
Tagabawa Silage, Catamaran, Quezon.
Geminiano owned a parcel of land in that
barrio which Ricohermoso cultivated as
kingpin. Geminiano asked Ricohermoso about
his share of the palay harvest. He added that he
should at least be allowed to taste the palay
harvested from his land. Ricohermoso
answered that Geminiano could go to his house
anytime and he would give the latter palay.
Geminiano rejoined that he could not get the
palay that morning because he was on his way
to Barrio Bag basin but, on his return, he
would stop at Ricohermoso house and get the
palay.
When Geminiano returned to Barrio Tagabawa
Silage, he stopped at Ricohermoso place. It
was about two oclock in the afternoon.

4.

5.

6.

7.

8.

9.

Geminiano sat on a sack beside Fabian Rosales


in front of the house while Marian to stood
about three meters behind his father.
Ricohermoso stood near the door of his house
while Severo Padernal was stationed near the
eaves of the house.
Geminiano asked Ricohermoso about the
palay. The latter, no longer conciliatory and
evidently hostile, answered in a defiant tone:
Whatever happens, I will not give you palay.
Geminiano restated: Why did you tell us to
pass by your house, you were not willing to
give the palay?
At that juncture, as if by pre-arrangement,
Ricohermoso unsheathed his bolo and
approached Geminiano from the left, while
Severo Padernal (Ricohermoso father-in-law)
got an axe and approached and approached
Geminiano from the right. The latter looked up
to the sexagenarian Severo Padernal, with both
hands raised and pleaded: Mama (grandpa),
why will you do this to us. We will not fight
you. While Geminiano was still looking up to
Severo Padernal on his right, Ricohermoso
walked to Guineans left, and, when about one
meter from him ,stabbed him on the nick with
his bolo. Geminiano fell face downward on the
ground. While in that helpless position, he was
hacked on the back with an axe by Severo
Padernal.
At the same place and time, while Severo
Padernal and Ricohermoso were assaulting
Geminiano de Leon, another episode was
taking place. Juan Padernal (Ricohermosos
brother-in-law and the son of Severo) suddenly
embraced Marianito de Leon from behind ,
with his right armed locked around
Marionettes neck and his left hand pressing
Marionettes left forearm. They grappled and
rolled downhill towards a camote patch.
Marianito passed out. When he regained
consciousness, his rifle was gone. He walked
uphill, saw his mortally wounded father
Geminiano in his death those, and embraced
him, He carried Geminiano for a short
distance. The fifty-one year old Geminiano
died at two oclock on that sane day.
Doctor Matundan said that the first wound was
fatal. It could have caused instantaneous death
because it was a deep wound which pierced the
carotoid artery and jugular vain. The second
wound on the back could likewise have caused
the victims death if it had penetrated the
kidney.
Doctor Matundan found that Marianito de
Leon sustained multiple abrasions on the nick
and abdomen and a lacerated wound on the left
foot which would heal from one to nine days
even without medical treatment.
Appellants version is that in the afternoon of
January 30, 1965, when Ricohermoso refused
to give any palay to Geminiano de Leon,
because the land tilled by the former as
allegedly a public land, Geminiano approached
Ricohermoso. When Geminiano unsheathed
his bolo, Ricohermoso met him drew his bolo
and struck Geminiano on the left side of the
neck. The latter tried to parry the blow. He was
wounded in the wrist. As Geminiano turned
right to flee, Ricohermoso struck him again on
the left side of the body, causing him to fall on
the ground. Geminiano died on the spot due to
the bleeding from the wound on his neck.
While Geminiano was being assaulted, his son
Marianito tried to shoot with his rifle but Juan
Padernal disabled him and wrested the gun.
Marianito suffered abrasions on the neck and
other parts of the body.

10. The appellants filed their brief on February 6,


1970. Later, Severo Padernal withdrew his
appeal
Ruling: Wherefore, the judgment of the lower court as to
appellant Juan Padernal is affirmed with costs against him.
Rationale:

The trial court rationalized its conclusion that there


was conspiracy by stating that their conduct revealed
unity of purpose and a concerted effort to encompass
Geminiano death.

Appellant Juan Padernal invokes the justifying


circumstance of avoidance of a greater evil or injury in
explaining his act of preventing Marianito de Leon
from shooting Ricohermoso and Severo Padernal. His
reliance on that justifying circumstance is erroneous.
The act of Juan Padernal in preventing Marianito de
Leon from shooting Ricohermoso and Severo
Padernal, who were the aggressors, was designed to
insure the killing of Geminiano de Leon without any
risk to his assailants.

Juan Padernal was not avoiding any evil when he


sought to disable Marianito. Adrenals malicious
intention was to forestall any interference in the
felonious assault made by his father and brother-inlaw on Geminiano. That situation is unarguably not
the case envisaged in paragraph 4 of article 11.

It should be recalled that, in the morning, Geminiano


had an understanding with Ricohermoso that he would
return in the afternoon to get his share of the palay
harvest Ricohermoso gave Geminiano the impression
that he was amenable to giving Geminiano his share of
the harvest. However, during the interval,
Ricohermoso changed his mind. Instead of remaining
steadfast to his original intention to give Geminiano
palay, Ricohermoso planned with his father in law,
Severo Padernal, the manner of liquidating Demonian
so as to stop him from pestering Ricohermoso with
demands for a share in the harvest. So, when
Geminiano reappeared at Ricohermosos place in the
afternoon, Severo Padernal, Ricohermoso, Juan
Padernal, like actors in a well rehearsed play,
performed their assigned roles with dramatic
positions. Severo Padernal and Ricohermoso, one
armed with an axe and the other with a bolo, in a
pincer movement, confronted Geminiano de Leon.
Simultaneously with that maneuver, the 35 year old
Juan Padernal embraces Marianito de Leon and
prevented him from firing at Severo Padernal and
Ricohermoso or from helping his father.

Considering the trios behavior and appellant Juan


Adrenals close relationship to Ricohermoso and
Severo Padernal ineluctable conclusion is that he acted
in conspiracy with them. The circumstances
surrounding the killing of Geminiano de Leon alevosia
and treachery. His hands were raised and he was
pleading for mercy with Severo Padernal, when
Ricohermoso struck him on the neck with a bolo.
21. PEOPLE VS. BERONILLA
96 PHIL 566 (1955)
Nature: This is an appeal by accused Manuel Beronilla,
Policarpio Paculdo, Filipino Velasco, and Jacinto Adriatico from
the judgment of the Court of First Instance of Abra (Criminal
Case No. 70) convicting them of murder for the execution of
Arsenio Borjal in the evening of April 18, 1945, in the town of
La Paz , Province of Abra.
Facts: Arsenio Borjal was the elected mayor of La Paz, Abra, at
the outbreak of war, and continued to serve as Mayor during the
Japanese occupation, until March 10, 1943, when he moved to
Bangued because of an attempt upon his life by unknown
persons. On December 18, 1944, appellant Manuel Beronilla
was appointed Military Mayor of La Paz by Lt. Col. R. H.
Arnold, regimental commander of the 15th Infantry, Philippine
Army, operating as a guerrilla unit in the province of Abra.
Simultaneously with his appointment as Military Mayor,

Beronilla received copy of a memorandum issued by Lt. Col.


Arnold to all Military Mayors in Northern Luzon, authorizing
them "to appoint a jury of 12 bolomen to try persons accused of
treason, espionage, or the aiding and abetting (of ) the enemy"
(Exhibit 9). He also received from the Headquarters of the 15th
Infantry a list of all puppet government officials of the province
of Abra (which included Arsenio Borjal, puppet mayor of La
Paz), with a memorandum instructing all Military Mayors to
investigate said persons and gather against them complaints
from people of the municipality for collaboration with the
enemy (Exhibit 12-a).
Sometime in March, 1945, while the operations for the
liberation of the province of Abra were in progress, Arsenio
Borjal returned to La Paz with his family in order to escape the
bombing of Bangued. Beronilla, pursuant to his instructions,
placed Borjal under custody and asked the residents of La Paz to
file complaints against him. In no time, charges of espionage,
aiding the enemy, and abuse of authority were filed against
Borjal; a 12-man jury was appointed by Beronilla, composed of
Jesus Labuguen as chairman, and Benjamin Adriatico, Andres
Afos, Juanito Casal, Santiago Casal, Benjamin Abella,
Servillano Afos, Mariano Ajel, Felimon Labuguen, Felix
Murphy, Pedro Turqueza, and Delfin Labuguen as members;
while Felix Alverne and Juan Balmaceda were named
prosecutors, Policarpio Paculdo as clerk of the jury, and Lino
Inovermo as counsel for the accused. Later, Atty. Jovito
Barreras voluntarily appeared and served as counsel for Borjal.
Sgt. Esteban Cabanos observed the proceedings for several days
upon instructions of Headquarters, 15th Infantry. The trial lasted
19 days up to April 10, 1945; the jury found Borjal guilty on all
accounts and imposed upon him instruction from his superiors.
Mayor Beronilla forwarded the records of the case to the
Headquarters of the 15th Infantry for review and on the night of
the same day, April 18, 1945, Beronilla ordered the execution of
Borjal. Jacinto Adriatco acted as executioner and Anotnio
Palope as grave digger.
Two years thereafter, Manuel Beronilla as military mayor,
Policarpio Paculdo as Clerk of the jury, Felix Alverne and Juan
Balmaceda as prosecutors, Jesus Labuguen, Delfin Labuguen,
Filemon Labuguen, Servillano Afos, Andres Afos, Benjamin
Adriatico, Juanito Casel, Santiago Casel, Mariano Ajel, Felix
Murphy, Benjamin Abella, and Pedro Turqueza as members of
the jury, Jacinto Adriatico as executioner, Severo Afos as grave
digger, and Father Filipino Velasco as an alleged conspirator,
were indicted in the Court of First Instance of Abra for murder,
for allegedly conspiring and confederating in the execution of
Arsenio Borjal. Soon thereafter, the late President Manuel A.
Roxas issued Executive Proclamation No. 8, granting amnesty to
all persons who committed acts penalized under the Revised
Penal Code in furtherance of the resistance to the enemy against
persons aiding in the war efforts of the enemy. Defendant Jesus
Labuguen, then a master sergeant in the Philippine Army,
applied for and was granted amnesty by the Amnesty
Commission, Armed Forces of the Philippines (Records, pp.
618-20). The rest of the defendant filed their application for
amnesty with the Second Guerrilla Amnesty Commission, who
denied their application on the ground that the crime had been
inspired by purely personal motives, and remanded the case to
the Court of First Instance of Abra for trial on the merits.
Issue: The crucial question thus becomes whether or not this
message, originally sent to Arnold's quarters in San Esteban,
Ilocos Sur, was relayed by the latter to appellant Beronilla in La
Paz, Abra, on the morning of April 18, 1945, together with the
package of records of Borjal's trial that was admittedly returned
to and received by Beronilla on that date, after review thereof by
Arnold (Exhibit 8-8-a). Obviously, if the Volckmann message
was known to Beronilla, his ordering the execution of Borjal on
the night of April 18, 1945 can not be justified.
Ruling: The judgment appealed from is reversed and the
appellants are acquitted with costs de officio.
Rationale:

The records are ample to sustain the claim of the


defense that the arrest, prosecution and trial of
the late Arsenio Borjal were done pursuant to
express orders of the 15th Infantry Headquarters.
(Exhibit 9 and 12-a), instructing all military
mayors under its jurisdiction to gather evidence
against puppet officials and to appoint juries of at

10

least 12 bolomen to try the accused and find them


guilty by two thirds vote. It is to be noted that
Arsenio Borjal was specifically named in the list
of civilian officials to be prosecuted (Exhibit 12b).
In truth, the prosecution does not seriously
dispute that the trial and sentencing of Borjal was
done in accordance with instructions of superior
military authorities, although it point to
irregularities that were due more to ignorance of
legal processes than personal animosity against
Borjal. The state, however, predicates its case
principally on the existence of the radiogram
Exhibit H from Col. Volckmann, overall area
commander, to Lt. Col. Arnold, specifically
calling attention to the illegality of Borjal's
conviction and sentence, and which the
prosecution claims was known to the accused
Beronilla.
We have carefully examined the evidence on this
important issue, and find no satisfactory proof
that Beronilla did actually receive the radiogram
Exhibit H or any copy thereof. The accused
roundly denied it. The messenger, or "runner",
Pedro Molina could not state what papers were
enclosed in the package he delivered to Beronilla
on that morning in question, nor could Francisco
Bayquen (or Bayken), who claimed to have been
present at the delivery of the message, state the
contents thereof.
The plain import of the affidavit is that the
witness Rafael Balmaceda was not with Beronilla
when the message arrived, otherwise Beronilla
would have given him his orders direct, as he
(Balmaceda) testified later at the trial. Moreover,
it is difficult to believe that having learned of the
contents of the Volckmann message, Balmaceda
should not have relayed it to Borjal , or to some
member of the latter's family, considering that
they were relatives. In addition to Balmaceda was
contradicted by Bayken, another prosecution
witness, as to the hatching of the alleged
conspiracy to kill Borjal. Balmaceda claimed that
the accused-appellants decided to kill Borjal in
the early evening of April 18, while Bayken
testified that the agreement was made about ten
o'clock in the morning, shortly after the accused
had denied Borjal's petition to be allowed to hear
mass.
Our conclusion is that Lt. Col. Arnold, for some
reason that can not now be ascertained, failed to
transmit the Volckmann message to Beronilla.
And this being so, the charge of criminal
conspiracy to do away with Borjal must be
rejected, because the accused had no need to
conspire against a man who was, to their
knowledge, duly sentenced to death.

22. TABUENA VS. SANDIGANBAYAN


268 SCRA 332 (1997)
Nature: This is a separate petition to review the decision of the
Sandiganbayan dated October 12, 1990 convicting them of
malversation under Article 217 of the Revised Penal Code as
well as the Resolution dated December 20, 1991 denying
reconsideration
3 criminal cases were filed against Tabuena who
appears as the principal accused
and one for Peralta since
the total amount of 55 million pesos was taken on 3
separate dates of January, namely 10, 16, and 29. They
were convicted of malversation under Article 217 of the
Revised Penal Code.
Facts:

Then President Marcos instructed Tabuena over the


phone to pay directly to the Presidents office and in
cash what the MIAA owes the PNCC to which
Tabuena replied. Yes, sir. I will do it.

On January 8, 1986, one week after the phone


conversation Tabuena received from Mrs. Gimenez,
the personal secretary of Pres. Marcos, a Presidential
Memorandum reiterating in black and white the verbal
instruction of the President.
In obedience to President Marcos verbal instruction
and Memorandum, Tabuena, with the help of Dabao
and Peralta caused the release of 55 million pesos of
MIAA funds by means of 3 withdrawals.
1st withdrawal was made on January 10, 1986 for P25
million following a letter of even date signed by
Tabuena and Dabao requesting the PNB extension
office at the MIAA depository branch of MIAA funds
to issue a manager check for said amount payable to
Tabuena and was encashed at the PNB Villamor
Branch. Subsequently, the P25 million in cash was
delivered by Tabuena with the use of an armored car
of the PNB to the office of Mrs. Gimenez located at
Aguado St. fronting Malacanang. There was no receipt
issued for the delivery of the money.
Similar circumstances surrounded the second
withdrawal/encashment and delivery of another P25
million made on January 16, 1986.
The 3rd and last withdrawal was made on January 31,
1986 for P5 million. Peralta was Tabuenas cosignatory to the letter-request for the managers check
for the amount Peralta did not go with Tabuena to
deliver the money to the office of Mrs. Gimenez. It
was only upon this delivery that Mrs. Gimenez issued
a receipt, dated January 30, 1986, for all the amounts
she received from Tabuena.

Issue:
1.
2.
3.

Whether the Sandiganbayan erred in


convicting petitioners of a crime not charged
in the amended informations
Whether the defense of Good Faith of the
petitioners relieve then from the crime of
malversation, and;
Whether the petitioners constitutional rights
to due process was violated

Held:

1.

2.

No. It is stated in Cabello vs. Sandiganbayan even on


putative assumption that the evidence against
petitioner yielded a case of malversation by
negligence but the information was for intentional
malversation, under the circumstances of this case his
conviction under the first mode of misappropriation
would still be in order. Malversation is committed
either intentionally or by negligence. The dolo or the
culpa present in the offense is only a modality in the
perpetration of the felony. Even if the mode charged
differs from the mode proved, the same offense of
malversation is involved and conviction thereof is
proper. Moreover, SEC. 5 Rule 116 of the Rules of
court does not require that all the essential elements of
the offense charged in the information be proved, it
being sufficient that some of said essential elements or
ingredients thereof be established to constitute the
crime proved.
Yes. Firstly, Marcos was undeniably. Tabuenas
superior and as a recipient of such kind of a directive
coming from the highest official of the land no less,
GOOD FAITH should be read on Tabuenas
compliance, without hesitation nor any question, with
the MARCOS Memorandum. The superiorsubordinate relationship was clearly established and so
is the lawfulness of the order contained in the
MARCOS Memorandum for its purpose partial
payment of the liability of one government agency
(MIAA) to another (PNCC). Tabuena is therefore
entitled to the justifying circumstance of Any person
who acts in obedience to an order issued by a superior
for some lawful purpose. Secondly, there is no
denying that the disbursement did not comply with
certain auditing rules and regulations. But this

11

3.

deviation was inevitable under the circumstances


Tabuena was in because he did not have the luxury of
time to observe all auditing procedures of
disbursement considering the last that the MARCOS
Memorandum enjoined his immediate compliance
with the directive that he forwarded to the Presidents
office P55 million in cash. Tabuena surely cannot
escape responsibility for the emission but since he was
acting in good faith, his liability should only be
administrative or civil in nature and not criminal.
Thirdly, the Sandiganbayan made the finding that
Tabuena had already converted and misappropriated
the P55 million when he delivered the same to Mrs.
Gimenez and not to the PNCC however it was stated
in the memorandum to pay immediately the PNCC,
thru his office, the sum of 55 million pesos and that
was what Tabuena precisely did when he delivered the
money to the Presidents office thru his secretary Mrs.
Gimenez. Therefore, Tabuenas good faith in
delivering the money to the President in strict
compliance with the memorandum was not at all
affected even if it later turned out that PNCC never
received the money. Fourthly, even assuming that the
real and sole purpose behind the MARCOS
Memorandum was to siphon-out public money for
personal benefit of those then in power still, no
criminal liability can be imputed to Tabuena. There is
no showing that he had anything to do whatsoever
with the execution of the MARCOS Memorandum.
There is also no proof that he profited from the
felonious scheme in short, no conspiracy was
established between Tabuena and the real embezzlers
of the P55 million. This is not a sheer case of blind
and misguided obedience, but obedience in good faith
of a duly executed order.
Yes. The cold neutrality of an impartial judge is an
essential requirement for due process because it would
show that the courts are impartial, and unbiased.
Moreover, the trial judge has the right to question
witness in view of satisfying his mind upon any
material point which presents itself during the trial o a
case over which he presides but it is limited only to
clarifactory questions only. However, the appellate
court, while going over the records, noticed that the
way the Sandiganbayan actively took part in the
questioning of a defense witness and of the accused
themselves and basing on its manner indicated
prejudgment of guilt, bias, hatred, or hostility against
the said appellants. It would be noticed by the volume
of questions hurled by the Sandiganbayan and more
importantly, it is noticed that the questions were in the
nature of cross-examinations characteristic of
confrontation, probing, and insinuation.

Decision: WHEREFORE, in view of the foregoing herein


petitioners Luis A. Tabuena and Adolfo M. Peralta are hereby
ACQUITTED of the crime of malversation as defined and
penalized under Article 217 of the Revised Penal Code The
Sandiganbayan Decision of
October 12, 1990 and the
Resolution dated December 20, 1991 are REVERSED and SET
ASIDE.
23. PEOPLE VS. PUNO
105 SCRA 151 (1991)
Nature: Automatic Review from the decision of Circuit
Criminal Court of Pasug, Rizal.
FACTS:
At about two oclock in the afternoon of September 8, 1970,
Ernesto Puno, 28, a jeepney driver, entered a bedroom in the
house of Francisca Col (Aling Kikay), 72, a widow. The house
was located in the area known as Little Baguio, Barrio
Tinajeros, Malabon, Rizal. On seeing Aling Kikay sitting in bed,
Puno insulted her by saying Mangkukulam ka mambabarang.
Mayroon kang bubuyog. Then, he repeatedly slapped her and
struck her several times on the head with a hammer until she
was dead. The assault was witnessed by Hilaria de la Cruz, 23,
who was in the bedroom with the old woman, and by Lina Pajes,

27, a tenant of the adjoining room. They testified that Punos


eyes were reddish. His look was baleful menacing.
After the killing Puno went to the room of Lina where Hilaria
had taken refuge and according to Hilaria, he made the
following
confession
and
threat:
Huwag
kayong
magkakamaling tumawag ng pulis at sabihin ninyo na umalis
kayo ng bahay at hindi ninyo alam kung sino ang pumatay sa
matanda. And according to Lina, Puno said: Pinatay ko iyong
matanda. Huwag kayong tumawag ng pulis. Pag tumawag kayo
ng pulis kayo ang paghihigantihan ko.
Puno, a native of Macabebe, Pampanga, who testified about five
months after the killing, pretended that he did not remember
having killed Aling Kikay. He believes that there are persons
who are mangkukulam mambabarang and mambubuyog
and that when one is victimized by those persons, his feet might
shrink of his hands might, swell. Puno believes that a person
harmed by a mambabarang might have a headache or a
swelling nose and ears and can be cured only by a quack doctor
(herbolaryo). Consequently, it is necessary to kill the
mangkukulam and mambabarang.
The trial court concluded that if Puno was a homicidal maniac
who had gone berserk, he would have killed also Hilaria and
Lina. The fact that he singled out Aling Kikay signified that he
really disposed of her because he thought that she was a witch.
RULING: WHEREFORE, the death penalty is set aside. The
accused is sentenced to reclusion perpetua. The indemnity
imposed by the trial court is affirmed. Costs de oficio.
24. PEOPLE VS. PAMBID
G.R. NO. 124453 (MARCH 2000)
Nature: Appeal from a decision of the Regional Trial Court of
Quezon City Branch 81
Facts: This an appeal from the decision of the Regional Trial
Court Branch 81 Quezon City, finding the accused appellant
Jose C. Pambid guilty beyond reasonably doubt of two counts of
statutory rape and sentencing him to suffer the penalty of
reclusion perpetua with all the accessibly penalties and to
indemnify the victim Maricon Delvie C. Grifalda at P50, 000
plus costs of each count of rape. Maricon told her mother on the
two incidents of rape, that sometime in April and May 1993,
when she was on the way home after having been told for an
errand, the accused appellant, their neighbor, brought her to his
house, and raped her. The accused appellant threatened to beat
her if she would not comply and used a deadly weapon. During
the time wherein the accused was already raping the victim, they
were interrupted by the knock on the door by the mother of the
accused. The other incident of rape happened on one morning
when Maricon was going on the nearby store where there was
no other people around. Joseph then carried her to the sofa of his
house and raped her. After the accused was arrested, he pleaded
not guilty and use alibi as means I escape by saying that he was
not in his mothers house at that time but in Caloocan City on
his fathers house.
Issue: Whether or not the accused appellant is guilty of two
counts of rape.
Held: The Supreme Court held that on the first error assign by
the accused that the victim is inconsistent of her statement. The
Supreme Court held that this contention is without merit
considering the revelation made by Maricon and her voluntary
submission of the medical examination and willingness to
undergo public trial where she was compelled to give details of
the assault on her dignity. Her testimony was given respect by
The Supreme Court considering his consistency and composure
during the interrogation was commendable. Secondly, the
defense that he was not in his mothers was not given credit. The
defense of insanity that the accused was suffering schizophrenia
and mind retardation was mere concoctions because he was not
able to establish that indeed he was deprived of intelligence or
freedom of will. Lastly, the decision of the trial court in
rendering the accused appellant in rendering the penalty
reclusion perpetua with the use of the deadly weapon
considering that it is punishable by reclusion perpetua to death,
this is in consideration also on the suspension of the death
penalty at the time that the crime was committed. The Supreme

12

Court said that the trial court was wrong with this decision and
since the accused did not object of charging two counts of rape
in one information. The trial court failed to consider that the
accused did not object to the validity of the information or raise
the issue of duplicity of offense since the information does not
charge him with more than one offense or occasion of rape.
Decision: WHEREFORE, the decision of the RTC Branch 81.
Quezon City is set aside and another one is rendered finding
accused appellant Joseph Pambid guilty of one count of rape,
sentencing him to suffer the penalty of reclusion perpetua and
order him to pay Maricon P50, 000 as civil indemnity and the
same amount on moral damages.
25. PEOPLE VS. DOQUENA
68 PHIL 580 (1939)
Nature: Appeal from an order of the Court of First Instance of
Pangasinan which held that the accused acted with discernment
in committing the act imputed to him and ordered him sent to
the Training School for Boys to remain therein until he reaches
the age of majority.
Facts: Between 1 and 2 oclock in the afternoon, Juan Ragojos
and Epifanio Rarang were playing volleyball in the yard of the
intermediate school. Valentin, who was also playing there
intervened and catching the ball, tossed it to Juan Ragojos
hitting him in the stomach. For his act, Juan chased Valentin
upon overtaking him slapped him on the nape. Valentin then
turned against Juan with a threatening attitude, so Juan struck
him on the mouth with his fist. Juan then returned and continued
playing with Epifanio. Valentin was offended and looked around
for a stone to attack Juan. He found none so he approached a
cousin named Romualdo Cecal to lend him his knife.Valentin
approached Juan and challenged the latter to give him another
blow with his fist to which Juan answered that he did not want
to because he was bigger than Valentin. Juan ignored Valentin
and continued playing. Valentin then stabbed him in the chest
Issue: Whether or not Valentin acted with discernment
Held: Decision is affirmed.
Points to Ponder:

Discernment is the mental capacity to


understand the difference between right and
wrong and such capacity may be known and
should be determined by taking into
consideration all the facts and circumstances
afforded by the records in each case, the
very appearance, the very attitude, the very
comportment of the said minor, not only
before and during the act, but also after and
even during trial.
26. PEOPLE VS. LORENO
130 SCRA 311 (1984)
Nature: Appeal on the conviction of the Court of First Instance
of Camarines Sur charging Eustaquio Malaga and Jimmy
Marantal of Robbery with Double Rape.
Facts: On the evening of January 7, 1978, Barangay Captain
Elias Monge was at his house with his two daughters, Monica,
14 years old and Cristina, 22 years old. His wife, Beata Monge,
was still changing the diaper of baby Rachel Baybayon. The
other occupants of their house that evening were Mario, 11 years
old. Nilo, 13 years old and farm helper, Francisco Fable.
At around 7:40pm, four men with flashlights
approached their house calling Elias saying that there was a
letter for the chief. Elias then invited the man with dark sweater
to come inside the sala. Monica and Elias the read the letter
Kami ang NPA. Elias was poked with a gun and they were all
ordered to stay on the floor. Fable the recognized one of the men
was Eustaquio Loreno. Loreno then tied him with rattan. Fable
also recognized Jimmy Marantal as one of the lookouts.
The man in the dark sweater then dragged Monica to
her room and succeeded in raping her. Below in the sala they
could hear Monicas shouts for help, but Loreno pointed hid gun

at them telling them not to rise off if they wanted to live. Loreno
then brought Beata to the other rooms to get other contents.
Then the man in the dark sweater returned and took Cristina and
also raped her in one of the rooms. Moreover, Loreno entered
the room and embraced her trying to kiss her and touch her
private parts.
The lower court found Loreno guilty of robbery with
double rape sentencing him to life imprisonment. Jimmy
Marantal was found guilty of robbery sentencing him to prison
correccional as minimum of prison mayor.
Issue: Whether or not the accused Loreno and Marantal were
acting on irresistible force or uncontrollable fear
Held: All facts demonstrated the voluntary participation and
conspiracy of the appellants. All the conspirators are liable as
co-principals regardless of the extend character of their
participation because in the contemplation of law, the act of one
is the act of all. Decision of the court is modified. Jimmy
Marantal is sentenced to life imprisonment as well.
27. PEOPLE VS. FORONDA
222 SCRA 71 (1993)
Nature:A review on the appealed decision of the Regional Trial
Court of Cagayan finding the accused guilty beyond reasonable
doubt for the crime of murder against the brothers Esminio and
Edwin Balaan.
Facts: At about 6:00am, June 11, 1986, the deceased Balaan
brothers were taken by 7 armed men in fatigue with long
firearms suspected to be NPA members, accompanied by the
accused Rudy Fronda and Roderick Padua from the house of one
Ferminio Balaan, at Brgy. Cataratan, Allacapan, Cagayan. Rudy
Fronda and Roderick Padua were residents of the same
places.The armed men tied the hands of the deceased at their
backs, in front of their house.The armed men together with
Fronda and Padua proceeded towards sitio Cataratan, Allacapan,
Cagayan passing through the rice fields (taking along with them
the Balaan brothers).
Trial court found Fronda guilty as a principal by indispensable
cooperation.
(Testimony of Rudy Fronda)
On the night of June 10, 1986, he was taken by the NPA from
his house, accompanied by Robert Peralta, alias Ka Jun and
Roderick Padua, to look for the Balaan brothers.
There were around 9 NPAs with them. They found the Balaan
brothers at the house of Ferminio Balaan, a brother.
They tied their wrists/hands and brought them to the mountains
of Sitio Tulong, Cataratan, Allacapan, Cagayan.
After that, the NPA instructed them to go home, but the in
afternoon of the same day, Robert Peralta, alias Ka Jun sent
Elmer Martinez, Orlando Gonzales, George Peralta, and Librado
Duran to get him and further he was ordered to get a spade and a
crowbar. They were ordered to dig a hole in the mountain, one
kilometer away from his house.
Appellant interposes the exempting circumstance of
uncontrollable fear (Art 12 [6] RPC) claiming that all his acts
were performed under the impulse of uncontrollable fear and to
save his life.
Issue: W o N the accused-appellants acts make him liable for a
principal by indispensable cooperation.
Held: Decision modified. Accused-appellant could only be
convicted as an accomplice of the crime. No incontrovertible
proof was adduced by the prosecution supporting the conclusion
that the appellant agreed with the members of the armed group
to kill the Balaan brothers. Undoubtedly, even without
appellants participation, the assailants could have easily located
the Balaan brothers through the assistance of Roderick Padua.
Taking account the numbers of the assailants alone, it is
apparent that the armed men could have nevertheless committed
the crime easily without the appellant abetting the commission
thereof. As aforesaid to be considered principal by indispensable
cooperation, there must be direct participation in the criminal
design by another act without which the crime could have not
been committed.

13

However, appellants act of joining the armed men in going to


the mountains, and his failure to object to their unlawful orders,
or show any reluctance in obeying the same, may be considered
as circumstances evincing his concurrence with the objectives of
the malefactors and had effectively supplied them with material
and moral aid, thereby, making him as an accomplice.
28. PEOPLE VS. BANDIAN
63 PHIL 530 (1936)
Nature: Charged with the crime of infanticide, convicted
thereof and sentenced to reclusion perpetua and the
corresponding accessory penalties, with the costs of the son.
Josefina Bandian appealed from said sentence.
Facts: At a0bout 7 oclock in the morning of January 31, 1936,
Valentin Aguilar, the appellants neighbor, saw the appellant go
to a thicket about four or five brazas from her house, apparently
to respond to a call of nature because it was there that the people
of the place used to go for that purpose. A few minutes later, he
again saw her emerge from the thicket with her clothes stained
with blood both in the front and back, staggering and visibly
showing signs of not being able to support her. He ran to her and
said, having noted that she was very weak and dizzy, he
supported and helped her go up to her house and placed her in
her own bed. Upon being asked before Aguilar brought her to
her house, what had happened to her, the appellant merely
answered that she was very dizzy. Not wishing to be alone with
the appellant in such circumstances. Valentin Aguilar called
Adriano Comcom, who lived nearby, to help them, and later
requested him to take bamboo leaves to stop the hemorrhage
which had come upon the appellant. Comcom had scarcely gone
about five brazas when he saw the body of newborn baby near a
path adjoining the thicket where the appellant had gone a few
moments before Comcom informed Aguilar of it and the latter
told him to bring the body to the appellants house. Upon being
asked whether the baby which had just been shown to her was
hers or not, the appellant answered in the affirmative.
Upon being notified of the incident of 2 oclock in the afternoon
of said day, Dr. Emilio Nepomuceno, president of the sanitary
division of Talisayan, Oriental Misamis, went to the appellants
house and found her lying in bed still bleeding. Her bed, the
floor of her house and beneath it, directly under the bed, was full
of blood. Basing his opinion upon said facts, the physician in
question declared that the appellant gave birth in her house and
in her own bed; that after giving birth she threw her child into
the thicket to kill it for the purpose of concealing her dishonor
from the man, Luis Kirol, with whom she had theretofore been
living martially, because the child was not but of another man
with whom she had previously had another relations. To give
force to his conclusions, he testified that the appellant had
admitted to turn that she had killed her child, when he went to
her house at the time and on the date above-stated.
Ruling: In conclusion, taking into account the foregoing facts
and considerations, and granting that the appellant was aware of
her involuntary childbirth in the thicket and that she later to take
her child therefore, having been so prevented by reason of
causes independent of her will, it should be held that the alleged
errors attributed to the lower court by the appellant are and it
appearing that under such circumstances said appellant has the
fourth and seventh exempting circumstances in her favor, she is
hereby acquitted of the crime of which she had been accused
and convicted with costs de oficio, and as she is actually
confined in jail in connection with the case it is ordered that she
be released immediately.

29. PEOPLE VS. URAL


56 SCRA 138 (1974)
NATURE: An appeal of defendant Domingo Ural from the
decision of Judge Vicente G. Ericta of the Court of First
Instance of Zamboanga del Sur, convicting him of murder,
sentencing him to reclusion perpetua, and ordering him to
indemnify the heirs of Felix Napola in the sum of twelve
thousand pesos and to pay the costs.

FACTS: Upon arrival of Brigido Alberto in the municipal


building at around eight o'clock, he witnessed an extraordinary
occurrence. He saw Policeman Ural (with whom he was already
acquainted) inside the jail. Ural was boxing the detention
prisoner, Felix Napola. As a consequence of the fistic blows,
Napola collapsed on the floor. Ural, the tormentor, stepped on
his prostrate body.
Ural went out of the cell. After a short interval, he returned with
a bottle. He poured its contents on Napola's recumbent body.
Then, he ignited it with a match and left the cell. Napola
screamed in agony. He shouted for help. Nobody came to succor
him.
Much perturbed by the barbarity which he had just seen, Alberto
left the municipal building. Before his departure, Ural cautioned
him: "You better keep quiet of what I have done" (sic). Alberto
did not sleep anymore that night. From the municipal building,
he went to the crossing, where the cargo trucks passed. He
hitchhiked in a truck hauling iron ore and went home.
Doctor Luzonia R. Bakil, the municipal health officer, certified
that the thirty-year old victim, whom she treated twice, sustained
second-degree burns on the arms, neck, left side of the face and
one-half of the body including the back (Exh. A). She testified
that his dermis and epidermis were burned. If the burns were not
properly treated, death would ensue from toxemia and tetanus
infection. "Without any medical intervention", the burns would
cause death", she said. She explained that, because there was
water in the burnt area, secondary infection would set in, or
there would be complications.
Napola died on August 25, 1966. The sanitary inspector issued a
certificate of death indicating "burn" as the cause of death (Exh.
B).
ISSUE: Whether or not the accused committed the crime as
guilty beyond reasonable doubt?
HELD: Lack of intent to commit so grave a wrong offsets the
generic aggravating, circumstance of abuse of his official
position. The trial court properly imposed the penalty of
reclusion perpetua which is the medium period of the penalty
for murder (Arts. 64[4] and 248, Revised Penal Code).
Finding no error in the trial court's judgment, the same
is affirmed with costs against the appellant.
RULING: This case is covered by article 4 of the Revised Penal
code which provides that "criminal liability shall be incurred by
any person committing a felony (delito) although the wrongful
act done be different from that which he intended". The
presumption is "that a person intends the ordinary consequences
of his voluntary act" (Sec. 5[c], Rule 131, Rules of Court).
There is a rule that "an individual who unlawfully inflicts
wounds upon another person, which result in the death of the
latter, is guilty of the crime of homicide, and the fact that the
injured person did not receive proper medical attendance does
not affect the criminal responsibility" (U.S. vs. Escalona, 12
Phil. 54). In the Escalona case, the victim was wounded on the
wrist. It would not have caused death had it been properly
treated. The victim died sixty days after the infliction of the
wound. It was held that lack of medical care could not be
attributed to the wounded man. The person who inflicted the
wound was responsible for the result thereof.
The crime committed by appellant Ural was murder by means of
fire (incendio) (Par. 3, Art. 248, Revised Penal Code; People vs.
Masin, 64 Phil. 757; U.S. vs. Burns, 41 Phil. 418, 432, 440).
The trial court correctly held that the accused took advantage of
his public position (Par. 1, Art. 14, Revised Penal Code). He
could not have maltreated Napola if he was not a policeman on
guard duty. Because of his position, he had access to the cell
where Napola was confined. The prisoner was under his
custody. "The policeman, who taking advantage of his public
position maltreats a private citizen, merits no judicial leniency."
The methods sanctioned by medieval practice are surely not
appropriate for an enlightened democratic civilization. While the
law protects the police officer in the proper discharge of his
duties, it must at the same time just as effectively protect the
individual from the abuse of the police." U.S. vs. Pabalan, 37
Phil. 352).
But the trial court failed to appreciate the mitigating
circumstance "that the offender had no intention to commit so
grave a wrong as that committed" (Par. 3, Art. 13, Revised Penal

14

Code). It is manifest from the proven facts that appellant Ural


had no intent to kill Napola. His design was only to maltreat him
may be because in his drunken condition he was making a
nuisance of himself inside the detention cell. When Ural realized
the fearful consequences of his felonious act, he allowed Napola
to secure medical treatment at the municipal dispensary.
30. PEOPLE VS. REGATO
127 SCRA 287 (1984)
Nature: For automatic review is death sentence imposed on
accused-appellants Miguel Regato and Jose Salceda by the then
Court of First Instance of Leyte, Branch IV, in Criminal Case
No.12, entitled People vs. Miguel Regato, et al., for robbery
with homicide. They were also ordered to indemnify, jointly and
severally, the heirs of Victor Flores the sum of P 12,000.00; the
further sum of P 8,000.00 and each to pay one third of the costs.
Issue: In this appeal, appellants contend that the trial court erred
(1)when it denied Salcedos motion for new trial and did not
acquit him of the crime charged; (2) in convicting Regato of
robbery with homicide and not with simple robbery; (3) in not
considering in their favor the mitigating circumstance of lack of
intent to commit so grave a wrong as that committed; (4) in
considering the aggravating circumstance of nocturnity against
them and (5) in failing to consider that the aggravating
circumstance of craft is absorbed by the aggravating
circumstance of nocturnity.
Facts: About nine oclock in the evening of November 22,
1969, three persons called at the house of Victor Flores at Sitio
Macaranas, Bo. Capirawa, Palo Leyte asking if they could buy
cigarettes. Felicisima Flores, wife of Victor, was then
maintaining a small sari-sari store inside their house. Upon
hearing them, she stood up and after lighting a small kerosene
lamp, opened the door of the house and extended the lamp out to
recognize the persons outside. She saw accused Miguel Regato
who was then at the porch and Jose Salceda.
As she kept on exposing the light at them, Regato approached
Felicisima and struck her hand holding the lamp, causing it to
fall. Regato then pointed a gun at Felicisima who moved
backwards, towards the kitchen after which she jumped out and
ran to the house of Filomeno Pilmaco, a neighbor. She asked for
help and was told to sty in the house while he and his
companions would rush to poblacion of Palo to inform the
police if the incident. After Pilmaco and his companions had
left, Felicisima heard a gun explosion from the direction of their
house.
In the meantime, Godofredo Flores, the 12 years old son of
Felicisima, who was sleeping in the sala, was awakened by the
voice of the robbers. He observed that his mother was not in the
house but his father was being dragged down the stairway by
Rito Ramirez and Miguel Regato. He saw also appellant Salceda
then lighted the lamp which was then on the floor of the sala of
the house and then he brought Florencio inside the bedroom
where Godofredo was then hiding. Rito Ramirez and appellant
Regato in turn, brought Victor Flores inside the sala. Thereafter,
Regato hit Victor Flores with the butt of his gun and said:
Where is your money? Where is your money? When Victor
answered that they do not have any, Rito Ramirez boxed Victor
at the mouth breaking one of his teeth.
While Victor was being maltreated by Rito and Regato force
him to reveal where their money was, Salceda was busy
ransacking a trunk inside the bedroom which contain P 870.00
in a box. Salceda took and went to the kitchen. He told Ramirez
that he had the money and Ramirez hit the man as he was angry
for Victor telling them earlier that he does not have money.
Ramirez shot Victor Flores following which Regato, Salceda
and Ramirez rushed out the house and fled.
After some minutes, Felicisima Flores went back and found her
husband bleeding. Things inside the bedroom and found the
money inside the trunk gone. With the help of a nephew, Victor
was brought to the poblacion of Palo. On the way, they met a
police patrol which proceeded to the scene of the robbery.
The party of Victor reached the municipal building of
Palo, Leyte about midnight of November 22 and few minutes
thereafter, he gave a written statement which is now marked as
Exhibit C.

The following morning, Victor was admitted at the


Leyte Provincial Hospital but due to severe hemorrhage,
secondary to gunshot wound, he died the same day.
Felicisima was formally investigated by the police to
whom she gave her affidavit now marked as Exhibit F. Jose
Salceda on November 26, 1969 was brought to the police
department as a suspect in the case. He was identified by
Felicisima Flores. Regato was likewise apprehended and a case
against the three. Miguel Regato, Jose Salceda and Rito Ramirez
were filed for Robbery with Homicide. The case was tried
against Regato and Salceda only because Rito Ramirez remains
at large.
The defense is in denial and alibi. Regato claimed that
on the night of November 22, 1969, he was in he Gacao, Palo,
Leyte attending a novena prayers for his late father-in-law were
his testimony corroborated by the defense witnesses. Salceda, on
the other hand testified that in the morning of November 22,
2969 he was in Bo. Gacao, Palo to transport palay. In that
evening he had a drinking spree with his group until 2 oclock in
the following morning.
HELD: WHEREFORE, the judgment appealed from is
AFFIRMED except as to the penalty, which is hereby modified
to reclusion perpetua.

31. PEOPLE VS. PAGAL


79 SCRA 570 (1977)

RATIONALE:

complex crime of robbery with homicide has been


committed.
On the other hand, appellants alibi must fall. In the
first place, Regatos submittal that he should have
been convicted of simple robbery only, instead of
robbery with homicide is an admission of his presence
at the scene of the crime contrary to his testimony that
he was in his house that evening of November 22,
1969 attending to novena prayers for his late father-inlaw. Secondly, to establish an alibi, it is not enough to
prove that appellants were at some other place when
the crime was committed but must, likewise
demonstrate that it was physically impossible for them
to have been at the place of commission at such time.
The distance between the place of the commission of
the offense to the place where appellants were
supposed to be at the time is only 1 kilometers, and
these places are connected with passable roads that
could have facilitated the easy negotiation by the
appellants between their respective homes and the
scene of the crime. Appellants evidence on this point
is not sufficient to overcome the positive identification
made by the prosecution witnesses Felicisima Flores
and Godofredo Flores.

We find no merit in the 2nd assigned error. Appellants


with Ramirez arrived together at the residence of
Victor Flores and pretended to buy cigarettes. When
Felicisima Flores opened the door, they went inside
the house and demanded Victor Flores to bring out the
money. When he refused, Ramirez and Regato
maltreated him while Salceda went inside the bedroom
and ransacked he trunk where the money was kept.
Ramirez then inquired whether he found the money
and he answered in the affirmative. It is the time that
the shooting of Victor Flores tool place after the
money had been taken and it was only when Flores
called them robbers that Ramirez shot him. As
partly stated by the lower court, It is clear that the
killing was done by reason or on the occasion of the
robbery, that the accused are guilty of the special
complex crime of robbery with homicide.
Likewise, we find no merit in the contention that there
was lack of intent to commit so grave a wrong as that
committed. Intention is a mental process and is an
internal state of mind, the intention must be judged by
his action, conduct and external acts of the accused.
What men do is the best index of their intention. IN
the case at bar, the aforesaid mitigating circumstance
cannot be appreciated considering that the acts
employed by the accused were reasonable sufficient to
produce the result they actually made death of the
victim.
With the respect to the fourth and fifth assigned errors,
nighttime and craft, the evidence is clear that the crime
was committed past 9:00 in the evening which the
culprits had especially sought he hiding mantle of the
night I order to facilitate its commission. Craft
involves intellectual trickery or cunning on the part of
the accused. Herein appellants, in order to enter the
house of Flores, shouted from the outside that they
wanted to buy cigarettes, which induced the inmates to
open the door for the, As held in People vs. Napili, 85
Phil 521, gaining entrance by pretending to buy
cigarettes or drink water constitutes craft.
By and large, the conspiracy among appellants and
Ramirez in the commission of the crime is evident
upon the facts as prove. Their acts, collectively and
individually executed, have demonstrated the
existence of a common design towards the
accomplishment of unlawful purpose and objective.
The shooting and death of Victor Flores bear a direct
relation and intimate connection between the robbery
and the killing which occurred during and on the
occasion of the robbery. Whether the latter be prior or
subsequent to the former, there is no doubt that he

15

Nature: This is an appeal on the Criminal Case No. CCC-VI-5


(70) of the Circuit Criminal Court of Manila, wherein the
accused, Pedro Pagal y Marcelino and Jose Torcelino y Torazo,
were charged with the crime of robbery with homicide.
Facts: That on or about December 26, 1969, in the City of
Manila, Philippines, the said accused, conspiring and
confederating together and mutually helping each other, did then
and there wilfully, unlawfully and feloniously, with intent to
gain, and by means of violence, take away from the person of
one Gau Guan, cash amounting to P1,281.00, Philippine
currency, to the damage and prejudice of the said Gau Guan in
the said sum of P1,281.00.
That on the occasion of the said robbery and for the purpose of
enabling them to take, steal and carry away the said amount of
P1,281.00, the herein accused, in pursuance of their conspiracy,
did then and there wilfully, unlawfully and feloniously, with
intent to kill and taking advantage of their superior strength,
treacherously attack, assault and use personal violence upon the
said Gau Guan, by then and there stabbing him with an icepick
and clubbing him with an iron pipe on different parts of his
body, thereby inflicting upon him mortal wounds which were
the direct and immediate cause of his death thereafter.
When the case was called for arraignment, counsel de oficio for
the accused informed said court of their intention to enter a plea
of guilty provided that they be allowed afterwards to prove the
mitigating circumstances of sufficient provocation or threat on
the part of the offended party immediately preceding the act, and
that of having acted upon an impulse so powerful as to produce
passion and obfuscation.
Held:
Since the alleged provocation which caused the
obfuscation of the appellants arose from the same incident, that
is, the alleged maltreatment and/or ill treatment of the appellants
by the deceased, these two mitigating circumstances cannot be
considered as two distinct and separate circumstances but should
be treated as one.
Secondly, the circumstance of passion and obfuscation cannot
be mitigating in a crime which as in the case at bar is
planned and calmly meditated before its execution.
Finally, the appellants claim that the trial court erred in
considering the aggravating circumstances of nighttime, evident
premeditation, and disregard of the respect due the offended
party on account of his rank and age. Although the trial court
correctly considered the aggravating circumstance of nocturnity
because the same was purposely and deliberately sought by the
appellants to facilitate the commission of the crime,
nevertheless, We disagree with its conclusion that evident
premeditation and disregard of the respect due the offended
party were present in the commission of the crime. Evident
premeditation is inherent in the crime of robbery. However, in
the crime of robbery with homicide, if there is evident

premeditation to kill besides stealing, it is considered as an


aggravating circumstance. In other words, evident premeditation
will only be aggravating in a complex crime of robbery with
homicide if it is proved that the plan is not only to rob, but also
to kill. In the case at bar, a perusal of the written statements of
the appellants before the police investigators show that then
original plan was only to rob, and that, they killed the deceased
only when the latter refused to open the "kaha de yero", and
fought with them. The trial court, therefore, erred in taking into
consideration the aggravating circumstance of evident
premeditation.
32. PEOPLE VS. PARANA
64 PHIL 331 (1937)
NATURE: The accused Primo Parana appeals from the
judgment of the Court of First Instance of Occidental Negros
sentencing him, for the crime of murder committed on the
person of Manuel Montinola, to the penalty of reclusion
perpetua and to infirmity the heirs of the deceased in the sum of
P1, 000.00 with costs.
FACTS: On the morning of May 19, 1936, in the municipality
of Silay, Occidental Negros, the deceased, who was taking part
in a game of monte in the house of Jose Lapuos, was informed
by the chauffeur Valentin Poblacion that his brother Glicerio
Montinolas car which he had ordered for his trip to the
municipality of Cadiz, was ready to start. Five minutes later the
deceased came downstairs and upon reaching the street, he
turned towards the car which was waiting for him.
At that moment the chauffeur Poblacion, who saw the
appellant behind the deceased in the attitude of stabbing him
with a dagger shouted to warn him of the danger, and the
deceaseds looking behind, really saw the appellant about to stab
him. The deceased, defending himself retreated until he fell on
his back into a ditch two meters wide and 1.7 meters deep.
Without lessening the aggression on the appellant mounted
astride of the deceased and continued to stab him with the
dagger.
After the appellant and the deceased had been
separated, the former still asked Montelibano for the weapon
taken from him but at that moment a policeman arrived and the
appellant was placed under arrest. When the deceased was later
removed from the ditch into which he had fallen, he was found
wounded and was taken to the provincial hospital where he was
treated by Dr. Ochoa, expiring six days later, as a result of
general peritonitis produces by one of his wounds.
The preceding nights, at about 11oclosk, monte had
also been played in the house of Glicerio Montinola, brother of
the deceased. The deceased took part in said game where the
appellant was designated to attend to the players. One Lamay,
who was also taking part in the game, gave appellant the sum of
P2 to buy beer. For failure of the appellant to immediately
comply with this request, a discussion ensued between him and
Lamay and, as both raised their voices, they were admonished
by the deceased. As the appellant disregarded said admonition,
the deceased slapped him and ordered him to leave the house.
The appellant left and went to Lapuos house where he lived,
where the deceased took part in another game on the following
day, and where said deceased came from when he was attacked.
At about 7 oclock in the morning of the crime, the
appellant purchased from the store of the Japanese Matzu
Akisama, a hunting knife (Exhibit F) which is the same knife
used by him in attacking the deceased.
On the same morning, at about seven thirty, the
appellant went to the house of Crispin Espacio for who he used
to work to ask to wreak vengeance on somebody. Espacio
advised him against it as he might again go to Bilibid prison,
inasmuch as he had already served a term for the crime
homicide.
The appellants testimony is the only evidence in his
defense. According to him, on the morning of the crime he saw
the deceased taking part in the game in Lapuos house where he
lived. The deceased then uttered threatening words to him which
he disregarded, leaving the house and going to a nearby Chinese
store. Sometime, later as he was on his way for Lapuos house,
he saw the deceased coming down and approaching the latter, he
spoke to him about the incident of the previous night and of their
meeting a few minutes before asking said deceased to forgive

16

and met wreak vengeance on him. The deceased, by way of an


answer drew the revolver which he carried on his belt, and the
appellant in the efface suck attitude, attempted to wrest the
weapon from him. In the struggle the deceased fell on his back
into a ditch and the appellant mounted astride of him tried to
wrest the revolver from him, and at the same time drew the knife
which he carried, attacking the deceased therewith. When the
appellant had succeeded in taking possession of the revolver, the
deceased got up and walked towards the car. At that moment
Liboro Montelibano appeared and the appellant turned over the
knife and the revolver to him.
The version of the incident given by the appellant in
his testimony, without any corroboration is contradicted by the
testimony of the chauffeur Poblacion and of Liboro
Montelibano. Furthermore, it is improbable taking into
consideration the fact that he was the offended [arty, suffering
from the injustice of the offense received, provided himself with
lethal weapon and approached the deceased, which
circumstances do not agree with his attitude according to his
testimony.
HELD: The court finds the appellant guilty of the crime of
murder qualified by treachery and taking into consideration the
presence of one aggravating and two mitigating circumstances in
the commission of the crime and applying the Indeterminate
Sentence Law, Act No. 4103, he is sentenced to the penalty of
from ten years of prision mayor, as the minimum, to seventeen
years, four months and one day of reclusion temporal, as the
maximum, affirming the appealed sentence in all other respects
with the costs.
RATIONALE:

The court correctly found that the qualifying


circumstance of treachery was present in the
commission of the crime. The appellant, inspite of
having seen the deceased in the upper story of Lapuos'
house, did nor wish to attack the latter there
undoubtedly to avid his being defended by the many
players who were with him. Instead, he waited for the
deceased at the nearby store until the latter came
down, and attack him while he had his back turned
and could not see the appellant. All these, which were
the beginning of the execution of the appellants
design to kill ye deceased, constitute treachery
inasmuch as they tended to avoid every risk to himself
arising from the defense which the deceased might
make.

The aggravating circumstance that the appellant is a


recidivist must be taken into consideration. The
mitigating circumstance that he had acted in the
immediate vindication of a grave offense committed
against him a few hours before, when he was slapped
by the deceased in the presence of many persons, must
likewise be taken into consideration. Although this
offense, which engenders perturbation of mind, was
not so immediate, that the influence thereof, by reason
o it gravity and the circumstances under which it was
inflicted, lasted until the moment the crime was
continued. Lastly, the other mitigating circumstance
that the appellant had voluntarily surrendered himself
to the agents of the authorities must be considered.
33. PEOPLE VS. DIOKNO
63 PHIL 601 (1936)
Nature: Appeal from a judgment of the CFI (Court of First
Instance now Regional Trial Court) of Laguna.
Facts: The deceased Yu Hiong was a vendor of sundry goods in
Lucena, Tayabas. At 7 oclock in the morning of January $,
1935, Salome Diokno, to whom Yu Hiong was engaged for
about a year, invited the latter to go with her. Yu Hiong accepted
the invitation but he told Salome that her father was angry with
him. At about 6 oclock in the afternoon of said day, Yu Hiong
and Salome Diokno took an automobile and went to the house of
Vicente Verina, Salomes cousin in Pagbilao.
On January 5th or 6th of said year, Roman Diokno telegraphed his
father Epifanio Diokno, who was in Manila, informing him that
Salome had eloped with the Chinese Yu Hiong, on the morning

of January 7, 1935, Epifanio and Roman Diokno went to San


Pablo, in search of the elopers. Upon arriving near the house,
they saw Yu Hiong coming down the stairs. When Yu Hiong
saw them, he ran upstairs and they pursued him. At the moment,
he was overtaken by the accused that carried knives locally
known as balisong, of different size. Yu Hiong fell on his knees
and implored pardon. Roman Diokno stabbed him with the knife
I the back and the later in the left side. Epifanio stabbed at once.
Issue: Whether or not said accused were physically stronger
than the deceased? And whether or not the said accused abused
such superiority?
Held: Neither does this court find the existence of the other
circumstance qualifying murder, that is, evident premeditation
proven beyond a reasonable doubt. In order that premeditation
may be considered either as an aggravating or as an qualifying
circumstance, it must be evident, that5 is the intention to kill
must be manifest and it must have been planned in the mind of
the offender and carefully meditated therefore, there having
been neither abuse of superior strength nor evident
premeditation, the crime committed by the accused is simple
homicide.
The presence of 5th mitigating circumstance of Article 13 of the
Revised Penal Code maybe taken into consideration in favor of
the two accused, because the offense did not cease while
Salomes whereabouts remained unknown and her marriage to
the deceased unlegalized. Therefore, there was no interruption
from the time of offense was committed to the vindication
thereof.
The presence of the 6th mitigating circumstance of Article 13 of
the RPC may also be taken into consideration in favor of the
accused. The fact that the accused saw the deceased ran upstairs
when he became aware of their presence, as if he refused to deal
with them after having gravely offended him.
The 7h mitigating circumstance of Article 13 of the RPC should
also be taken in favor of the accused Epifanio Diokno.
The court conclude that accused are guilty of reasonable beyond
reasonable doubt of the crime of homicide, the penalty therein
being reclusion temporal in its full extent, 3 mitigating
circumstances must be taken into consideration in favor of the
accused Epifanio and 2 mitigating circumstances in favor of the
accused Roman Diokno with no aggravating circumstances.
Both accused should be granted the benefits of the indeterminate
sentence which prescribe a penalty the minimum of which shall
be taken from the next lower to prision mayor, or prision
correccional of from 6 months and 1 day to 6 years. Taking into
account the circumstances of the case, the indeterminate penalty
to which each of said accused must be sentenced is fixed at from
2 years and 1 day of prision correccional. Wherefore, the
accused was guilty of the crime of homicide and sentence each
of them to an indeterminate penalty from 2 years, 1 day of
prison correccional to 8 years, 1 day of prision mayor.
34. PEOPLE VS. MUIT
117 SCRA 696 (1982)
NATURE: Appeal from the Decision of the Court of First
Instance of Camarines Sur, Branch I (Naga City), in Criminal
Case No. R-7 (1847), convicting the accused Delfin Muit, a
retired PC 2nd Lieutenant, of Murder, and sentencing him to
suffer the penalty of reclusion perpetual for the gunning down
of the victim, Rodolfo Torrero.
FACTS: On February 26, 1976 at about 2:45 o'clock in the
afternoon, while the deceased Rodolfo Torrero, his family and
friends, were passing by the house of accused Delfin Muit at
barrio Tamban, Tinambac, Camarines Sur, on their Nay home
from a picnic, the accused invited them to his house to take a
rest. At that time Muit was alone as his wife and children were
not around. Once the group was inside his house, accused Muit
who is a retired PC lieutenant remarked that his invitation
showed that he had no ill-feeling against the Torreros and that
he knew the latter had no ill-feeling also against him. As they
engaged in some amenities, a group of barangay members and
PC authorities in !barge of the sanitation and cleanliness
program on that particular lay, which was Community Day, paid
them a visit, and after a short talk, said group left. The deceased

17

Torrero conducted the group on their way out and upon his
return; accused Muit requested him to take a seat.
The accused then confronted Rodolfo Torrero why the latter
always visits his wife even during nighttime and why he often
invites her out. Torrero replied that being the barangay zone
auditor, he had to confer with the accused's wife on barangay
matters as the latter was the barangay zone president. The
accused then asked why Torrero even gave food and money to
his (accused's) children if he had no bad intention at all on his
wife. Torrero's wife answered that they did it out of pity because
there were times when they would see the accused's children in
need of food and money. The accused, however, angrily stood
up and countered, "Why should you give when your husband
had also a family to support? To avoid any trouble, the
deceased Torrero likewise stood up and said, 'If that is the way
we talked about this will end to nothing, so it is better that I
should leave', and he proceeded to move out of the house.
When Torrero was already outside the house of the accused and
while walking along the pathway, the accused followed him and
on reaching the door the accused shouted, "Wait because we
have not yet finished". At that instant, the accused raised his left
hand towards Torrero and with his right hand; he pulled out his .
45 caliber pistol and aimed it at the deceased. Angrily, he fired
his gun at Torrero who was just 3 meters away, hitting the
latter at the lower left side below the nape. On being hit by the
bullet, Torrero spun from his left to the right, with his two hands
inclined to the right, his face writhing in pain, his left elbow
raised parallel to his armpit and his right hand placed on his
breast.
Upon hearing the gunshot, witness Gubatan immediately
grabbed and held the accused from behind with an embrace, and
said, "Manoy Delfin, why are you like that? But as soon as
Gubatan embraced the accused from behind, a second shot was
fired, this time hitting the elevated left hand of Torrero, with the
bullet penetrating through the breast. Consequently, Torrero fell
on his knees, bent forward with face downward and body in a
prone position his left elbow supporting him on his left lap while
his right hand extended to the ground. Witness Gubatan on the
other hand tightened his grip around the accused as he tried to
wrestle with him.
The wife of Torrero, who was shocked by the first shot
thereupon rushed towards her fallen husband. But the accused
on seeing Mrs. Torrero rushed towards the deceased, aimed his
gun at her. Fortunately, witness Gubatan quickly grabbed the
right forearm of the accused that held the gun and jerked it
upward so that the third shot was fired towards the sky, thus
missing its target Id). Witness Gubatan then said, "Manoy Delfin
that is enough". Gubatan thereafter moved the accused away and
brought him near a coconut tree. Mrs. Torrero, on the other
hand, hugged her husband and cried for help, even as blood was
oozing out from the deceased's body and mouth. Shortly
thereafter, Torrero died.
Near the coconut tree, the accused tried to free himself from the
hold of Gubatan. He even pointed his gun at Gubatan and said,
"Set me free Benny or I will shoot you. When Gubatan could
no longer hold the accused as the latter kept on struggling, he let
him go and said, "Alright Kuya Delfin, shoot me, after all I have
no fault". Slowly, the accused put down his arm, his eyes at
static condition. He (appellant) then started to move away, and
as he did, he made a short last look at his victim, after which, he
continued on his way. After the incident, the accused proceeded
to the PC detachment to surrender himself and his gun.
ISSUE: Whether or not the accused can involve passion and
obfuscation as mitigating circumstance?
HELD: Modifying the judgment appealed from, the accused
Delfin Muit, is hereby sentenced to suffer the indeterminate
penalty of eight (8) years of prision mayor, as minimum, to
fourteen (14) years and eight (8) months of reclusion temporal,
as maximum, with the judgment being affirmed in an other
respects.
RATIONALE:
During the trial the accused involved passion and
obfuscation as mitigating circumstance. There can be no
question that the accused was driven strongly buy jealousy
because of rumors regarding the amorous relationship between
his wife and the victim. The feeling of resentment resulting from

rivalry in amorous relation with a woman is a powerful


stimulant from rivalry and amorous relation with a woman is a
powerful stimulant to jealously and is sufficient to produce loss
of reason and self-control. In other words, it is a powerful
instigation of jealously and prone to produce anger and
obfuscation.
35. PEOPLE VS. AQUINO
G.R. O. 128887 (JAN. 2000)
NATURE: Appeal from a decision of the Regional Trial Court
of Olongapo City, Branch 75
FACTS: On January 19, 1996, Roselyn Lampera, daughter of
Valerio and Esmeralda Lampera was in their house, together
with her mother, younger brother Daniel and younger sister.
Their house is like a small cubicle without any partitions,
elevated from the ground by about 2 1/2 feet.
In the morning of that fateful day, Roselyn's mother, Esmeralda,
was in their house taking care of Roselyn's younger sister who
was sick at the time. Her younger brother, on the other hand,
was playing on the ground near their house. Appellant Edgardo
Aquino (who was their neighbor) arrived, looking for their
father. Both Roselyn and her mother informed Edgardo that
Valerio, Roselyn's father, was in Olongapo.
Unsatisfied with their answer, Edgardo (who was near the door
at the time) peeped in their house and when he did not see
Valerio, pulled out his knife. Initially, he tried to stab Roselyn's
younger brother. When Roselyn and her mother saw this, they
rushed towards the younger boy in an attempt to protect him.
When Edgardo saw their reaction, Edgardo stepped inside their
house, eager to vent his ire on Roselyn, intending to stab her.
Roselyn's mother pulled her aside, shouting. Edgardo went for
her mother who tried valiantly to evade his thrust as she was
then carrying Roselyn's sick younger sister. Roselyn saw
Edgardo repeatedly stab her mother in the latter's stomach and
chest areas. Out of fear, Roselyn managed to destroy their nipa
wall and jumped out of their house. Despite her shouts for help,
no help came.
At about the same time also, Benjamin Costimiano, a purok
leader, was in his house when he heard some kind of shouting or
commotion. Being a purok leader, he went to the place of
incident and saw the victim. He heard the people there say that
the culprit was Edgardo Aquino. He went after Edgardo and was
able to catch up with him in the house of one Francisco Franco.
Benjamin asked Edgardo (who was still armed with a knife at
that time) to put down the knife and the latter gave him the
knife. Benjamin described the knife used as a double-bladed
one, and when it was handed to him, the handle still had some
blood on it.
Dr. Nancy Valdez, Medico-legal Officer III of the San
Marcelino District Hospital, testified that she was the one who
conducted the autopsy on the cadaver of the victim. She noted
four (4) stab wounds at the xiphoid processes/chest area, two (2)
of which were fatal as they penetrated the thoracic cavity,
causing lacerations on the anterior portion of the superior lobe of
the left lung.
Valerio Lampera, Esmeralda's husband, declared that the
untimely death of Esmeralda caused him pain and compelled the
family to incur expenses in the amount of P2, 500. Daniel Isaac,
Esmeralda's 8-year-old son, was likewise psychologically and
emotionally affected by the unexpected demise of his mother.
He cried on the witness stand when asked of the whereabouts of
his mother.
EDGARDO had another story to tell. According to him,
Esmeralda's husband was his business partner in the sale of fish.
In the evening of 19 January 1996, he went to the house of the
Lamperas to get his capital for the business. He saw Roselyn
standing by the stairs of the house and asked her about the
whereabouts of her father Valerio. When she informed him that
Valerio was not there, he left for the store of Francisco Franco.
On his way to the store, he heard shouts coming from the
Lampera's house, which he mistook to be just another ordinary
fight. He proceeded to Franco's store. Then Benjamin
Costimiano, a purok leader, arrived at the store, carrying with
him a knife which, according to him, was recovered from inside
Esmeralda's house. Benjamin invited EDGARDO to go with
him to the Police Department of Subic, Zambales. Upon arrival
thereat Costimiano ordered the detention of EDGARDO

18

allegedly because the latter was a suspect in the killing of


Esmeralda. EDGARDO was detained for two months but was
not investigated by the police. He could not remember having
been brought to the office of the Provincial Prosecutor and
having given a statement thereat. He insisted that he did not kill
Esmeralda and that the knife presented by the prosecution was
not taken from him. Besides, he had no reason to kill the wife of
his business partner
ISSUE: Whether or not that treachery can be appreciated as
aggravating circumstance; and temporary insanity, passion and
obfuscation, intoxication, and voluntary surrender as mitigating
circumstances in the crime committed by the accused
HELD: The challenged decision of Branch 75 of the Regional
Trial Court of Olongapo City in Criminal Case No. 56-96 is
MODIFIED. As modified, accused-appellant EDGARDO
AQUINO y PUMAWAN is found guilty beyond reasonable
doubt, as principal, of the crime of homicide, defined and
penalized under Article 249 of the Revised Penal Code, and is
hereby sentenced to suffer an indeterminate penalty ranging
from eight (8) years and one (1) day of prision mayor as
minimum to seventeen (17) years and four (4) months of
reclusion temporal as maximum. The awards of P50, 000 as
indemnity, P50, 000 as moral damages, P30, 000 as exemplary
damages, and P2, 500 as actual damages stand
RATIONALE: There was treachery in view of the sudden and
unexpected attack upon the unarmed victim, who had not
committed the slightest provocation and who was totally
unaware of EDGARDO's murderous designs. Neither the victim
nor her children anticipated the attack. EDGARDO did not give
any warning that he was about to start a stabbing spree. The
victim, then carrying a sick child, never had the chance to
defend her or to retaliate. All that she managed to do was to try
to evade EDGARDO's knife blows. In this case, the victim,
Esmeralda, was forewarned of the impending attack on her,
since it was preceded by EDGARDO's attempts to attack her son
and daughter. It cannot be said that she was in no position to
defend her; for, in fact, she succeeded in repelling appellant's
aggression against her children. When EDGARDO turned to
her, she "tried to evade the thrust" causing her 6-year-old child
whom she was carrying to be thrown away. Furthermore, there
is no sufficient evidence that the appellant deliberately and
consciously adopted the means of execution employed by him.
What is apparent is that the killing was done impulsively or on
the spur of the moment.
"Temporary insanity" is not recognized in this
jurisdiction and that mere abnormality of the mental faculties
will not exclude imputability. In any case, EDGARDO had the
burden of proving his alleged "temporary insanity," as it is a
basic principle in our rules on evidence that he who alleges a
fact must prove the truth thereof. However, he did not raise this
argument below, and it is only now that he belatedly raises it.
Anent EDCARDO's claim of the mitigating
circumstance of passion or obfuscation, the same is bereft of
merit because his acts did not result from an impulse arising
from lawful sentiments but from a spirit of lawlessness.
We disagree with the trial court in appreciating in
appellant's favor the mitigating circumstance of intoxication,
EDGARDO declared that he drank liquor on the day of the
incident in question, and the trial court held that his intoxication
was corroborated by Roselyn's testimony that EDGARDO's eyes
were "red" when she saw him. For intoxication to be mitigating,
the following conditions must be present: (1) the same is not
habitual or is not subsequent to the plan of the commission of a
felony; otherwise, it is aggravating if it is habitual and
intentional; and (2) the consumption of alcoholic drinks was in
such quantity as to blur the accused's reason and deprive him of
a certain degree of control. In this case, EDGARDO was unable
to prove both requisites.
Nevertheless, we appreciate in EDGARDO's favor the
mitigating circumstance of voluntary surrender. Immediately
after the incident, when purok leader Benjamin Costimiano
followed him in the house of Francisco Franco, EDGARDO
voluntarily gave the knife to Franco and went with the latter to
the Police Headquarters where he was forthwith detained. The
information against him was filed much later.

36. PEOPLE VS. CAMAGUIN


229 SCRA 166 (1994)
37. PEOPLE VS. DULOS
237 SCRA 141 (1994)
Nature: It is an appeal from a decision of the Regional Trial
Court of Cotabato City Br. 13 which found accused Efren Dulos
guilty of the crime of Murder committed with treachery and
sentencing him reclusion perpetua.
Facts: On the evening of march 15, 1987, professional
entertainers, Susan and Alice, were sitting at the lobby of the
New Imperial Hotel in Cotabato city, waiting for prospective
clients. A military police assigned at the said hotel as watchman
approached them and told them that the accused-appellant had
some male guests who wished to be entertained. Both parties
agreed to a charge of Php 100.00 each for Susan and Alice as fee
for their services. The parties then went to a disco house
however Alice left early and decided to wait for Susan at the
lobby of the Hotel. Susan , apparently decided to check in with
one of the accused-appellants guests at the upper floor of the
disco house for an additional fee of Php 500.00. After receiving
the money, she changed her mind, thus, spawning a fight with
her customer. After MP Gara intervene, Susan came down to the
lobby of the Hotel to meet her boyfriend Paul Tamse who was
waiting for her. Upon hearing the reneged deal, the irate
accused-appellant confronted Susan and Paul. Accusedappellant demanded the return of his money. Susan handed the
Php 100.00 to accused-appellant however she denied having
received Php 500.00 unless her boyfriend would find out that
she agreed to check in with a customer. She tried to grapple with
the accused-appellant for the gun but was violently pushed
aside. The boyfriend on the other hand pleaded for mercy by
kneeling down and raising his hands up. Despite the plea, he
was shot by the accused-appellant twice killing him on the spot.
The accused-appellant surrendered the gun to the military
authorities in Camp Siongco, Maguindanao and was not placed
under custody by the military authorities as he was free to roam
around as he pleased.
Issue: Won a mitigating circumstance of voluntary surrender
could be appreciated in this case.
Ruling: No, a mitigating circumstance of voluntary surrender
could not be appreciated in this case. The SC held that in order
that voluntary surrender may be appreciated, it is necessary that
it must be spontaneous and made in such manner that it shows
the intent of the accused to surrender unconditionally to the
authorities, either because he acknowledged his guilt or because
he wishes to save the trouble and expenses necessarily incurred
in his search and capture. In this case the elements were not
present. There was no conscious effort on the part of Dulos to
voluntarily surrender to the military authorities. As he himself
admitted in his testimony, he was not placed under custody by
the military as he was free to roam around as he pleased.
Likewise, his claim that he surrendered his gun without
surrendering his person to the authorities does not constitute
voluntary surrender
38. PEOPLE VS. CRISOSTOMO
160 SCRA 47 (1998)
Nature: It is an appeal from the decision of the Court of First
Instance of Bulacan which found Eugenio Crisostomo guilty of
the crime of murder and sentences him to Reclusion Perpetua,
and to indemnify the heirs of the deceased in the sum fo Php
12,000.00 and to pay the costs.
Facts: On December 25, 1967 between six and seven oclock in
the evening at Sto. Rosario, Hagonoy, Bulacan, while Eugenio
Crisostomo was passing near the house of Romeo Geronimo, he
met the latter and invited him to have a drink in the place of a
friend. Romeo declined the offer. Suddenly Eugenio rushed
towards Romeo who was then standing near a store facing the
street with his back towards Eugenio and shot him with a .22
caliber revolver at a distance of one meter. The bullet entered
into his armpit and came out on the right side of the chest about
one inch in the sternum. Romeo fell to the ground mortally

19

wounded while Eugenio ran away. By-standers who were near


the place came to the aide of the fallen victim and brought him
to the Reyes Hospital in Hagonoy where the doctor pronounced
the victim dead upon arrival. Upon arraignment wherein accused
entered a plea of not guilty and again during the trial, the
accused signified his intention to withdraw hi plea of not guilty
to the charge of murder and to substitute it with a plea of guilty
to a lesser charge of homicide and prayed that he be allowed to
prove the mitigating circumstances. The same plea was made by
the accused after the prosecution had rested its case but the
fiscal did not agree. Thus the lower court denied the petition.
Issue: Won a mitigating circumstance of voluntary plea of guilt
be appreciated in this case.
Ruling: No, the mitigating circumstance of voluntary plea of
guilt is not appreciated in this case. The SC held that he cannot
be credited with the mitigating circumstance of a plea of guilty
to a lesser offense of the charge of homicide. The requisites of
the mitigating circumstance of voluntary plea of guilty are: (1)
that the offender spontaneously confessed his guilt; (2) that the
confession of guilt was made in open court, that is, before the
competent court that is to try the case; and (3) that the
confession of guilt was made prior to the presentation of
evidence for the prosecution. In the present case the appellant
offered to enter a plea of guilty to the lesser offense of homicide
only after some evidence of the prosecution had been presented.
He reiterated his offer after the prosecution rested his case. This
is certainly not mitigating.
39. PEOPLE VS. JOSE ET AL.
37 SCRA 450 (1971)
Nature: Appeal from and automatic review of a decision of the
Court of first Instance of Rizal.
Facts: At about 4:30 am, June 26, 1967, Miss De la Riva, was
driving her car accompanied by her maid Helen Calderon. As
she was approaching her house at No. 48, 12th Street, New
Manila, Quezon City, a Pontiac two-door convertible car with
the four accused came abreast of her car and tried to bump it.
Pineda stopped the car which he was driving, jumped out of it
and rushed towards her.
The girl became so frightened at this turn of events that she
tooted the horn of her car continuously. Undaunted, Pineda
opened the door of Miss De la Riva's car and grabbed the the
victims arm and dragged her inside the car. The complainant
was made to sit between Jaime Jose and Edgardo Aquino at the
back seat; Basilio Pineda, Jr. was at the wheel, while Rogelio
Caal was seated beside him. The two men seated on each side
of Miss De la Riva started to get busy with her body: Jose put
one arm around the complainant and forced his lips upon hers,
while Aquino placed his arms on her thighs and lifted her skirt.
The car reached a dead-end street. Pineda turned the car around
and headed towards Victoria Street. Then the car proceeded to
Araneta Avenue, Sta. Mesa Street, Shaw Boulevard, thence to
Epifanio de los Santos Avenue. When the car reached Makati,
Aquino took a handkerchief from his pocket and, with the help
of Jose, blindfolded Miss De la Riva. Not long after, the car
came to a stop at the Swanky Hotel in Pasay City. The
blindfolded lady was led out of the car to one of the rooms on
the second floor of the hotel.
Inside the room Miss De la Riva was made to sit on bed. Her
blindfold was removed. She saw Pineda and Aquino standing in
front of her, and Jose and Caal sitting beside her, all of them
smiling meaningfully. Pineda ordered the victim to striptease to
which the other men agreed to. After which, the four men took
turns in raping her starting with Jose, Aquino, Pineda and Canal.
Whenever dela Riva passed out, they would pour water on her
face and slapp her to revive her. Mention must be made of the
fact that while each of the four appellants was struggling with
the complainant, the other three were outside the room, just
behind the door, threatening the complainant with acid and
telling her to give in because she could not, after all, escape,
what with their presence.
After the appellants had been through with the sexual carnage,
they dropped her in front of the Free Press Building not far
from Epifanio de los Santos Avenue near Channel 5 to make it

appear, according to them, that the complainant had just come


from the studio.
Issue: W/N a plea of guilty is mitigating, at the same time it
constitutes an admission of all the material facts alleged in the
information.
Held: the judgment under review is hereby modified as follows:
appellants Jaime G. Jose, Basilio Pineda, Jr., and Edgardo P.
Aquino are pronounced guilty of the complex crime of forcible
abduction with rape, and each and every one of them is likewise
convicted of three (3) other crimes of rape. As a consequence
thereof, each of them is hereby sentenced to four (4) death
penalties.
Ratio: Appellant Pineda claims that insofar as he is concerned
there was a mistrial resulting in gross miscarriage of justice. He
contends that because the charge against him and his coappellants is a capital offense and the amended complaint cited
aggravating circumstances, which, if proved, would raise the
penalty to death, it was the duty of the court to insist on his
presence during all stages of the trial. The contention is
untenable.
Because of the aforesaid legal effect of Pineda's plea of guilty, it
was not incumbent upon the trial court to receive his evidence,
much less to require his presence in court. It would be different
had appellant Pineda requested the court to allure him to prove
mitigating circumstances, for then it would be the better part of
discretion on the part of the trial court to grant his request.
The technicalities in plain simple language of the contents of
aggravating circumstances and apprised him of the penalty he
would get, and we have given said accused time to think. After a
while I consulted him - for three times - and his decision was
still the same.
Three days after the arraignment, the same counsel stated in
court that he had always been averse to Pineda's idea of pleading
guilty, but that he acceded to his client's wish only after the
fiscal had stated that he would recommend to the court the
imposition of life imprisonment on his client.
40. PEOPLE VS. VILLA
G.R. 129899 (APRIL 2000)
Nature: Appeal from a decision of the Regional Trial Court of
Olongapo City
Facts: In the early morning of 22 June 1991 Dionito Fernandez
was cutting grass in his yard in New Cabalan, Olongapo City.
Accused Rodolfo Villa, Jr., a member of the CAFGU and
neighbor of Dionito, suddenly came out of his house with his M1 Garand rifle and shot Dionito from behind killing him
instantly. Ronald Fernandez and Sheila Fernandez, children of
Dionito, rushed to their father's rescue after hearing the gunshot
but the accused also fired at them fatally hitting Ronald who was
embracing his father, and mortally wounding Sheila on the thigh
and stomach. Samuel Eclevia, another neighbor of the
Fernandezes, attempted to wrestle the rifle from the accused but
Samuel too was gunned down.
After his rampage, Rodolfo Villa Jr. surrendered to a certain
Captain Dolino of S2 OMDC (Olongapo Metropolitan District
Command
Issue: W/N taking advantage of his public position as a
CAFGU member should be considered against accusedappellant.
Held: The assailed Decision of the trial court convicting
accused-appellant RODOLFO VILLA, JR. Y DELGADO of
four (4) separate counts of Murder is AFFIRMED, subject to the
MODIFICATION of the penalties imposed.
The SC did not agree that the aggravating circumstance of
"taking advantage of his public position" as a CAFGU member
should be considered against accused-appellant. The mere fact
that he was a member of the CAFGU and was issued an M-1
Garand rifle is not sufficient to establish that he misused his
public position in the commission of the crimes.

Nature: This is an appeal from the decision of the Regional


Trial Court, Branch XVI, Isabela in Criminal Case No. IV-781,
finding appellant guilty beyond reasonable doubt of murder
qualified by treachery, with the attendance of the mitigating
circumstance of voluntary surrender, and the aggravating
circumstances of taking advantage of public position and evident
premeditation sentencing him to suffer reclusion perpetua.

FACTS: According to prosecution witness Alberto Carrido, he


and Rodrigo Ballad left the house of Enteng Teppang at about
2:00 P.M. of October 6, 1979 after attending the "pamisa" for
the deceased father of Teppang. Jerry Calpito followed them.
While they were walking along the barangay road, Calpito was
shot by appellant with an armalite rifle. When Calpito fell on the
ground, appellant fired more shots at him. Thereafter, accused
Amor Saludares planted a .22 caliber revolver on the left hand of
Calpito. Upon hearing the shots, Faustina Calpito ran to succor
her fallen husband.
Accused Nicanor Saludares pointed his gun at Faustina while
accused Soriano fired his gun upwards. Saludares warned that he
would kill any relative of Jerry Calpito who would come near
him. Faustina and the other relatives of the victim scampered
away as the Saludares' group chased them.
Appellant invoked self-defense. He testified that he was issued a
mission order on September 23, 1979 to investigate a report
regarding the presence of unidentified armed men in Barrio San
Jose, Roxas, Isabela. The following day, he was instructed by
Sgt. Dominador Ignacio to get in touch with Nicanor Saludares
who may be able to give him information on the identities of the
persons with unlicensed firearms in the place. When appellant
met Nicanor Saludares on September 29, 1979, he was informed
that Jerry Calpito had an unlicensed firearm.
The body of Calpito was autopsied by Dr. Bernardo Layugan,
who found that the victim sustained four bullet wounds: (1) on
the right lateral side of the arm fracturing the humerus; (2) on
the right lateral side of the thorax between the 7th and 8th ribs
with exit wound at the sternum; (3) on the left side of the thorax,
anterior, between the 5th and 6th ribs; and (4) on the right
fronto-parietal portion of the head "severing the skull and brain
tissues" (Exh. "F"). Dr. Layugan opined that the victim was in a
standing position when he was shot by someone positioned at
his right.

ISSUE: Whether or not the appellants are guilty beyond


reasonable doubt of murder qualified by treachery, with the
attendance of the mitigating circumstance of voluntary
surrender, and the aggravating circumstances of taking
advantage of public position and evident premeditation?
HELD: Appellants are guilty beyond reasonable doubt of
murder qualified by treachery, with the attendance of the
mitigating circumstance of voluntary surrender, and the
aggravating circumstances of taking advantage of public
position and evident premeditation.
Appellant's claim of self-defense is belied by the finding of the
trial court that the victim was shot by someone who was
standing on his right side.
The fact that the prosecution witnesses are relatives of the victim
does not necessarily indicate that they were biased as to impair
their credibility.
42. PEOPLE VS. RODIL
109 SCRA 308 (1981)
Nature: Automatic Review of the judgement of the Circuit
Criminal Court of Pasig Rizal.
Facts: At about 1:00 o'clock in the afternoon of April 24, 1971,
the deceased, PC Lt. Guillermo Masana, together with PC
soldier Virgilio Fidel, Philippine Coast Guard serviceman
Ricardo Ligsa, and Patrolman Felix Mojica of Indang, Cavite,

41. PEOPLE VS. GAPASIN


231 SCRA 728 (1994)

20

was having lunch inside a restaurant in front of the Indang


market (pp. 2, 3, t.s.n., Oct. 30, 1971; pp. 10, 19, t.s.n., Nov. 22,
1971; p. 21, t.s.n., Jan. 20, 1972). While they were eating, they
saw, through the glass panel of the restaurant, appellant outside
the restaurant blowing his whistle. Their attention having been
drawn to what appellant was doing, Lt. Masana, then in civilian
clothing, accompanied by PC soldier Virgilio Fidel, went out of
the restaurant, approached appellant and asked the latter, after
identifying himself as a PC officer, whether the gun that was
tucked in his waist had a license. Instead of answering the
question of Lt. Masana, appellant moved one step backward and
attempted to draw his gun. PC soldier Virgilio Fidel
immediately grabbed appellant's gun from appellant's waist and
gave it to Lt. Masana. After that, Lt. Masana told the appellant
to go inside the restaurant. PC soldier Virgilio Fidel followed.
Lt. Masana and the appellant occupied a separate table about
one and one-half (1 1/2) meters from the table of Lt. Masana's
three companionsFidel, Ligsa and Mojica (p. 10, t.s.n., Nov. 22,
1971). After the two were already seated, Lt. Masana placed
appellant's gun on the table. After that Lt. Masana pulled out a
piece of coupon bond paper from his pocket and wrote thereon
the receipt for the gun, and after signing it, he asked appellant to
countersign the same, but appellant refused to do so. Instead, he
asked Lt. Masana to return the gun to him. Lt. Masana rejected
appellant's plea, telling the latter that they would talk the matter
over in the municipal building of Indang, Cavite. When Lt.
Masana was about to stand up, appellant suddenly pulled out a
double-bladed dagger and with it he stabbed Lt. Masana several
times, on the chest and stomach causing his death several hours
thereafter.
Issue: W/N the attack on the victim, who was known to the
appellant as a peace officer, could be considered only as
aggravating, being "in contempt of/or with insult to the public
authorities", or as an "insult or in disregard of the respect due
the offended party on account of his rank.
Held:
GUILTY BEYOND REASONABLE DOUBT OF
HOMICIDE AGGRAVATED BY CONTEMPT FOR OR
INSULT TO A PUBLIC AUTHORITY, THE JUDGMENT
APPEALED FROM IS AFFIRMED, PENALTY MODIFIED.
While the evidence definitely demonstrated that appellant knew
because the victim, who was in civilian clothing, told him that
he was an agent of a person in authority, he cannot be convicted
of the complex crime of homicide with assault upon an agent of
a person in authority, for the simple reason that the information
does not allege the fact that the accused then knew that, before
or at the time of the assault, the victim was an agent of a person
in authority. The information simply alleges that appellant did,
"attack and stab PC Lt. Guillermo Masana while the latter was
in the performance of his official duties, . . . " Such an allegation
.cannot be an adequate substitute for the essential averment to
justify a conviction of the complex crime, which necessarily
requires the imposition of the maximum period of the penalty
prescribed for the graver offense. Like a qualifying
circumstance, such knowledge must be expressly and
specifically averred in the information; otherwise, in the absence
of such allegation, the required knowledge, like a qualifying
circumstance, although proven, would only be appreciated as a
generic aggravating circumstance.
43. PEOPLE VS. DANIEL
86 SCRA 511 (1978)
Nature: APPEAL from the judgment of the Court of First
Instance of Baguio City.
Facts: On September 20, 1965, at about three o'clock in the
afternoon, Margarita Paleng had just arrived in the City from
Tublay in a Dangwa bus. Because it was then raining and the
bus was parked several meters away from the bus station, she
waited inside the bus . After about three minutes of waiting, the
accused came and started molesting her by inquiring her name
and getting hold of her bagShe called the attention of the bus
driver and the conductor about the actuation of the accused, but
it seemed that the former were also afraid of him
"Despite the rain, she left the bus and went to ride in a jeep
parked some 100 meters away. The accused closely followed her

21

(P. 4, id.). When the jeep started to go, the accused also rode and
sat beside her
"When the jeep reached Guisad, she alighted on the road but she
still had to negotiate a distance of ten meters The accused also
alighted and again he tried to carry her bag Although he was not
allowed to carry her bag, he was adamant in following her
"Reaching her boarding house, she opened the door and was
about to close it when the accused dashed in and closed the door
behind him When she entered her room, the accused went in,
pulled a dagger eight inches long and threatened her and then
raped her.
Issue: W/N renting a bedspace in a boarding house constitute
for all and purposes a dwelling.
Held: The judgment of conviction of Amado Daniel for the
crime of rape as charged is affirmed.
Although Margarita was merely renting a bedspace in a
boarding house, her room constituted for all intents and purposes
a "dwelling" as the term is used in Article 14(3), Revised Penal
Code. It is not necessary, under the law, that the victim owns the
place where he lives or dwells. Be he a lessee a boarder or a
bed-spacer, the place is his home the sanctity of which the law
seeks to protect and uphold.
44. PEOPLE VS. MANDOLADO
123 SCRA 128 (1983)
Nature: Appeal from the decision of the Court of First Instance
of Cotabato convicting Martin Mandolado and Julian Ortillano
of murder qualified with aggravating circumstances of treachery,
evident premeditation and abuse of confidence/obvious
ungratefulness.
Facts: Sometime in October 1977, four draftees of third infantry
Batallion were passengers of a bus bound for Midsayap
Cotabato City. Arriving at the terminal, they decided to drink
ESQ rum where Martin after going inside the market and fired
his caliber machine gun. They then boarded a car and forced the
driver to bring them to Midsayap Crossing and in the way,
Herminigildo got his knife and tried to attack the driver. After
they alighted from the jeep, the accused started firing his gun
and hit the occupants of the jeep while Julian fired his armalite
downwards in order to show that they were fighting with some
MILF rebels.
Issue: W/N abuse of confidence and obvious ungratefulness
can be inferred from the mere fact that an army draftee who was
allowed on board a vehicle later on fired his gun at its
occupants.
Held. Decision is modified as to penalty but affirmed in all other
respects.
There is merit in appellants' contention that there could be no
abuse of confidence as the evidence on record showed the lack
of confidence by the victims to the appellants, that this
confidence was abused, and that the abuse of the confidence
facilitated the commission of the crimes. In order that abuse of
confidence be deemed as aggravating, it is necessary that "there
exists a relation of trust and confidence between the accused and
one against whom the crime was committed and the accused
made use of such a relationship to commit the crime." (People
vs. Comendador, 100 SCRA 155, 172). It is also essential that
the confidence between the parties must be immediate and
personal such as would give that accused some advantage or
make it easier for him to commit the crime; that such confidence
was a means of facilitating the commission of the crime, the
culprit taking advantage of the offended party's belief that the
former would not abuse said confidence (People vs. Hanasan, 29
SCRA 534). In the instant case, there is absolutely no showing
of any personal or immediate relationship upon which
confidence might rest between the victims and the assailants
who had just met each other then. Consequently, no confidence
and abuse thereof could have facilitated the crimes.
Similarly, there could have been no obvious ungratefulness in
the commission of the crime for the simple reason that the
requisite trust of the victims upon the accused prior to the
criminal act and the breach thereof as contemplated under

Article 14, par. 4 of the Revised Penal Code are manifestly


lacking or non-existent. In all likelihood, the accused Army men
in their uniforms and holding their high-powered firearms
cowed the victims into boarding their jeep for a ride at machine
gun point which certainly is no source of gratefulness or
appreciation.
45. PEOPLE VS. MARRA
236 SCRA 565 (1994)
Nature: APPEAL from a decision of the Regional Trial Court of
Dagupan City.
Facts: Jimmy Din, recounted that at around 2:00 A.M. on
March 7, 1992, he and his friend, Nelson Tandoc, were
conversing with each other in front of Lucky Hotel located at M.
H. del Pilar Street, Dagupan City, which was owned by the
witness' father and of which he was the administrator. He
noticed a man pass by on the opposite side of the street. The
man made a dirty sign with his finger and Din informed Tandoc
thereof. The man repeated his offensive act and called them by
waving his hands. Infuriated, they followed the man until the
latter stopped in front of the Dunkin' Donuts store at the corner
or Arellano and Fernandez streets. They demanded an
explanation from the man but they were not given any.
At that instant, two men arrived and one of them inquired what
was going on. Tandoc informed him that they were just
demanding an explanation from the man. Din was surprised
when Tandoc unexpectedly slapped one of the two men. A
brawl ensued, with Tandoc clashing with the two men while Din
exchanged blows with the man who made the dirty finger sign.
After the fisticuffs, their three opponents ran away in a
westward direction.
Tandoc and Din then decided to walk back to the hotel. When
they were about to enter the place, they noticed that the men
with whom they just had a fight were running towards them.
Sensing danger, they ran inside the annex building of the hotel
and immediately secured the lock of the sliding outer door. They
entered a room and waited until they felt that the situation had
normalized. After ten to fifteen minutes, thinking that the men
were no longer in the vicinity, they left the room. Having
decided to go home, Tandoc opened the sliding door, All of a
sudden, Din saw appellant, who at that time was wearing a
security guard's uniform, shoot Tandoc with a revolver, There
was a fluorescent bulb installed at the front of the hotel which
enabled Din to identify the assailant. Tandoc was shot in the
middle of the chest and he fell down. Then, Din saw four to five
men scamper away from the scene.
Aware of his injury, Tandoc told Din, "Tol, I was shot." The
latter tried to chase appellant and his companions but he failed
to catch up with them. Din and his wife then brought Tandoc to
the Villaflor Hospital. The victim was taken to the emergency
room but he expired an hour later.
Issue: W/N nocturnity was specially sought by appellant or
taken advantage of by him to facilitate the commission of the
crime or to ensure his immunity from capture.

Juan, Bacon, Sorsogon. 2 other houses in the neighborhood, 150


meters away, cannot be seen because of the fruit tree. Paulina
Dioneda, Normas mom at about 10am of Oct, 9, 1983 (day of
crime), was informed by emmanuels mother that the couple had
an altercation. He slapped and boxed her on the stomach. At
5pm same day, Norma complained that Emmanuel was a jealous
man, even before he manhandled by Emmanuel.
Vicente Dioneda, father of Norma, stated that around 6 or 7pm,
Emmanuel left his child with them. The following morning,
around 6 or 7am, he went to house of Emmanuel and Norma.
He saw plates scattered, rope of hammock missing. He thought
of feeding pig so he climbed the coconut tree. On the third step,
he saw the back of the body of Norma. He went down the tree,
called her, he touched her, and she swayed!
He realized he was hanging, her feet approximately 4 inches
above the ground. He informed his wife and they went to Carlito
Dichoso and fetched the authorities. He saw Emmanuel at the
municipal building of Bacon on Oct. 10, he asked him why he
killed her, he did not answer but just stooped down.
Carlito Dichoso, neighbor, testified that around 6 or 7pm of Oct.
9, Emmanuel went to his house, it was raining. Emmanuel
borrowed a flashlight because he was looking for his wife.
After 2 hrs, Emmanuel returned to Carlitos house, he sat on a
bench. Carlito asked him if he found his wife, he did not answer.
He said that = my wife is continuously possessed with
devils.
Around 5am, Emmanuel told Carlito that if there is something
that happened, Manoy Carlito, what would I do?
Accused was convicted based on circumstantial evidence.
Issue: W o N the place can be considered uninhabited.
Held: The decision appealed from is hereby modired. Aclisedappellant is found guilty beyond reasonable doubt of the
complex crime of parricide with unintentional abortion and
sentenced to suffer the penalty of reclusion perpetua
The aggravating circumstance of uninhabited place is present.
The uninhabitedness of a place is determined not by the distance
of the nearest house to the scene of the crime but whether or not
in the place of the commission, there was reasonable possibility
of the victim receiving some help. Considering that the killing
was done during nighttime and many fruit trees and shrubs
obstruct the view of neighbors and passersby, there was no
reasonable possibility for the victim to receive any assistance.
Note that the trial court convicted accused-appellant of the crime
of parricide only. This is an error. The evidence on record has
shown beyond reasonable doubt that accused-appellant has
committed the complex crime of parricide with unintentional
abortion. The abortion was caused by the same violence that
caused the death of the victim. It is unintentional because
accused-appellant must have merely intended to kill the victim
but not necessarily to cause an abortion.
47. PEOPLE VS. LUG-AW (TREACHERY)
Nature: Appeal from a decision of the RTC of Cabarroquis,
Quirino convicting Julio Lug-aw and Rogelio Bannay of murder
sentencing both to reclusion perpetua or life imprisonment.

Held: The judgment of the court is AFFIRMED.


However, the crime committed by appellant was murder
qualified by treachery, the SC rejected the finding that the same
was aggravated by nighttime. No evidence was presented by the
prosecution to show that nocturnity was specially sought by
appellant or taken advantage of by him to facilitate the
commission of the crime or to ensure his immunity from
capture. At any rate, whether or not such aggravating
circumstance should be appreciated, the penalty to he imposed
on appellant would not be affected considering the proscription
against the imposition of the death penalty at the time when the
offense in the instant case was committed.
46. PEOPLE VS. DESALISA (UNINHABITED PLACE)
299 SCRA 35 (1994)

Facts: On Dec. 12, 1985, Carlos Pal-oy was putting up a fence


allegedly upon the instruction of the public forester to straighten
out the boundary line. His daughters Sonia and Carina were with
him. Pal-oy was proceeding towards the house when Sonia
heard a gun report. Immediately, she went uphill and just as a
second gun report resounded, she saw Rogelio Bannay and Julio
Lug-aw from a distance of around four meters. She saw too that
as her father was about to draw his bolo, Lug-aw shot him.
Trial court appreciated both treachery and evident premeditation
against the accused.
Issue: W o N the trial court correctly ruled that the crime
committed was murder.

Nature: Appeal on the conviction of Emmanuel Desalisa of the


crime of parricide.
Facts:
Emmanuel lived with his 18 year old wife, Norma and 2 year
old daughter on a small nipa house on a hill at Pinaductan, San

22

Held: Appellant Lug-aw is found guilty beyond reasonable


doubt of the crime of homicide and Bannay is acquitted.

Sonias testimony concludes that the crime committed was


homicide instead of murder. The qualifying circumstances of
treachery and evident premeditation had not been proven beyond
reasonable doubt. The trial court drew the conclusion of the
presence of treachery because the attack was sudden as Pal-loy
was simply going about his task of fencing. The SC however,
finds that no one witnessed the initial attack. Absent any
particulars as to the manner in which the aggression commenced
or how the act which resulted in the death of the victim
unfolded, treachery cannot be appreciated to qualify the killing
to murder. Evident premeditation cannot be appreciated because
its requisites are not present.
As to Bannay, his presences at the scene of the crime, unless
conspiracy is proven, do not by itself, indicate criminal
culpability. Conspiracy must be proved beyond reasonable
doubt.
48. PEOPLE VS. CAMILET (EVIDENT
PREMEDITATION)
Nature: Appeal from the judgment of the Regional Trial Court
of Iloilo convicting Regino Camilet of murder and sentenced to
life imprisonment.
Facts: On July 2, 1982 at around 7pm, in Coyogan Sur, Leon,
Iloilo City, a prayer meeting was held at the residence of Brgy.
Capt. Perfecto Camancho Sr. At around 9pm, Dione Camancho,
a dumb nephew of Perfecto arrived crying and making signs that
he was spanked at the buttocks by someone at a certain place.
Accompanied by Dione, Perfecto Camancho, Jr., Rosita Camayo
and Joven Cagayoa and Perfecto, Sr. went to the place indicated
by Dione. When they had walked a distance of around 150
meters, Camilet suddenly stepped from a grove of banana plants
and without word or warning, stabbed Perfecto, Sr. with a onefoot long, sharp-bladed knife. Perfecto, Sr. died as a result.
Prior to the incident, there was a misunderstanding between the
Camanchos and the accused-appellant regarding a portion of
land belonging to Camilets mother-in-law, where Camilet was
working. The settlement reached apparently did not satisfy
Camilet.
The information alleged treachery, evident premeditation,
nighttime and disregard of rank and age as aggravating
circumstances, however, the trial courts decision did not
indicate what circumstances qualified the killing to murder but
considered disregard of rank an aggravating circumstance.
Issue: W o N evident premeditation can be considered to qualify
the killing into murder.
Held: Judgment of trial court modified. Regino Camilet is found
guilty of homicide, instead of murder.
In the absence of a qualifying circumstance, the fatal stabbing of
Perfecto, Sr. is a homicide, not a murder. Treachery cannot be
appreciated in this case. Mere suddenness of an attack is not
sufficient to constitute treachery where it does not appear that
the aggressor adopted such mode of attack to facilitate the
perpetration of the killing without risk to himself. Likewise,
evident premeditation was not established by the prosecution.
Although the facts tend to show that Camilet may have harbored
ill-feelings towards the Camanchos regarding the parcel of land,
there is no evidence of: (1) the time when he determined to
commit the crime, (2) an act manifestly indicating that he has
clung to his determination, and (3) sufficient lapse of time
between determination and execution to allow him to reflect
upon the consequences of his act and to allow his conscience to
overcome the resolution of his will. Nighttime cannot be
considered aggravating since there was no proof that it was
especially sought by the accused to perpetuate the crime. Lastly,
disregard of rank cannot likewise be considered because, there is
no clear evidence that the accused committed the crime in
disregard of the respect due to the victim.
49. PEOPLE VS. ILAOA (CRUELTY)

23

Note: Cruelty exists when the culprit enjoys and delights in


making his victim suffer slowly but gradually causing him
unnecessary pain in the consummation of the act.
Requisites:
injury caused be deliberately increased by causing another
wrong;
the other wrong must be unnecessary for the execution of the
purposes of the offender.
Nature: Appeal from judgment of RTC of Angeles City finding
RUBEN and ROGELIO ILAOA guilty of Murder with the
aggravating circumstances of Evident premeditation, abuse of
superior strength, and cruelty, sentencing them to life
imprisonment.
Facts: On Nov. 5, 1987, Nestor de Loyolas body was found in a
grassy portion of Tinio St., Angeles City. He was decapitated,
had 43 stab wounds on the chest and slight burns all over his
body.
The night before, at around 11pm, de Loyola was seen drinking
with Ruben Ilaoa and 3 others outside Ilaoas apartment. A few
minutes later an argument arose between the two, after which
Ruben and his drinking companions mauled, kicked and dragged
the victim towards the accused apartment. He was heard crying
aray! and pare, bakit niyo ako ginaganito, hirap na hirap na
ako.
At around 2am, Ruben borrowed Alex Villamils tricycle on the
pretext that a neighbor needed to be brought to the hospital as
she was about to give birth. However, he was seen driving alone
with a sack placed in the sidecar, which looked like as if it
contained a human body. When the tricycle was returned, there
wee blood stains on the floor. Susan Ocampo, Rubens live-in
partner, was also seen sweeping blood at the entrance of their
apartment and blood was found on his shirt when police
investigated him and the hair near his right forehead was partly
burned.
Held: Rogelio was acquitted, while Ruben was convicted for
homicide only.
Rogelio was acquitted because the circumstances relied upon to
establish his guilt, particularly the dragging of the victims body
into the house of his brother, is totally inadequate for a
conviction.
Rubens liability was modified to homicide only because the
qualifying circumstances were not sufficiently proved. There
was no evidence whatsoever that appellant was physically
superior to the deceased and tat he took advantage of such
superior physical strength to overcome the latter. (Mere
numerical advantage is not enough to be qualified as an abuse of
superior strength). Number of wounds alone is not the criterion
for the appreciation of cruelty, neither can it be inferred from the
mere fact that the victims dead body was dismembered. There
was no showing that appellant for his pleasure and satisfaction
caused the victim to suffer slowly and painfully and inflicted on
him unnecessary physical and moral pain. Evident premeditation
cannot be appreciated also since there is no proof that such
killing was the result of meditation, calculation or resolution on
the part of Ruben.
50.PEOPLE VS. MARQUEZ (CRAFT, FRAUD,
DISGUISE)
117 SCRA 165 (1982)
Nature: An appeal from the judgment of the Court of First
Instance of Quezon finding the accused guilty of the crime of
robbery with rape as defined under Article 294, paragraph 2 of
the Revised Penal Code and sentenced them to life
imprisonment.
*note: Renato Marquez died during trial.
Facts: On November 16, 1966, between seven and seven-thirty
in the evening, Francisca Marquez-Tan was in their house in
barrio Dahican Catanauan, Quezon together with her seven
children and maid Rufina Martinez. She heard somebody call
out in front of their window who identified themselves as PC
soldiers looking for contraband. She replied that they did not
have any contraband and that her husband, Angel Tan, was in
the poblacion at that time. The men ordered her to open up

otherwise they will shoot up their house. She opened up her


window and Renato jumped inside. She was able to recognize
Renato Marquez as the light was bright, and as he was her
distant relative. Renato held her by the nape and pushed her
towards the door and at gunpoint ordered her to open the same.
When she opened the door, accused Samuel Jacobo and
Francisco Forneste, both armed with guns, entered and ordered
her to put out their contraband and when she told them that they
did not have any, the intruders demanded for money. She
pointed at the table, which Renato open and took P300 there
from. Jacobo also pried open their aparador where he got P200.
Jacobo also dispossessed her of her ring worth P15.00 and a pair
of earrings worth also that much. At that instant, the other
accused Forneste was upstairs guarding her children and helper.
Samuel Jacobo asked her why they have only a small amount of
money when they are copra-buyers and she replied that they
were just starting on their business. Whereupon, Jacobo
demanded: "kuarta o buhay" so that she put out her pillow which
Jacobo grabbed and ripped open and took there from P820.00.
Afterwards, Samuel Jacobo raped her at gunpoint while Renato
ransacked their store and took merchandise there from. After
five minutes, Jacobo took her upstairs and tied both her arms
and made her he face down on the floor together with her
children. Subsequently, her daughter Leticia and helper Rufina
Martinez were taken downstairs by Francisco Forneste. Shortly
after the men left, the victims were able to free themselves and
Francisca found out that the two women were raped also.
Leticia Tan corroborated the foregoing testimony of
her mother. The rapes, according to her were committed in the
following manner: when her mother, Francisco Marquez was
taken upstairs, Renato Marquez brought her downstairs, to their
store. Inside the store Renato Marquez to her " to give
something and if I refused I would be killed. Simultaneously
Renato Marquez "poked a gun and also a balisong" at her
causing her to be afraid she called for her mother but then she
was told not to shout because "I am going to be killed."
Thereafter, she was forcibly made to lie down and Renato
Marquez committed the act on her. Leticia related that during
the time that she was with Renato Marquez, Rufina Martinez
was with Francisco Forneste and that immediate after the
departure of Renato Marquez, Francisco Forneste and Samuel
Jacobo, Rufina Martinez told her that she was also raped by
Francisco Forneste.

three times. Crisologo joined in and with his own knife also
stabbed Fidel. At this time, gunshots were heard outside of the
house; and a neighbor of the Saromineses, Balbino Bulak,
recognized one of those doing the shooting as certain Carlito
Antiga. 8 A voice was heard from below saying, "Stab him!" 9 to
which Langomez replied, "I already stabbed (him)."
From his little sister's room, Fidel's thirteen-year old
son, Peter, saw his father fighting for his life with Romualdo and
Crisologo Empacis. Heeding his father's cry for help, Peter took
hold of a "pinuti" (a long bolo), and struck inflicting wounds on
the latters shoulder and neck. The two accused jumped out of
the house and fled. Peter then turned to his wounded father, but
found him already dead from his injuries.
Crisologo Empacis repaired to the clinic of Dr.
Eustaquio Deiparine at the poblacion of Sibonga, Cebu, for
treatment of the wounds inflicted on him by Peter, arriving there
between 10 and 11 o'clock that same night. Dr. Deiparine asked
Crisologo how he had come by these wounds. Crisologo said
that at around 6 to 7 o'clock that evening, near the Papan
Market, he was assaulted without warning by a young man, who
injured him with a bolo.
The next day police officers went to Dr. Deiparines
clinic and asked for information regarding a man who might
have been treated for hacking wounds. They were directed to the
public market where Empacis was and he was arrested.

Held: The evidence adduced is not sufficient to show any


conspiracy among the accused in the commission of the crime of
rape against the persons of Francisca Marquez, Leticia Tan and
Rufina Martinez. Therefore, the lower court was correct in
concluding that the crime committed by the accused appellants
was robbery with rape not robbery with multiple rapes as alleged
in the information.

Nature: an appeal from the decision of the Regional Trial Court


of Davao City, Branch 13, convicting the accused, Fordito
Ruelan, of the crime of MURDER and imposing on him the
penalty of "life imprisonment."

51. PEOPLE VS. EMPACIS


222 SCRA 59 (1993)
Nature: An appeal from the decision of the Regional Trial
Court of Cebu City, Branch 14 finding the accused guilty of
robbery with homicide as defined and penalized under Article
294 (1) of the Revised Penal Code, and considered the
attendance of the four generic aggravating circumstance of
dwelling, nighttime, craft or fraud and superior strength, not
offset by any mitigating or extenuating circumstance, sentencing
him to death.
Facts: At about 9 o'clock on the night of September 16, 1986, as
Fidel Saromines and his wife, Camila, were about to close to
their small store, located in their house at Kanguha, Dumanjug,
Cebu, two men came and asked to buy some sardines and rice.
They were Romualdo (or Maldo) Langomez and Crisologo
Empacis. Camila served them and they proceeded to make a
meal of the rice and sardines.
After they finished eating, Romualdo told Fidel to sell
him cigarettes. As Fidel was handing over the cigarettes,
Romualdo announced a "hold-up" and commanded Fidel to give
up his money. As it happened, Fidel then had P12, 000.00 in his
house, wrapped in cellophane. This he started to give to
Romualdo but as the latter was taking hold of the packet, Fidel
suddenly decided to fight to keep his money. A struggle
followed in the course of which Romualdo stabbed Fidel about

24

Issue: Whether or not craft, fraud or disguise was used for the
commission of the crime
Held: Judgment affirmed, modification in costs. The
aggravating circumstance of craft or fraud was properly
appreciated against Empacis. He and Romualdo pretended to be
bona fide customers of the victim's store and on his pretext
gained entry into the latter's store and later, into another part of
his dwelling.
The SC has held stratagems and ruses of this sort to constitute
the aggravating circumstance of fraud or craft, e.g.: where the
accused a) pretended to be constaabulary soldiers and by that
ploy gained entry into the residence of their prey whom they
thereafter robbed and killed.
52. PEOPLE VS. RUELLAN (SUPERIOR STRENGTH)
231 SCRA 650 (1994)

Facts: On August 4, 1988, Spouses Ricardo and Rosa Jardiel


hired appellant as a store helper at their store located in
Bankerohan Public Market, Davao City. Appellant helped
Jardiel spouses in selling and delivering rice to various
customers. He stayed in the couple's residence but he had
separate quarters for sleeping.
On August 18, 1988, at around 4:00 a.m., Ricardo
Jardiel was roused by the closing of the bedroom door and he
saw his wife Rosa Jardiel leaving his room. Ricardo Jardiel
stood up and followed his wife who went towards the gate of the
house. Rosa Jardiel was joined by appellant since they would
open the store in Bankerohan Public Market. Rosa Jardiel talked
to appellant and ordered him to bring an axe which would be
used in repairing some fixtures in the store. Appellant followed
her order and took an axe and sack. When they were about to
leave the premises, Rosa Jardiel's house dog got loose and went
out towards the street. Rosa Jardiel got angry and scolded
appellant while she walked ahead of him along Tulip Drive
going to McArthur Highway. Appellant pleaded Rosa Jardiel to
stop berating him but Rosa Jardiel did not heed to his request.
Appellant got fed up and with the use of his axe, he struck Rosa
Jardiel behind her right ear causing her to fall face down.
Thereafter, appellant dragged Rosa Jardiel to a grassy portion at
the side of the street and then immediately left the place.
Issue: W o N there was abuse of superior strength.
Held: Judgment modified. Accused is guilty only of homicide
and ordered to suffer a penalty of six (6) years and one (1) day

of prison mayor as minimum to fourteen (14) years, eight (8)


months and one (1) day of reclusion temporal as maximum.
To properly appreciate the aggravating circumstance
of abuse of superior strength, the prosecution must prove that
the assailant used purposely excessive force out of proportion to
the means of defense available to the person attacked.19 In the
instant case, the appellant clearly took advantage of his superior
strength as the victim while the appelllant was then only 20
years old, of good statute and build and was armed with an axe
with which to kill the victim. However, the aggravating
circumstances of abuse superior strength cannot qualify the
killing of the victim and raised it to the category of murder
because the same was not alleged in the information. The rule in
case like this is clear. A qualifying circumstance like abuse of
superior strength must be pleaded in considered as a generic
aggravating circumstance in the imposition of the correct
penalty.
53. PEOPLE VS. PADILLA
233 SCRA 46 (1994)
Nature: An appeal from a decision of the then Court of First
Instance of Catbalogan, Samar finding the accused guilty of
murder qualified by treachery with the generic aggravating
circumstance of taking advantage of his public position, but
appreciating at the same time the mitigating circumstance of
sufficient provocation and imposing the penalty of reclusion
perpetua.

employee at the nearby Bahay Kawayan Disco situated along


Callejon, a narrow street connecting Rizal Avenue. Lilia
struggled and screamed for help as Pfc. Ontuca held her tightly
by the waist, using her as a human shield against the accused
who was pointing his pistol at him. Finally, Pfc. Ontuca and
Lilia fell to the ground giving the latter a chance to escape. Left
without any protection, Pfc. Ontuca squatted on the ground and
reached for a piece of plywood which he held upward to cover
his head.
Maj. de la Cruz and Pat. Omega was just across the
street standing in front of the Bonifacio Nardo Store some
fifteen (15) meters away. From where they stood, they could
clearly see the side view of the accused and the victim facing
each other.
Pfc. Ontuca begged for his life. "I am not going to
fight with you," he said. But the accused, showing no mercy,
squeezed the trigger of his .45 cal. automatic pistol pumping a
single bullet into the head of his victim who was just some three
to four meters from him. The time was exactly two o'clock in the
morning. After shooting Pfc. Ontuca, the accused backtracked
and then returned to the fallen policeman and tauntingly kicked
him saying, "Are you still alive?" The accused then went to Maj.
de la Cruz and talked to him.
Issue: Whether or not there was an abuse of superior strength

Facts: On 4 May 1981, Pat. Omega was on duty at Pier 1 in


Catbalogan, Samar, from eleven o'clock in the evening to seven
o'clock the following morning. At past midnight, 5 May 1981,
Pfc. Edino Ontuca, Officer-in-Charge of the Talalora Police
Sub-Station, approached him for assistance claiming he was
maltreated by strangers. Pat. Omega responded and both
proceeded to where complainant was reportedly assaulted,
passing by Malayan Hotel at the pier area to get Ontuca's service
revolver.
When they reached the corner of Rizal Avenue and
Del Rosario Street, the two (2) policemen saw three (3) men in
the company of a woman. Pat. Omega approached them,
identified himself as a police officer, and then began to
investigate the reported "castigo" or manhandling of
complainant. A certain C1C Belino spoke for the group. He
introduced himself first and then his companions, Maj. de la
Cruz and Sgt. Padilla. Pat. Omega tapped Pfc. Ontuca's shoulder
and admonished the latter saying, "Brod, let us stop; just keep
silent; just go home and sleep. Pfc. Ontuca took the advice and
returned to the pier with Pat. Omega.
When they reached Malayan Hotel, Pfc. Ontuca stayed
behind at the entrance while Pat. Omega proceeded to the Lion's
Waiting Shed some fifty (50) meters away. Suddenly, there was
a commotion in front of the hotel. Pfc. Ontuca was being ganged
up by three (3) men. Pat. Omega then rushed towards the hotel
where he saw his companion already down on his right knee
with both arms stretched behind him, his left arm held by Maj.
de la Cruz and his right by Sgt. Padilla, while C1C Belino held
him by the waist and took his service revolver away. Pat.
Omega tried to intervene but C1C Belino and Maj. de la Cruz
poked their pistols at him. C1C Belino disarmed Pat. Omega and
handed over his service pistol to Maj. de la Cruz who then
grabbed Omega by the collar saying, "Let us go to the hospital
because you are drunk."
Pat. Omega denied that he was drunk. Maj. de la Cruz
summoned a certain Sgt. Bongosia to accompany them to the
hospital purportedly in order to have the two policemen undergo
"liquor
test".
Sgt. Bongosia obliged and then cocked his armalite.
All six (6), namely, Maj. de la Cruz, Pat. Omega, Pfc.
Ontuca, C1C Belino, Sgt. Bongosia and the accused then
proceeded east along Curry Avenue. Upon reaching the
intersection of Rizal Avenue and San Bartolome Street, Pfc.
Ontuca turned right, eastward. The accused followed by Maj. de
la Cruz and Pat. Omega, pursued Pfc. Ontuca, while Sgt.
Bongosia did not take the same route; he ran straight along
Curry Avenue and then turned left, north, along San Francisco
Street. C1C Belino dashed to the opposite direction, turning left
towards Del Rosario Street. When Pfc. Ontuca reached a fruit
stand beside Cinex Theater, he grabbed a girl named Lilia, an

25

Held: Judgment affirmed. The killing was qualified by the


aggravating circumstance of abuse of superior strength which
was alleged in the information and proved during the trial.
Abuse of superior strength is present not only when the
offenders enjoy numerical superiority, or there is a notorious
inequality of forces between the victim and the aggressor, but
also when the offender uses a powerful weapon which is out of
proportion to the defense available to the offended party. The
accused was armed with a powerful pistol, which he purposely
used, gaining him an advantage over his victim who only had a
piece of plywood to cover himself after he was disarmed.
The accused did not abuse his public position in
committing the crime. For this circumstance to be appreciated as
aggravating, the public official must use his influence, prestige
and ascendancy, which his office gives him in realizing his
purpose. It could not be said that the accused purposely used or
took advantage of his position or rank in killing the victim
because he could have committed the crime just the same by
using another weapon not necessarily his service firearm.
54. PEOPLE VS. VERCHEZ
233 SCRA 174 (1994)
Nature: An appeal from the Decision of the Regional Trial
Court, Branch 19, Bacoor, Cavite convicting the accused of
murder qualified by treachery.
Facts: Verchez invited Balane on August 15, 1985 to visit his
brother, who was then living in Queen's Row Subdivision,
Bacoor, Cavite. On their way, the two chanced upon Aldave, a
"compadre" of Verchez' brother, who joined them. They arrived
at their destination at about 2:30 P.M. but found that Verchez'
brother was not at home. However, they saw Alfredo
Mamuntag, the caretaker of the house, Alfredo's son, Hector,
and Gilbert Ang, who were then visiting with Alfredo.
Verchez decided to wait and drink liquor at the yard.
At about 3:00 P.M., Balane drove away to buy cigarettes and
"pulutan." He had not driven far when a car blocked his way,
with the occupants pointing their firearms at him. Then another
car arrived. One of the passengers from the second car
approached Balane and frisked him. Thereafter he was dragged
out of the car, handcuffed and blindfolded. After he was boxed
on the face and stomach, he was pushed inside a car.
Verchez saw several cars stop in front of the house.
Men in civilian clothes with firearms alighted from the cars. One
of the men ordered him to open the door. Suddenly, he heard a
gunshot and Aldave, who was then at the back of the house,
shouted that someone took a shot at him. After the two ran
inside the house, they heard more gunshots.
Verchez got a loaded M-16 Armalite rifle from one of
the rooms and fired back at his attackers. Aldave looked around
and found an Armalite rifle. He also fired back.

Balane, still blindfolded and handcuffed, was ordered


by Capt. Castaneda to advise his companions to surrender.
Hence, he shouted, "Sumuko na kayo si Vic ito." However,
someone also shouted "Huwag na kayong sumurender,
papatayin nalang namin kayo."
The firing continued for 15 minutes, after which the
police were able to enter the house. Verchez and Aldave,
together with the other occupants of the house, surrendered and
were brought to Camp Crame.
Verchez and Aldave claimed that at Camp Crame,
they were tortured into admitting participation in several bank
robberies. They were forced into signing a prepared statement
confessing their illegal activities, including having engaged the
police officers in a fire fight on August 15, 1985.
Issue: Whether or not there was treachery
Held:
In convicting appellants of murder, the trial court
ruled that the killing of Sgt. Norcio was qualified by treachery as
the firing of the guns was sudden and unexpected. We find,
however, that treachery was not sufficiently established.
For the qualifying circumstance of treachery to be
present, two conditions must concur: (a) the employment of
means of execution that gives the person attacked no
opportunity to defend himself or to retaliate; and (b) that said
means of execution was deliberately or consciously adopted
(People v. Dela Cruz, 207 SCRA 632 [1992]).
There is no showing that appellants deliberately and
consciously adopted their mode of attack. Neither is there any
showing that they planned to ambush the lawmen, much less
that they knew that the lawmen were coming. What is apparent
is that appellants were caught by surprise by the lawmen, hence,
acting on the spur of the moment, they fired back.
Absent the qualifying circumstance of treachery,
appellants can only be convicted of homicide for the death of
Sgt. Norcio and frustrated homicide for the wounding of Cpl.
Noora.

There was (implied) conspiracy between Jerry and Baello


when they went together to the house to rob it These acts as a
whole are more than sufficient to establish common design, joint
purpose and a community of interest.
(RE: UNLAWFUL ENTRY) The aggravating circumstance of
unlawful entry was appreciated because the accused, using the
2nd floor window for entry, used an opening not intended for
ingress or egress, therefore unlawful entry.
56. PEOPLE VS. RENEJANE (INTOXICATION)
Nature: Appeal from the decision of the RTC of Cebu finding
Beniano Renejane guilty beyond reasonable doubt of the crime
of double murder sentencing him to reclusion perpetua in both
cases.

55. PEOPLE VS. BAELLO (UNLAWFUL ENTRY)

Facts: On Nov. 1, 1981, Reynoso and Regino Mara-asin, Pablo


Sumandig, Patrolman Mario de Jesus and his wife, Violeta, and
Michael Madrigal were in the house of Artemio Ripdos in Brgy.
Lamesa, balamban, Cebu. At around 5pm, BENIANO
RENEJANE, NICK LABORTE, PAULINO LABORTE and
PURISIMA arrived and invited the Mara-asins, Sumandig, de
Jesus and Madrigal to the accused home to partake some food.
From Renejanos house, they then proceeded to Nick Labortes
house and then to his moms house and finally to Paulinos
home. Reynoso, however, did not join them at Paulinos house
but went instead to his parents house to inform them of his
whereabouts. Upon reaching Paulino Labortes house (at around
11 pm), an altercation between de Jesus and Renejane broke out
regarding the appellants apprehension by de Jesus for illegal
possession of marijuana last Oct. 21, 1981. (Regino was
suspected as the informer) While the patrolman was arguing
with the accused, Paulino Laborte pulled out a knife and stabbed
him. The accused stabbed the victim the 2nd time. Regino was
killed too.
The lower court appreciated abuse of strength, outraging the
victims corpses, disregard of rank and drunkenness against the
accused.

Nature: Appeal from the judgment of the RTC of Pasi


convicting Baello of Robbery with Homicide and sentencing
him to reclusion perpetua.

Held: Decision modified with regard to penalties. In each case, a


minimum of reclusion temporal in its maximum to reclusion
perpetua.

Facts: On October 10, 1990, at around 4am, JOHN AMMET


BAELLO together with a certain JERRY unlawfully entered,
through the 2nd floor window, the house of BRGY. CAPT.
EUSTAQUIO BORJA and stole 1 colored TV, 1 stereo cassette
player, 1 camera plus assorted jewelry, over all amounting to
P64, 000+.

Only the disregard of rank can be appreciated. Drunkenness or


intoxication is aggravating if it is habitual or intentional. It can
neither be considered mitigating in the absence of proof that the
intake of alcoholic drinks was of such quantity as to blur the
appellants reason and deprive him of a certain degree of
control. As to abuse of strength, it is inherent in treachery and
therefore deemed absorbed. Outraging the victims corpses
should not be appreciated as assailants were carried away by the
intensity of their attack as attested by the nature of the wounds
inflicted but had no desire to add ignominy to the offense.

Borja woke up at around 5 AM to discover that the front door of


their house was open and their TV was missing. He informed his
wife and together they checked on their daughter, VERONICA
(22) upstairs and discovered that she was dead due to multiple
stab wounds.
At 6 PM of the same day, the police were able to recover the TV
set at the house of Eugenio Tagipa, Baellos brother-in-law.
Tagipa pointed to the accused as the one responsible for placing
the TV under the stairs of his house. On Oct. 13, at around 5:30
pm, accused was arrested by the Intelligence and Special
Operations Unit of Pasig Police. He made an oral admission to
the robbery but denied having any knowledge of the killing of
Veronica as he was already downstairs stealing the TV while
Jerry remained upstairs.
Issue: W o N the accused is guilty of robbery with homicide
even if he had no knowledge of the killing.
Held: Judgment affirmed. (Art. 4- under the doctrine of
proximate cause)
Even though Baello had no knowledge or participation in the
killing of Veronica, he is not absolved from any liability of her
death. The rule is likewise settled that when homicide takes
place as a consequence, or on the occasion of a robbery, all
those who took part in the robbery are guilty as principals of the
crime of robbery with homicide unless there is proof that the
accused tried to prevent the killing.

26

57. PEOPLE VS. CAMANO


115 SCRA 258 (1982)
Nature: A review of the death sentence imposed upon the
accused Filomeno Camano by the Court of First Instance of
Camarines Sur, for the killing of Godofredo Pascua and Mariano
Buenaflor.
Facts: In 1967, the two victims had a misunderstanding with the
accused while fishing along Sagnay River. During this occasion
it appears that the accused requested Godofredo Pascua to low
his fishing boat with due motor boat owned by Mariano
Buenaflor but the request was refused by both. This refusal
greatly offended and embittered the accused against the victims.
From this time on, the accused begrudged the two, and
entertained personal resentment against them. It was noticed
that defendant when intoxicated or drunk, used to challenged
Mariano Buenaflor to a fight, and announce his evil intention to
kill them.
On February 17, 1970, after the accused had been drinking
liquor, he stabbed twice the victim Godofredo Pascua with a
bolo while the latter was walking alone along the barrio street
almost in front of the store of one Socorro Buates. The victim,
Godofredo Pascua, sustained two mortal wounds for which he

died instantaneously.
After hacking and stabbing to death Godofredo Pascua, the
accused proceeded to the seashore of the barrio, and on finding
Mariano Buenaflor leaning at the gate of the fence of his house,
in a kneeling position, with both arms on top of the fence, and
his head stooping down hacked the latter with the same bolo,
first on the head, and after the victim fell and rolled to the
ground, after said blow, he continued hacking him, until he lay
prostrate on the ground, face up, when the accused gave him a
final thrust of the bolo at the left side of the chest causing instant
death.
The trial court found the accused guilty of two counts of
murder with the aggravating circumstance of evident
premeditation and intoxication.
Issued: Can intoxication be an aggravating circumstance to the
case?
Held: The judgment is modified because of attending mitigating
circumstance of intoxication.
It is respectfully submitted that there was no proof that
the accused was intoxicated at the time of the killing other than
the bare testimony of Payago that from his house he allegedly
saw the accused drinking in his house which is about 30 meters
away. The prosecution did not present any police report or
doctor's certification that accused was found to be intoxicated at
the time of the killing. Moreover, it was not shown by
competent evidence that accused purposely became drunk to
facilitate the commission of the offense.
"If at all, intoxication should be properly appreciated as a
mitigating circumstance because it affected accused mental
faculties such that it diminished his capacity to know the
injustice of his acts and to comprehend fully the consequences
of his acts."14
There is merit in the contention. Drunkenness or intoxication is
mitigating if accidental, not habitual nor intentional, that is, not
subsequent to the plan to commit the crime. It is aggravating if
habitual or intentional.1 To be mitigating, it must be indubitably
proved.1 A habitual drunkard is one given to intoxication by
excessive use of intoxicating drinks. The habit should be actual
and confirmed. It is unnecessary that it be a matter of daily
occurrence. It lessens individual resistance to evil thought and
undermines will-power making its victim a potential evildoer.1
The records of these cases do not show that the appellant was
given to excessive use of intoxicating drinks although he used to
get drunk every now and then.
58. ATTY. AQUILINA R. ARANETA VS. COURT OF
APPEALS
NATURE: A petition to review the decision of the then Court of
Appeals finding the accused-appellant guilty of the crime of
bribery.
FACTS: Gertrudes Yoyongco is a widow of Antonio, an
employee of the National Irrigation Administration (NIA).
When Antonio died, she approached the appellant, a hearing
officer of the Workmens Compensation Unit, to inquire on the
procedure for filing a claim for death compensation, which upon
learning prepared and filed them. After a few days, she went
back to ask about the status of her claim. When she saw the
appellant, she was told she needed to pay P100 for her claim to
be acted upon. She then complained to her brother-in-law, Col.
Yoyongco. The former then instructed Carlito Carlos to entrap
the accused. Two 50 peso bills were dusted with ultra-violet
powder. The two, with Balos, then went to the appellant, with
Carlos posting as the nephew of the widow. When the appellant
asked for the money, Balos grabbed Araneta and arrested her.
ISSUE: W o N it is entrapment or instigation.
Held: Decision of the lower court affirmed. Petition dismissed
for lack of merit.
The petitioner confuses entrapment with instigation.
There is entrapment when law officers employ ruses and
schemes to ensure the apprehension of the criminal while in the
actual commission of the crime. There is instigation when the
accused was induced to commit the crime. The difference in the

27

nature of the two lies in the origin of criminal intent. In


entrapment, the mens rea originates from the mind of the
criminal. The idea and the resolve to commit the crime comes
from him. In instigation, the law officer conceives the
commission of the crime and suggest to the accused who adopts
the idea and carries it into execution.
59. CABRERA VS. PAJARES
142 SCRA 127 (1986)
Nature: ADMINISTRATIVE MATTERS in the Supreme
Court.
Facts: "On January 16, 1985, the complainant Enrico Cabrera
gave a sworn statement to the National Bureau of Investigation
in Naga City, denouncing the respondent Judge James B. Pajares
for having allegedly asked money from him in connection with
his case. Cabrera said that in September, 1984 Judge Pajares
intimated to him that he needed money. Cabrera said he gave
P1, 000.00 to the respondent judge because the latter had been
unduly strict, preventing him from making statements during the
trial of his case.
It appears that the complainant is the defendant in
Civil Case No. R-751 which the respondent judge was trying.
The case was filed by the complainant's father, Juan Cabrera,
and by his half brothers and sisters, for the annulment of the sale
made to the complainant of about 28 hectares of land in San
Juan, Canaman, Camarines Sur. (See Exhs. 6 and 7-B) Cabrera
said he had been advised by his counsel, Atty. Roberto
Verdadero, to accommodate any request for money from the
respondent so that he would not be unduly hard on the
complainant. In September, 1984, according to the complainant,
Judge Pajares intimated to him that he needed money. Following
his counsel's advice, Cabrera said he expressed willingness to
help the judge financially and, the following day after their
meeting, gave him P1, 000.00. However, according to Cabrera,
after two months (i.e., before Christmas of 1984), Judge Pajares
again told him that he needed money. Cabrera said the judge
saw him in front of the Hall of Justice in Naga City and called
him. It was then, according to him, that he decided to denounce
the judge to the authorities; Cabrera asked the assistance of the
NBI in entrapping Judge Pajares.
Issue: Whether or not the accused I guilty of the crime of
indirect bribery.
Held: ACCORDINGLY, respondent Judge is hereby dismissed
from the service, with forfeiture of all retirement benefits and
pay and with prejudice to reinstatement in any branch of the
government or any of its agencies or instrumentalities. The
Clerk of Court is hereby ordered to return the ten P100, 00 bills
(Exhibits D-1 to D-10) to the complainant Atty. Enrico M.
Cabrera. This decision is immediately executory.
60. PEOPLE VS. VENERACION (ACTS NOT COVERED
BY LAW)
249 SCRA 244 (1995)
Nature: Petition for certiorari to review a decision of the
Regional Trial Court of Manila, Br. 47
Facts: Two criminal cases were consolidated to Branch 47 of
RTC of Manila and presided over by respondent Judge Lorenzo
Veneracion regarding the brutal rape and killing of Angel
Alquiza, a 7 year old girl on August 2, 1994. After trial and
presentation of the evidence of the prosecution and defense, the
trial court rendered the decision on January 31, 1995 finding the
defendants Henry Lagarto and Ernesto Cordero guilty beyond
reasonable doubt of the crime of Rape with Homicide and
sentenced both accused with the penalty of reclusion perpetua
instead of imposing the penalty of death as provided for in RA
7659, Sec. 11.
Issue: W o N the respondent judge acted with grave abuse of
discretion and in excess of jurisdiction when he failed and/or
refused to impose the mandatory penalty of death after finding
the accused guilty of the crime rape with homicide.

Held: The petition is granted and the case is remanded to the


RTC for the imposition of the penalty of death.

and the other two, namely Danilo Carpio and Efren Salangsang,
by indispensable cooperation.

The Rules of Court mandates that after an adjudication of guilt,


the judge should impose the proper penalty and civil liability
provided for by the law on the accused. In the case at bar, a
judge fully aware of the appropriate provisions of the law,
refuses to impose a penalty to which he disagrees. In so doing,
he acted without or in excess of his jurisdiction with grave abuse
of discretion amounting to a lack of jurisdiction.

62. PEOPLE VS. DELA CERNA


21 SCRA 569 (1967)

The law plainly and unequivocably provides that when by


reason or on the occasion of rape, a homicide is committed, the
penalty shall be death. The provision leaves no room for the
exercise of discretion on the part of the trial judge to impose a
penalty under the circumstances described, other than the
penalty of death. As long as the death penalty remains in the
statute books, and as long as our criminal law provides for its
imposition in certain cases, it is the duty of judicial officers to
respect and apply the law regardless of their private opinions.
The only function of the judiciary is to interpret the laws and, if
not in disharmony with the Constitution, to apply them.
61. PEOPLE VS. NUNAG
173 SCRA 274 (1989)
Nature: An appeal from the judgment of the Court of First
Instance of Pampanga, Branch I
Facts: On May 1978 at about 7:30 pm, Lorenza Lopez, then
about fifteen and a half years old, was watching a television
program in the house of her neighbor, Carmen Laxaniana. She
stood outside the house and peeped through the open window.
As she was standing there, she saw the accused Mario Nunag,
one of her neighbors, coming towards her Mario Nunag was
staggering and appeared to be drunk. The moon was bright and
she really recognized him.
Mario Nunag came to her and asked her to go with
him. But she refused, so that Nunag held her by the hand and
poked a knife at her stomach and threatened to kill her. Nunag
then placed something in her month and led her to a nearby rice
field, about 15 meters behind the house of Carmen Laxamana. 1
Very soon thereafter, they were joined by the other accused
Arnel Mandap, Efren Salangsang, Danio Carpio and Diosdado
Manalili, who were also very well known to her. After
conferring in whispers, Arnel Mandap and Efren Salangsang
held her hands, while Danilo Carpio and Diosdado Manalili held
her feet, and forced her to be on the ground. She struggled to
free herself, but the accused held her tightly.
Mario Nunag then undressed her and had sexual
intercourse with her, at the same time fondling her breasts. She
felt pain in her vagina. After Mario Nunag had finished, Arnel
Mandap followed. After Arnel Mandap had finished, she lost
consciousness and regained it while Diosdado Manalili was
abusing her.
Then, the five accused left, after warning her not to
report the incident to anybody. Otherwise, they would kill her,
her parents and brothers. The complainant felt pains and aches
all over her body, especially in her breasts and vagina. She
rested for a while and when the pains had somewhat subsided,
she went home. She did not report the incident to anybody for
fear of what the accused might do to her and her family.
Issue: W o N the accused should be guilty of 5 counts of rape by
virtue of conspiracy existing among them.
Held: Judgment modified.
It would appear, however, that there is no conclusive
evidence that the accused-appellants Danilo Carpio and Efren
Salangsang had sexual intercourse with the complainant, since
the complainant said that she lost consciousness after the second
man (Arnel Mandap) the first being Mario Nunag-had sexually
abused her and she regained consciousness while Diosdado
Manalili was abusing her sexually, and that she merely assumed
that Danilo Carpio, and Efren Salangsang had also sexually
abused her. Consequently, each of the five (5) accusedappellants must be found guilty of three (3) district and separate
crimes of rape, the first three, namely, Mario Nunag, Arnel
Mandap and Diosdado Manalili, by direct act and participation

28

Nature: An appeal from a decision of the Court of First Instance


of Cotabato finding the accused guilty for double murder.
FACTS: Early in the morning of February 3, 1958, Rafael
Cabizares, accompanied by his wife, Hospicia, his brothers
Margarito and Romualdo, and his sons Gumercindo, Marcelo,
Casiano, Juan and Lamberto, left Barrio Cebuano headed for the
poblacion of Tupi, Cotabato, bringing five sacks of corn loaded
on a bull cart to be milled in Tupi, Juan, Marcelo and Lamberto,
who were all minors, were then going to school. Upon
approaching a hilly part, they had to stop since the carabao could
not pull the bull cart uphill. Rafael then requested his two
brothers and his son Gumercindo to accompany him up the hill
and carry on their backs the sacks of corn. With Rafael leading,
the four proceeded uphill.
As the four approached Sulpicio de la Cerna's house
oiltop of the hill and were about to put down the sacks of corn,
appellant Sulpicio, who was in the house, fired at and hit Rafael,
who fell down. Sulpicio then ordered his companions to burn his
house so that they would have an excuse. Meanwhile, Casiano,
Gumercindo, Marcelo and Romualdo brought the wounded
Rafael Cabizares to the house of the latter's father, Demetrio,
100 meters away. Felisa Bastismo, Rafael's mother, Ursula
Cabizares and Segundino Cabizares were there at the time.
After the group reached the house, Rafael's wounds
were washed with hot water and then he was brought inside the
third room of the house. Subsequently, appellant Sulpicio and
the other accused arrived at the premises, armed with firearms,
bolos and canes. They stoned the house and thrust their bolos
thru the bamboo walls and flooring. Finding that there were
women inside the house, the accused ordered them to get out or
else they would be killed also. As Felisa Bastismo and Ursula
Cabizares alighted from the besieged house, Marcelo Cabizares
followed them, and although held by accused Conrado Pardillo
and boxed by Serapio Maquiling, he was able to escape to the
nearby forest.
Serapio Maquiling then climbed up the window of the
kitchen, and with the carbine which he got from appellant
Sulpicio de la Cerna, shot at Rafael Cabizares who was sitting in
the third room. At this moment, Casiano Cabizares jumped
down from the house thru the kitchen door and ran away.
Serapio Maquiling followed him and shot the latter at the back,
killing him a few meters away from Demetrio's house. Appellant
Sulpicio de la Cerna then got back the carbine, climbed up the
house and fired once more at Rafael, who was now lying down
on the floor, killing him finally. Thereafter, the cadaver of
Casiano Cabizares was tied to a bamboo pole, carried by
accused Ramon Alquizar and one Wilfredo Malias (at large) and
placed near the burned house of Sulpicio de la Cerna, as some of
the accused followed while the rest proceeded to Rafael's house.
ISSUE: Whether the five appellants are all guilty as principals?
Held: The five appellants guilty as co-principals in the murder
of Rafael Cabizares.
The positive identification of the several prosecution
witnesses must prevail over the alibis proffered by these
appellants. Their presence and active participation in the
meeting in Abapo's house make them actual conspirators in the
killing of Rafael. They were also present and zealously
participating in the execution of their criminal design, giving a
carbine magazine and instructionns to appellant Rotor,
threatening Rafael and giving encouragement to Sulpicio to
shoot at the latter. They were among those who laid siege to
Demetrio's house and left together with the others after finally
accomplishing their criminal deeds as agreed upon. Appellants
Bautista and Matchoca, are therefore also liable as co-principals
in Rafael's murder. Regarding motive, it was proved that both
were among those involved in the land conflict with Rafael
Cabizares and were among the respondents in the case before
the Agrarian Court
The aggravating circumstance of treachery, applicable
against appellant Sulpicio de la Cerna only, is offset by his

voluntary surrender after the incident.


This mitigating
circumstance however cannot benefit the remaining appellant
who did not voluntary surrender. For all the appellants,
therefore, the penalty for Rafael Cabizares' murder must be
imposed in the medium period. For the killing of Casiano
Cabizares appellant Sulpicio de la Cerna must be acquitted.
63. PEOPLE VS. DELA CRUZ
97 SCRA 385 (1980)
Nature: Automatic review of the decision of the Court of First
Instance of Basilan City.
Facts: Antonio Yu owned 200 hectares of rubber and coconut
land in Lantawan, Isabela, Basilan City. The victim, Yu Chi
Chong, is his brother. The accused Agapito dela Cruz was an
overseen of Antonio Yu for no less than 10 years.
Eleven were charged for kidnapping and slaying of Yu
Chi Chong but only Agapito dela Cruz, Jamas Jumaidi and
Oyong Asidin were apprehended. The rest have remained at
large.
On September 24, 1968, the city fiscal asked for the
discharged of Jamas Jumaidi and Oyong Asidin to be utilized as
state witness. The Trial Court granted the motion.
The evidence of the prosecution mainly rested on the
testimonies of the two discharged witnesses and that on
Mohammad Sagip who all pointed to Agapito as mastermind in
the kidnapping of Yu Chi Chong.
Mohammad Sagip testified that sometimes in October,
1967, the accused Agapito met with him, Alih Itum and a certain
Asmad at which he proposed to them the killing of Antonio Yu
and the kidnapping of the younger brother. Yu Chi Chong for a
ransom. Apparently, Asmad contacted some people in Jolo,
Sulu, for the purpose the accused herein among them.
The two discharged witness narrated what transpired
thereafter. In Isabela, the group waited in the ambush spot for
the truck that would carry sacks of copra and supposed to be the
two brothers but only Yu Chi Chong was in the truck because
Antonio Yu had to go Tairan on some other business Isabelo
Mancenido accompanied YU Chi Chong in the truck. When the
truck came to a half near the ambush spot, the ambushers
approached it and dragged Yu Chi Chong and Isabelo
Mancenido there from. Shortly after, the group released
Mancenido upon the latter's please of mercy. Upon reaching
Bancao Sapa they found that the tide was low, rendering
impossible for them to reach their boat. While waiting, Yu Chi
Chong, in an attempt to escape, struck Angih with a piece of
wood and tried to grab the gun of the latter but failed. Angih, in
anger, fired at Yu Chi Chong several times, killing him. They
dumped the body in the middle of the sea and it was never
recovered.
As a defense. Agapito claimed that on March 5, 1968
he reported for work in the land of Antonio Yu and that in the
evening he had dinner in the house of Alfonso Flores and slept
there that night. He strongly asserted that he never left that
house from 7:30 in the evening until 6:00 in the morning.
Issue: Whether or not the accused be held principally liable for
the death of Yu Chi Chong.
Held: Appellant is guilty as principal by inducement due to the
fact that he was the one who lays down the strategy of the crime,
he knew the route that the truck would take and approximate
time that it was to pass by. He even selected the ambush place.
He also presented the strongest temptation, a pecuniary gain in
the form ransom, which was the determining factor of the
commission of the crime by his co-accused. Without him, the
crime would not have been conceived, much less committed.
64. PEOPLE VS. MONTELEAGRE
161 SCRA 700 (1988)
Nature: Appeal from judgment of the Court of First Instance of
Cavite City.
Facts: At about 11:30 in the evening of March 11, 1983,
while Edmundo Abadilla was eating at the Meding's Restaurant
in Cavite City, he detected the smell of marijuana smoke coming
from the nearby table. He then went outside to report the matter

29

of Pfc. Renato Camantigue a policeman Camantigue joined


Abadilla in the restaurant. He approached the two and collard
both of them, saying "namamarijuana kayo, ano?"
While Camantigue was holding the two, Montealegre
with his right hand and Capalad with his left hand, Capalad
suddenly pulled out the knife tucked in his waist and started
stabbing Camantigue. Camantigue let loose Montealegre to draw
the gun from his holster but Montealegre thus released
restrained Camantigue's hand to prevent the latter in defending
himself Montealegre used both his hands for this purpose as
Capalad continued stabbing the victim.
Capalad fled into the dark alley, while the accused
escaped during the confusion Capalad was late found slumped in
an alley with a bullet wound in his chest. Neither Camantigue
nor Capalad survived, both expiring the next day.
Issue: Whether or not Montealegre be held liable for the death
of Pfc. Renato Camantigue
Held: The accused-appellant was correctly considered a coprincipal for having collaborated- with Capalad in the killing of
the police officer. The two acted in concert, with Capalad
actually stabbing Camantigue seven times and the accusedappellant holding on to the victim's hands to prevent him from
drawing his pistol and defending himself. While it is true that
the accused-appellant did not himself commit the act of
stabbing, he was nonetheless equally guilty thereof for having
prevented Camantigue from resisting the attack against him. The
accused-appellant was a principal by indispensable cooperation
under Article 17, par. 3, of the Revised Penal Code.
As correctly interpreted, the requisites of this provision are: "(1)
participating in the criminal resolution, that is, there is either
anterior conspiracy or unity of criminal purpose and intention
immediately before the commission of the crime charged; and
(2) cooperation in the commission of the offense by performing
another act without which it would not have been
accomplished."
65. PEOPLE VS. MANDOLADO
123 SCRA 133 91983)
66. PEOPLE VS. DOCTOLERO
193 SCRA 632 (1991)
Nature: Appeal from the decision of the then Court of First
Instance of Lingayen, Pangasinan, Br. 2, convicting Ludovico
Doctolero, Conrado Doctolero and Virgilio Doctolero of the
crime multiple murder and unspecified physical injuries.
Facts: At about 6:30 pm of Nov. 8, 1970, Marcial Sagun and his
wife, Maria Oviedo-Sagun and Lolita de Guzman-Oviedo were
on their way home to Barrio Binday. They came from the field
where they bundled their harvest. Upon reaching a crossing of
the road in bo. Binday they met the accused Ludovico Doctolero
who, without warning and without cause or reason, held the left
shoulder of Marcial Sagun with his left hand and struck her with
a bolo.
Paciencia Sagun-Diamoy testified that while she was cleaning
palay in the yard of her uncle, the deceased Marcelo Doctolero,
she saw the accused, Ludovico. Conrado and Virgilio throw
stones at the house of Marcial Sagun. While throwing stones,
Ludovico allegedly shouted for the men in the house to come
out. Paciencia Sagun-Diamoy went towards the house of
Marcial Sagun and saw the three accused, Ludovico, Conrado,
and virgilio, coming down from the house going towards her. At
about that time, Marcelo Doctolero, the half-brother of Antonio,
and the uncle of the three accused was going towards the house
of Marcial. The accused struck Marcelo several times with their
bolo.
Maria Oviedo-Sagun corroborated the testimony of Paciencia.
As she was about to go to their house to get their children, she
saw the three accused going up the house then she heard
Epifania, her adopted mother, shouting at her Enieng, your
children! then she saw the three accused coming down the
house.
The lower court held that Conrado and his brother
Viregilio participated as accomplices in the slaying of the
women and the infliction of injuries on the child.

Held: The decision of the trial court is modified regarding the


penalties rendered.
The Trial Court correctly ruled that Conrado
participated as an accomplice. It is impossible that the appellant
did not know or were not aware that his brother, Ludovico, was
brutally killing the two women and wounding the child. It is,
therefore, reasonable to believe that Conrado merely stood by as
his brother was murdering deceased women, ready to lend
assistance. Indeed, there is no question that his presence gave his
brother the encouragement and the reliance to proceed as he did
proceed in committing the heinous crimes.
Whatever doubt the court a quo entertained in the
criminal responsibility of Conrado did not refer to whether or
not he was liable but only with regard to the extent of his
participation. There being ample evidence of their criminal
participation but a doubt exists on the nature of their liability
and so the court should favor the milder form or responsibility,
which is that of being a mere accomplice.
*note: Virgilio died while case was pending appeal
67. PEOPLE VS. TALINGDAN
84 SCRA 19 (1978)
NATURE: APPEAL from the judgment of the Court of First
Instance of Abra.
FACTS: Prior to the violent death of Bernardo Bagabag on the
night of June 24, 1967, he and appellant Teresa Domogma and
their children, lived together in their house at Sobosob,
Salapadan. For sometime, however, their relationship had been
strained and beset with troubles, for Teresa had deserted their
family home a couple of tunes and each time Bernardo took time
out to look for her. Bernardo had gotten wind that illicit
relationship was going on between Talingdan and Teresa, and
during a quarrel between him and Teresa, he directly charged
the latter that should she get pregnant, the child would not be
his. between 10:00 and 11:00 o'clock the following Friday
morning, Bernardo's daughter, Corazon, who was then in a creek
to wash clothes saw her mother, Teresa, meeting with Talingdan
and their co-appellants Magellan Tobias, Augusto Berras and
Pedro Bides in a small hut owned by Bernardo. Teresa
Domogma noticed the presence of her daughter; she shoved her
away saying "You tell your father that we will kill him".
Saturday, June 24, 1967, while the same 12-year old daughter of
Bernardo was cooking food for supper in the kitchen of their
house, she saw her mother go down the house through the stairs
and go to the yard where she again met with the other
appellants. She noted that the appellants had long guns at the
time. His time, she informed her father about the presence of
persons downstairs, but Bernardo paid no head to what she said.
At that moment, he was suddenly fired upon from below the
stairs of the "batalan". The four accused then climbed the stairs
of the "batalan" carrying their long guns and seeing that
Bernardo was still alive, Talingdan and Tobias fired at him
again.
Issue: Whether or not Teresita is guilty for being an accessory
to the offense committed by her co-accused.
HELD: True it is that the proof of her direct participation in the
conspiracy is not beyond reasonable doubt, for which reason,
she cannot have the same liability as her co-appellants. Indeed,
she had no hand at all in the actual shooting of her husband.
Neither is it clear that she helped directly in the planning and
preparation thereof, albeit the SC was convinced that she knew
it was going to be done and did not object.
But this is not saying that she is entirely free from
criminal liability. There is in the record morally convincing
proof that she is at the very least an accessory to the offense
committed by her co-accused. She was inside the room when her
husband was shot. As she came out after the shooting, she
inquired from Corazon if she was able to recognize the
assailants of her father. When Corazon identified appellants
Talingdan, Tobias, Berras and Bides as the culprits, Teresa did
not only enjoin her daughter not to reveal what she knew to
anyone, she went to the extent of warning her, "Don't tell it to
anyone. I will kill you if you tell this to somebody." Later, when

30

the peace officers who repaired to their house to investigate


what happened, instead of helping them with the information
given to her by Corazon, she claimed she had no suspects in
mind. In other words, whereas, before the actual shooting of her
husband, she was more or less passive in her attitude regarding
her coappellants' conspiracy, known to her, to do away with
him, after Bernardo was killed, she became active in her
cooperation with them. These subsequent acts of her constitute
"Concealing or assisting in the escape of the principal in the
crime" which makes her liable as an accessory after the fact
under paragraph 3 of Article 19 of the Revised Penal Code.

68. PEOPLE VS. FERRER


48 SCRA 382 (1972)
NATURE: Special civil action in the Supreme Court to review
on certiorari the constitutionality of the Anti-Subversion Act
(RA 1700)
FACTS: On March 5, 1970, a criminal complaint for violation
of section 4 of RA 1700 was filed against Felicaino Co in the
CFI of Tarlac. Co moved to quash on the ground that the said
RA was a bill of attainder. Meanwhile, Nilo Tagay, et al. had
been charged for the violation of RA 1700. On July 21, 1970, he
moved to quash the charges on the grounds that the said RA was
a bill of attainder, and that it was vague and embraced more than
one subject not expressed in its title. Resolving the constitutional
issues raised, the trial court declared the stature void on the
grounds that it is vague and overboard, and dismissed the
informations against the two accused.
Issue: W o N RA 1700 is a bill of attainder.
Held: The questioned resolution of the lower court is set aside,
and the two cases are remanded to the court for trial on the
merits.
It is not a bill of attainder. A bill of attainder is a
legislative act which inflicts punishment without trial. Its
essence is the substitution of a legislative for a judicial
determination of guilt. When the act is viewed in its actual
operation, it will be seen that it does not specify the Communist
Party of the Philippines or the members thereof for the purpose
of punishment. The term Communist Party of the Philippines
is used solely for definitional purposes. Its focus is not on the
individuals but on conduct.
As to the membership, the act does not punish nominal
membership but membership that is knowing or active, with
specific intent to further the illegal objectives of the Party. It is
only when a statute applies either to named individuals or to
easily ascertainable members of a group in such a way as to
inflict punishment on them without judicial trial does it become
a bill of attainder.
A bill of attainder is similar to an ex post facto law.
RA 1700 is not such because it punishes only acts committed
after the approval of such act. Only those who knowingly,
willfully and by over acts affiliate themselves with, become or
remain members after June 20, 1957 are punished.
69. GUMABON, et al. VS. DIRECTOR OF BUREAU OF
PRISONS (PROSPECTIVITY)
Nature: Original petition in the Supreme Court for Habeas
Corpus
Facts: The CFI of Manila sentenced Mario Gumabon on May 5,
1953 to reclusion perpetua for the complex crime of rebellion
with multiple murder, robbery, arson and kidnapping. Gumabon,
together with Agapito, Palmares, Padua and Bagobagol are
invoking habeas corpus, citing the case of People vs. Hernandez
that no such complex crime of rebellion exist under Art. 13 of
RPC. According to the case of Hernandez, rebellion cannot be
complexed with other common crimes since such common
crimes assume political complexion of the main crime of which
they are mere ingredients and consequently cannot be punished
separately from the principal offense. They cite the ruling of
Hernandez to be retroactively applied to them by virtue of Art.

22 of RPC. (People vs. Lava- the leaders of the rebellion have


been freed while their followers, the petitioners, suffer life
imprisonment)
Issue: W o N the ruling in the Hernandez case should retroact to
the case at bar.
Held: Petition for habeas corpus granted and the petitioners are
set free.
The actual case of the petitioners is that at the time of their
conviction, it was believed that the crime committed by them
was punishable by life imprisonment, but the court has
subsequently judicially determined it not be so and that the
maximum penalty imposable is prision mayor or 12 years.
Petitioners-convicts are entitled to the benefits of this later
judicial declaration, just as if a statutory amendment had been
enacted. The writ prayed for should be issued since it is the only
means of giving retroactive effect to a penal provision favorable
to the accused where the trial judge has lost jurisdiction over the
case. Furthermore art. 22 of the RPC extends its benefits even to
convicts serving sentence, and the only legal remedy open to
them is the writ of habeas corpus.
70. PEOPLE VS. GATWARD
267 SCRA 785 (1997)
Nature: Appeal from a decision of the Regional Trial Court of
Pasay City, Branch.
Facts: Gatward was charged with violating Section 4 of the
Dangerous Drugs Act of 1972 for having transported hereon
contained in separate carton envelopes with a total weight of
5237 70 grams which is legally considered as a prohibited drug
on or about the 31st day of August 1994, in the vicinity of the
Ninoy Aquino International Airport, Pasay City.
Meanwhile, U Aung Win was indicted for
transgressing Section 3 of the Dangerous Drug Act 1972 for
having imported and brought in the Philippines 5570 80 grams
of herein which is legally considered as a prohibited drug.
Gatward pleaded not guilty of the charge when
arranged while U Aung Win pleaded guilty.
Both were convicted of the offense charged. The
penalty to de imposed under the Dangerous Drug Act shall range
from reclusion perpetua to death. In imposing the proper
penalty, the trial court declared that the penalty of "reclusion
perpetua to death" shall have the following periods Death as the
maximum thirty (30) years and one (1) day to forty (40) years as
the medium and twenty (20) years and one (1) day to thirty (30)
years as the maximum it regarded reclusion perpetua as an
indivisible penalty.
Thus Gatward was sentenced to suffer the penalty of
imprisonment for thirty five (35) years of reclusion perpetua and
to pay a fine of Five Million Pesos (5,000,000.00) in view of the
presence of one (1) mutilating circumstance of voluntary plea of
guilty, without any aggravating circumstance.
Issue: Whether or not reclusion perpetua a divisible penalty.
Held: The trial court had by considering reclusion perpetua as a
divisible penalty imposed an unauthorized penalty on both
accused which would remain uncorrected if the appeal had been
allowed to be withdrawn. In fact it would stamp a mlul
obstantient on a penalty that in law does not exist and which
error initially committed by the Court in another case on which
the trial court held had already been set aright by these.
The penalty of reclusion perpetua is now accorded a
"defined duration" ranging from twenty (20) years and one (1)
day to forty (40) years, through the amendment introduced by
RA 7659 to Article 27 of the Revised Penal Code. The Court
held that in spite of the amendment putting the duration of
reclusion perpetua at 20 years and 1 day to 40 years; it should
remain as an indivisible penalty since there was never any intent
on the part of Congress to reclassify it into a division penalty.
The judgment of the court a quo, specifically with
regard to the penalty imposed on accused-appellant Nigel
Richard Gatward and that of accused U Aung Win was
MODIFIED in the sense that both accused were sentenced to

31

serve the penalty of reclusion perpetua in its entire duration and


full extent.
71. PEOPLE VS. FORMIGONES
87 PHIL 658 (1950)
Nature: An appeal from the decision of the CFI Camarines Sur
finding the appellant guilty of parricide, sentencing him to
reclusion perpetua.
Facts: On Nov. 1946, defendant Abelardo Formigones lived on
his farm in Bahao, Libmanan, Sipocot, Camarines Sur w/ wife
Julia Agricola and 5 children. Sometime later they went to
Binahian, Sipocot to seek employment as harvesters of palay,
lived w/ his half-brother, Zacarias Formigones
On Dec. 28, 1946, while Julia was sitting on top of the stairs of
the house, w/o any previous quarrel or provocation, Abelardo
stabbed his wife w/ a bolo blade punctured Julia's back and right
lung, caused a severe hemorrhage resulting in her death. Julia
toppled down the stairs. Abelardo followed and carried and laid
her on the floor of the living room and he had down beside her.
Their eldest daughter Irene, who witnesses the incident, shouted
for help, people came in response. The accused suspected them
that they are maintaining illicit relations with his half-brother.
Held: Appellant is guilty of parricide. Judgment of the lower
court affirmed w/ modification, appellant will be credited with
of any preventive imprisonment he has undergone.
The SC convinced that the appellant is not imbecile.
During his marriage of 16 years, he had not done anything to
warrant an opinion that he was an imbecile. Killing his wife
whom he suspect of being unfaithful to him in the belief that he
was vindicating his honor could not be regarded as an imbecile.
His feeling of jealousy may or may not be true bee in is
statement, observed that the his half brother who was living in
his grandmother not only frequented his house after they have
transferred, but also sleep there during the night this aroused
or partly confirmed Abelardo's suspicions. It was an act of
remorse: lying beside his wife for hours after he killing her, he
made no effect to flee and compel the police to hunt him down
and arrest him, he readily admitted that he killed his wife in his
statement
72. LACANILAO VS. CA
62 SCRA 563 (1988)
Nature: A petition which calls for our exercise of the power of
Judicial Review- question of Law.
Facts: The court Of First Instance of Manila finding the
petitioner guilty of homicide for the death of one Ceferino
Erese, and was sentenced to an indeterminate penalty of six
years and one day of prision mayor as minimum to fourteen
years and one day of reclusion temporal as the maximum.
The petitioner appealed to the CA which modified the decision
of the CFI changing the maximum penalty from fourteen years
and one day to twelve years and one day respectively.
The court of appeals that Bernardo Lacanilao acted in the
performance of his duty but that the shooting of the victim was
not the necessary consequence of the due performance thereof,
therefore, crediting to him the mitigating circumstance of
incomplete fulfillment of duty. In the words of the respondent
court saying while the appellant should be commended for
responding to the call of duty when he tried to stop the victim
and the latters companions from their drunken and disorderly
conduct, nevertheless he cannot be exonerated from overdoing
his fulfillment of duty to the extent of admittedly shooting and
thereby killing said victim. Thus, the respondent court lowered
the penalty merely by one period applying Art. 64 par. 2 of the
RPC.
The two conditions that must be met to justify fulfillment of
duty are the ff: 1) that the accused acted in the performance of a
duty or in the lawful exercise of a right or office, and 2) that the
injury or offense committed be necessary consequence of the
due performance of such duty or the lawful exercise of such
right of office. In the case at bar, only the first condition was
met.

Issue: W o N art. 69 of the RPC is applicable to the case at bar.


Held: Yes. Art. 69 is applicable, for the requirement of having a
majority of the conditions be present to lower the penalty by one
or two degrees is immaterial because there are only two
conditions in order that the circumstance in no. 5 of art. 11 may
be taken into account. The petition is granted in so far as it seeks
to the modification of the penalty pursuant to art. 69 of the RPC
and the ruling in Oanis. The petitioner is hereby sentenced to an
indeterminate penalty of from two years, four months and one
day of prision correctional to eight years and one day of prision
mayor.

73. PEOPLE VS. GERONIMO


100 PHIL 99 (1956)
Nature: Appeal from a judgment of the Court of First Instance
of Camarines Sur
Facts: In information filed by the provincial Fiscal in the CFI of
Camarines Sur, appellant Federico Geronimo and many others
were charged with the complex crime of rebellion with murder,
robberies and kidnapping.
Accused Federico Geronimo first entered a plea of not
guilty to the information. When the case was called for trial, he
asked the information of the court to substitute his original plea
with one of guilty, and was allowed to change his plea. On the
basis of the plea of guilt, the fiscal recommended that the
penalty of life imprisonment be imposed upon the accused, his
voluntary plea of guilt being considered as a mitigating
circumstance. Geronimos counsel argued that the penalty
imposable upon the accused was only prision mayor, for the
reason that there is no such complex crime as rebellion with
murder, robberies and kidnapping, because the crimes of
murder, robbery and kidnapping being the natural consequences
of the crime of rebellion, the crime charged against the accused
should be considered only as simple rebellion.
The trial court rendered judgment finding the accused
guilty of the complex crime of rebellion with murder, robbery
and kidnapping and giving him the benefit of the mitigating
circumstance of voluntary plea of guilt, sentenced him to suffer
the penalty of Reclusion Perpetua.
Issue: Whether the crime committed is the complex crime of
rebellion with murder, robbery and kidnapping, or simple
rebellion.
Ruling: The decision appealed from was modified and the
accused convicted for the simple crime of rebellion and
considering the mitigating effect of his plea of guilt, accusedappellant Federico Geronimo was sentenced to suffer 8 years of
prision mayor.
The acts when committed as a means to or in
furtherance of the subversive ends become absorbed in the crime
of rebellion, and cannot be regarded or penalized as distinct
crimes in themselves. In law they are part and parcel of the
rebellion itself, and cannot be considered as giving rise to a
separate crime that would constitute a complex one with that of
rebellion.
If the killing, robbing and kidnapping were done for
private purposes or profit, without any political motivation, the
crime would be separately punishable and would not be
absorbed by the rebellion. But even then, the individual misdeed
could not be taken with the rebellion to constitute a complex
crime, for the constitutive acts and intent would be unrelated to
each other; and the individual crime would be a means necessary
for committing the rebellion as it would not be done in
preparation or in furtherance of the latter.
74. PONCE ENRILE VS. SALAZAR
86 SCRA 217 (1990)
Nature: An appeal from a judgment of the Court of First
Instance of Camarines Sur.

32

Facts: In the afternoon of February 27, 1990, Senate Minority


Floor Leader Juan Ponce Enrile was arrested by law
enforcement officers led by Director Alfredo Lim of the
National Bureau of Investigation on the strength of a warrant
issued by Hon. Jaime Salazar of the Regional Trial Court of
Quezon City Branch 103, in Criminal Case No. 9010941. The
warrant had issued on an information signed and earlier that day
filed by a panel of prosecutors composed of Senior State
Prosecutor Aurelio C. Trampe, State Prosecutor Ferdinand R.
Abesamis and Assistant City Prosecutor Eulogio Mananquil, Jr.,
charging Senator Enrile, the spouses Rebecco and Erlinda
Panlilio, and Gregorio Honasan with the crime of rebellion with
murder and multiple frustrated murder allegedly committed
during the period of the failed coup attempt from November 29
to December 10, 1990. Senator Enrile was taken to and held
overnight at the NBI headquarters on Taft Avenue, Manila,
without bail, none having been recommended in the information
and none fixed in the arrest warrant. The following morning,
February 28, 1990, he was brought to Camp Tomas Karingal in
Quezon City where he was given over to the custody of the
Superintendent of the Northern Police District, Brig. Gen.
Edgardo Dula Torres.
On the same date of February 28, 1990, Senator
Enrile, through counsel, filed the petition for habeas corpus
herein (which was followed by a supplemental petition filed on
March 2, 1990), alleging that he was deprived of his
constitutional rights.
Issue: Whether or not there is a complex crime of rebellion
Held: The petitioners' case does not fall within the Hernandez
ruling because-and this is putting it very simply-the information
in Hernandez charged murders and other common crimes
committed as a necessary means for the commission of
rebellion, whereas the information against Sen. Enrile et al.
charged murder and frustrated murder committed on the
occasion, but not in furtherance, of rebellion. Stated otherwise,
the Solicitor General would distinguish between the complex
crime ("delito complejo") arising from an offense being a
necessary means for committing another, which is referred to in
the second clause of Article 48, Revised Penal Code, and is the
subject of the Hernandez ruling, and the compound crime
("delito compuesto") arising from a single act constituting two
or more grave or less grave offenses referred to in the first
clause of the same paragraph, with which Hernandez was not
concerned and to which, therefore, it should not apply.
75. PEOPLE VS. ESCOBAR
157 SCRA 541 (1988)
Nature: APPEAL from the decision of the Regional Trial Court
of Quezon City, Br. 97. Leviste, J.
Facts: One of the alleged co-conspirator Amadeo Abuyen alias
Roberto Alorte, was formerly a co-security guard of appellant
Juan Escober at the Bee Seng Electrical
Supply, Inc., a family corporation owned by the couple Vicente
Chua and Lina Chua. About 4 months prior to the incident,
Abuyen was relieved by Domingo Rocero for being always
absent and found sleeping while on duty. "At the time of the
incident on December 3, 1982, Rocero's tour of duty was from
7:00 in the morning to 7:00 in the evening. He left his post at
about 7:30 P.M. that evening after he was relieved by appellant
Juan Escober. On his way home, he passed by Barangay
Balingasa in Balintawak, where he saw Amadeo Abuyen in the
store of Colonel Samson drinking beer with three companions,
one of whom he later identified as the appellant Macario
Punzalan, Jr. "After Rocero had left his point, Vicente Chua
went to his office at the Bee Seng Electrical Supply as he
usually does after office hours, accompanied by his 13-year old
son Irvin and 6-year old daughter Tiffany. On their way, he saw
appellant Escober at his post. At the office, the two children
watched a television program, as their father proceeded to the
bathroom to take a bath "Meanwhile, Abuyen and his three
companions rode a tricycle and proceeded to the Bee Seng
Electrical Supply. Upon alighting thereat, Abuyen knocked at
the little door of the gate. Appellant Escober peeped thru the
hole and opened the door. Then after Abuyen had talked with

Escober, the former asked Punzalan to wait outside, while he


(Abuyen) and his two other companions went inside
"At this juncture, the victims 'mother, Mrs. Lina B.
Chua, left their residence to join her husband and two children.
On her way, she noticed that the pedestrian gate was wide open
with the appellant Punzalan standing there. She shouted why the
gate was opened, but nobody answered. Suddenly, she heard of
shot coming from the direction of the garage; and when she
looked thereat, she saw Abuyen and the appellant Escober
walking towards the gate. So, she rushed back inside the house
to contact her husband through the intercom. But since the
intercom was out of order, she hurriedly went outside and met
appellant Escober who volunteered the information that he was
not hit' 'Upon the other hand, Vicente Chua was inside the
bathroom, when he heard the gunshot. He hurriedly went out
and saw her son Irvin lying on the sofa while Tiffany was lying
on the floor, both mortally wounded. Beside her daughter, he
saw a scissor blade [full of blood. He also observed that
everything was scattered in his office, with all his drawers
opened. Later, he found out that the P5, 000.00 cash he kept in
one of the drawers was lost
Juan Escober, together with four unidentified persons designated
as John Doe, Peter Doe, Richard Doe and Juan Doe, were
charged with the crime of Robbery with Homicide before the
Regional Trial Court of Quezon City in Information dated
December 9, 1982. He entered a plea of "Not Guilty" with the
assistance of counsel Atty. Hipolito de Peralta upon arraignment
on March 2, 1983. Punzalan was later included in the charges.
Issue: Whether or not the accused participated in a complex
crime.
Held: Decision reversed and set aside. The accused is acquitted.
The act of opening a gate upon hearing a knock is by
itself an innocent gesture. One who imputes an evil motive or
purpose thereto must prove his allegations convincingly. In the
case at bar, even if the version of Macario Punzalan, Jr, that
Escober opened the gate at the knock of the alleged mastermind
Amadeo Abuyen/Roberto Alorte were to be believed, the same
would not constitute sufficient and convincing proof that
Escober had knowledge of the nefarious plan. The worse that
could be attributed to him is lack of better judgment or laxity in
the performance of his duties as a security guard in having failed
to exercise the minimum precaution dictated by his occupation
to exclude from the premises being guarded persons who have
not demonstrated any legitimate reason for getting in.
76. MEJORADA VS. SANDIGANBAYAN
51 SCRA 339 (1987)
Nature: A petition for certiorari seeks to reverse the May 23,
1979 decision of the Sandiganbayan finding the accused Arturo
A. Mejorada in Criminal Cases Nos. 002-009 guilty beyond
reasonable doubt of violating Section 3(E) of Republic Act No.
3019, otherwise known as the Anti-Graft and Corrupt Practices
Act
Facts: Arturo A. Mejorada was a public officer who was first
employed as a temporary skilled laborer in the Bureau of Public
Works on March 16, 1947, and then as right-of-way agent in the
Office of the Highway District Engineer, Pasig, Metro Manila,
from February, 1974 up to December 31, 1978. As a right-ofway agent, his main duty was to negotiate with property owners
affected by highway constructions or improvements for the
purpose of compensating them for the damages incurred by said
owners.
Among those whose lots and improvements were
affected by the widening of the proposed Pasig-Sta. CruzCalamba Road. 2nd IBRD Project, at Binangonan, Rizal was
Isagani de Leon, Isaac Carlos, Napoleon Maybituin, Dominga
Villaroza, Florentino de la Cruz, Cipriano Aran, Celestina S.
Mallari and Rodolfo Rivera, all residents of Mambog,
Binangonan, Rizal.
Sometime in October or November 1977, petitioner
contacted the forenamed persons and informed them that he
could work out their claims for payment of the values of their
lots and/or improvements affected by the widening of said
highway. In the process, Mejorada required the claimants to sign
blank copies of the "Sworn Statement on the Correct and Fair

33

Market Value of Real Properties" and "Agreement to Demolish,


Remove and Reconstruct improvements" pertinent to their
claims. The claimants complied without bothering to find out
what the documents were all about as they were only interested
in the payment of damages.
In said "Sworn Statements" and "Agreements to
Demolish", the value of the respective properties of the
claimants was made to appear very much higher than the actual
value claimed by them. Likewise, the said "Agreements to
Demolish" reflected the value of the improvements as per
assessor" which on the average was only P2, 000.00 lower than
the value declared by the owners in their sworn statements. The
value as per assessor was, in turn, supported by the Declarations
of Real Property in the names of the claimants containing an
assessed value exactly the same as that stated in the Agreements
to Demolish "as per assessor", except the claims of De la Cruz
and Aran where there is only a difference of P400.00 and
P200.00, respectively. It turned out, however, that said
Declarations of Property are not really intended for the
claimants as they were registered in the names of other persons,
thus showing that they were all falsified.
A few months after processing the claims, accused
accompanied the claimants to the Office of the Highway District
Engineer at the provincial capitol of Pasig, Metro Manila, to
receive payments and personally assisted the claimants in
signing the vouchers and encashing the checks by certifying as
to their Identities and guaranteeing payment.
Right after the claimants had received the proceeds of
their checks, accused accompanied them to his car which was
parked nearby where they were divested of the amounts paid to
them leaving only the sum of P1, 000.00 to each, except Isaac
Carlos to whom P5, 000.00 was left, explaining to them that
there were many who would share in said amounts. All the
claimants were helpless to complaint because they were afraid
of the accused and his armed companion.
Issue:
I. Whether or not the essential elements constituting the offense
penalized by section 3(e) of Republic Act No. 3019, otherwise
known as the Anti-Graft and Corrupt Practices Act have been
clearly and convincingly proven by the prosecution;
II. Whether or not the Sandiganbayan is a court of competent
jurisdiction duly constituted in accordance with Pres. Dec. No.
1606;
III. Whether or not the penalty imposed upon the petitioner is
excessive and contrary to the three-fold rule as provided for by
Article 70 of the Revised Penal Code;
IV. Whether or not there is a variance between the offense
charged in the information and the offense proved;
V. Whether or not the conclusion drawn from the record of the
Sandiganbayan in arriving at a verdict of conviction of petitioner
is correct is a question of law which this Honorable Court is
authorized to pass upon.
Held: The argument is devoid of merit. The Sandiganbayan
established the fact that the petitioner took advantage of his
position as a right-of-way-agent by making the claimants sign
the aforementioned agreements to demolish and sworn
statements which contained falsified declarations of the value of
the improvements and lots. There was evident bad faith on the
part of the petitioner when he inflated the values of the true
claims and when he divested the claimants of a large share of the
amounts due them.
We also dispose of the fourth issue which relates to
the allegation that petitioner cannot be convicted for a violation
of the Anti-Graft Law because the evidence adduced by the
prosecution is not the violation of Section 3 (e) but the crime of
robbery. Contrary to the petitioner averment. We find no
variance between the offense charged in the information and the
offense proved. The prosecution was able to establish through
the corroborating testimonies of the witnesses presented how
through evident bad faith, petitioner caused damage to the
claimants and the Government. The manner by which the
petitioner divested the private parties of the compensation they
received was part of' the scheme which commenced when the
petitioner approached the claimants and informed them that he
could work out their claims for payment of the values of their
lots and/or improvements affected by the widening of the Pasig-

Sta. Cruz-Calamba Road. The evidence presented by the


prosecution clearly establishes a violation of Section 3(e).
The judgment convicting petitioner was a unanimous
Decision of the First Division duly constituted. It thus met the
requirement for the pronouncement of a judgment as required by
Section 5 of P.D. 1606 supra.
Petitioner is mistaken in his application of the three-fold rule as
set forth in Article 70 of the Revised Penal Code. This article is
to be taken into account not in the imposition of the penalty but
in connection with the service of the sentence imposed (People
v. Escares, 102 Phil. 677 [1957]). Article 70 speaks of "service"
of sentence, "duration" of penalty and penalty "to be inflicted".
Nowhere in the article is anything mentioned about the
"imposition of penalty". It merely provides that the prisoner
cannot be made to serve more than three times the most severe
of these penalties the maximum of which is 40 yrs.
77. BALA VS. MARTINEZ
181 SCRA 459 (1990)
Nature: This is petition for certiorari and prohibition with
preliminary injunction and/or temporary restraining order seeks
reversal of the order dated April 2, 1984 of the CFI of the RTC
of Manila Branch XX
Facts: The petitioner had been indicted for removing and
substituting the picture of Maria Eloisa Criss Drazen which had
been attached to her United States of America passport, with
that of Florencia Notarte, in effect falsifying a genuine public or
official document. On January 3, 1978, the trial court adjudged
petitioner Manuel Bata in Criminal Case No. 24443, guilty of
the crime of falsification of a public document. The petitioner
seasonably appealed, but the Court of Appeals, on April 9, 1980,
affirmed in toto the lower court's decision. The petitioner
applied for and was granted probation by the respondent judge
in his order dated August 11, 1982. The petitioner was then
placed under probation for a period of one (1) year, subject to
the terms and conditions enumerated therein. By the terms of
the petitioner's probation, it should have expired on August 10,
1983; one year after the order granting the same was issued.
But, the order of final discharge could not be issued because the
respondent probation officer had not yet submitted his final
report on the conduct of his charge. On December 8, 1983, the
respondent People of the Philippines, through Assistant City
Fiscal Jose D. Cajucom of Manila, filed a motion to revoke the
probation of the petitioner before Branch XX of the Regional
Trial court (RTC) of Manila, presided over by the respondent
judge. The motion alleged that the petitioner had violated the
terms and conditions of his probation. On January 4, 1984, the
petitioner filed his opposition to the motion on the ground that
he was no longer under probation, his probation period having
terminated on August 10, 1983, as previously adverted to. As
such, no valid reason existed to revoke the same, he contended.
As if to confirm the Manila Assistant City Fiscals motion to
revoke the petitioner's probation, the respondent probation, the
respondent probation officer filed on January 5, 1984. the same
motion, however, became the subject of a "Manifestation," dated
January 10, 1984, which stated that the probation officer was not
pursuing the motion to terminate dated January 6, 1984, instead,
he was submitting a supplemental report which recommended
the revocation of probation "in the light of new facts,
information, and evidences," As stated at the outset, the
respondent judge denied the motion to dismiss for lack of merit.
Held: The Court finds no merit in the petition. Probation is
revocable before the final discharge of the probationer by the
court, contrary to the petitioner's submission. It is worthy to
note, that what was actually resolved and denied was the motion
to dismiss and/or strike out the motion to revoke probation,
which disposed of only the issue of the petitioner's transfer of
residence. The motion did not touch on the issue of the
timeliness to revoke probation. The respondent judge has not
yet heard and received evidence, much less acted on the matter.
Rule of Law:
PD 968 is clear on this score:
See, 16, Termination of Probation After the period
of probation and upon consideration of the report and

34

recommendation of the probation officer, the court may order


the final discharge of the probationer upon findings that he had
fulfilled the terms and conditions of his probation and thereupon
the case in deemed terminated.
Thus, the expiration of the probation period along
does not automatically terminate probation. Nowhere is the
ipso facto termination of probation found in the previsions of
the probation law. Probation is not coterminous with its
period. There must first be issued by the court of an order of
an order of final discharge based on the report and
recommendation of the probation officer. Only from such
issuance can the case of the probationer be deemed terminated.
78. SALGADO VS. CA
189 SCRA 304 (1990)
Nature: This petition for review on certiorari seeks to set aside
the decision of the Court of Appeals in CA-G.R. SP No. 15493
entitled, "Agustin Salgado v. Hon. Antonio P. Solano, et. Al,"
who affirmed the Order dated December 22, 1987 of the
Regional Trial Court of Quezon City (Branch 86) sustaining its
previous order dated November 18, 1987 directing the issuance
of a writ execution to enforce the civil liability of herein
petitioner in Criminal Case No. 0-33798.
Facts: Petitioner was charged with the crime of serious physical
injuries in Criminal Case No. 0-33798 entitled. "People of the
Philippines v. Agustin Salgado," before the Regional Trial Court
of Quezon City (Branch 86). After trial, judgment was rendered
on October 16, 1987 finding him guilty beyond reasonable doubt
of the crime charged. On October 17, 1986, petitioner filed an
application was granted in an Order dated April 15, 1987. For
the months of May, June, July, August, September and October,
1987, petitioner complied with the above condition by paying in
checks the said sum of P2, 000.00 monthly, through the City
Probation Officer, Perla Diaz Alonzo. Private respondent
Francisco Lukban, Jr. voluntarily accepted the checks and
subsequently encashed them. On September 19, 1987, private
respondents Francisco Lukban, Jr. filed a motion for the
issuance of a write of execution for the enforcement of the civil
liability adjudged in his favor in the criminal case. The motion
was opposed by the petitioner. On November 18, 1987, the trial
court issued an order granting the motion for issuance of a write
of execution. A motion for reconsideration was filed by
petitioner but it was denied on December 22, 1987. After the
denial of his motion for reconsideration, the petitioner filed
directly with this Court a petition for review of the trial court's
order granting the motion for issuance of a writ of execution.
We referred the petition to the Court of Appeals in the resolution
dated April 13, 1988. On March 16, 1989 the petitioner went to
this Court via a petition for review which was filed on
September 26, 1989.
Issue: Whether or not the probation extinguishes civil liability.
Whether or not the trial court may impose as a condition of
probation the manner in which a probationer may settle his civil
liability against the offended party during the period of
probation.
Held: The decision of respondent Court of Appeals affirming
the order of the trial court granting the motion for the issuance
of a writ of execution as well as the resolution dated August 3,
1989 of the same court are hereby REVERSED and SET
ASIDE.
Probation affects only the criminal aspect of the case.
Interpreting the phrase within the context of that case, it means
that although the execution of sentence is suspended by the grant
of probation, it does not follow that the civil liability of the
offender, if any, is extinguished.
The conditions which trial courts may impose on a
probationer may be classified into general or mandatory and
special or discretionary. The mandatory conditions, enumerated
in Section 10 of the Probation Law, require that probationer
should a) present himself to the probation officer designated to
undertake his supervision at such place as may be specified in
the order within 72 hours from receipt of said order, and b)
report to the probation officer at least once a month at such time
and place as specified by said officer. Special or discretionary
conditions are those additional conditions, listed in the same

Section 10 of the Probation Law, which the courts may


additionally impose on the probationer towards his correction
and rehabilitation outside of prison. The enumeration, however,
is not inclusive. Probation statutes are liberal in character and
enable courts to designate practically any term it chooses as long
as the probationer's constitutional rights are not jeopardized.
There are innumerable conditions, which may be relevant to the
rehabilitation of the probationer when viewed in their specific
individual context. It should, however, be borne in mind that the
special or discretionary conditions of probation should be
realistic, purposive and geared to help the probationer develop
into a law-abiding and self-respecting individual, Conditions
should be interpreted with flexibility in their application, and
each case should be judged on its own merits-on the basis of the
problems, needs and capacity of the probationer.
*This is an exception because there was a CLEAR condition in
the probation, therefore, when the decision of the SC came out,
the period had already lapsed.
79. MONSANTO VS. FACTORAN
170 SCRA 190 (1989)
Facts: March 25, 1983, the Sandiganbayan convicted petitioner
Salvacion A. Monsanto (then assistant treasurer of Calbayog
City) and three other accused, of the complex crime of estafa
thru falsification of public documents and sentenced them t
imprisonment of four (4) years, two (2) months and one (1) day
of prison correctional as minimum, to ten (10) years and one (1)
day of prison mayor as maximum, and to pay a fine of P3, 500.
They were further ordered to jointly and severally indemnify the
government in the sum of P4, 892.50 representing the balance of
the amount defrauded and to pay the costs proportionately
Petitioner Monsanto appealed her conviction to this Court which
subsequently affirmed the same. She then filed a motion for
reconsideration but while said motion was pending, she was
extended on December 17, 1984 by then President Marcos
absolute pardon which she accepted on December 21, 1984. By
reason of said pardon petitioner wrote the Calbayog City
treasurer requesting that she be restored to her former post as
assistant city treasurer since the same was still vacant.
Issue: Whether or not a public officer, who has been granted an
absolute pardon by the Chief Executive, is entitled to
reinstatement to her former position without need of a new
appointment.
Held:
The absolute disqualification or ineligibility from
public office forms part of the punishment prescribed by the
Revised Penal Code for estafa thru falsification of public
documents. It is clear from authorities referred to that when her
guilt and punishment were expunged by her pardon; this
particular disability was likewise removed.
Henceforth,
petitioner may apply for reappointment to the office, which was
forfeited by reason of her conviction. And in considering her
qualifications and suitability for the public post, the facts
constituting her offense must be and should be evaluated and
taken the account with public funds. Stated differently, the
pardon to petitioner has resulted in removing her disqualification
from holding public employment but it cannot go beyond that, to
regain her former pose as assistant city treasurer, she must reapply and undergo the usual procedure required for a new
appointment.
Ruling: It is well too remember that petitioner had been
convicted of the complex crime of estafa thru falsification of
public documents and sentenced to imprisonment of four years,
two months and one day of prision correccional as minimum, to
ten years and one day of prision mayor as maximum. The
penalty of prision mayor carries the accessory penalties of
temporary absolute disqualification and perpetual special
disqualification from the right of suffrage, enforceable during
the term of the principal penalty, the accessory penalties remain
unless the same have been expressly remitted by the pardon. The
penalty of prision correccional carried, as one of its accessory
penalties, suspension from public office.
The propositions earlier advanced by petitioner reveal
her inadequate understanding of the nature of pardon and its
legal consequences. This is not totally unexpected considering

35

that the authorities on the subject have not been wholly


consistent particularly in describing the effects of pardon.
A pardon looks to the future, it is not retrospective. It
makes no amends for the past, it affords no relief for what has
been suffered by the offender, it does not impose upon the
government any obligation to make reparation for what has been
suffered since the offense has been established by judicial
proceedings, that which has been rightfully done and justly
suffered, and no satisfaction for it can be required. This would
explain why petitioner, though pardoned cannot be entitled to
receive back pay for lost earnings and benefits.
The better considered cases regard full pardon (at least
one not based on the offenders innocence) as relieving the party
from all the punitive consequences of his criminal act, including
the disqualifications or disabilities based on the of guilt. But it
relieves him from nothing more. To say, however, that the
offender is a new man and as innocent as if he had never
committed the offense. Is to ignore the difference between as if
he had never committed the offense. is to ignore the difference
between the crime and the criminal. A person adjudged guilty of
an offense is a convicted criminal, though pardoned; he may be
deserving of punishment, though left unpunished; and the law
may regard him as more dangerous to society than one never
guilty of crime, though it places no restraints upon him
following his conviction.
Pardons cannot mask the acts constituting the crime.
These are historical facts which despite the public
manifestation of mercy and forgiveness implicit in pardon,
ordinary, prudent men will take into account in their
subsequent dealings with the actor.
80. DOROJA VS. CARPIO
Nature: A petition by certiorari the decision of the Municipal
Trial Court of Zamboanga City, Branch IV, which denied
petitioners motion for subsidiary writ of execution against the
owner of the vehicle which figured in the accident.
Facts: October 23, 1985, accused-respondent Edwin Ramirez,
while driving a passenger Fuso Jeepney owned and operated by
Eduardo Toribio, bumped Dionisio Carpio, a pedestrian crossing
the street, as a consequence of which the latter suffered from
fractured left clavicle as reflected in the medico-legal certificate
and sustained injuries which required medical attention for a
period of (3) three months. Finding the accused Edwin Ramirez
y Wee guilty as a principal beyond reasonable doubt of the
Amended |Information to which he voluntarily pleaded guilty
and appreciating the mitigating circumstance in his favor,
hereby sentences him to suffer the penalty of one (1) month and
one (1) day to two (2) months of arresto mayor in its minimum
period. The accused filed an application for probation. A writ of
execution dated Mach 10, 1988 was duly served upon the
accused but was, however, returned unsatisfied due to the
insolvency of the accused as shown by the sheriffs return. Thus,
complaint moved for a subsidiary writ of execution against the
subsidiary liability of the owner-operator of the vehicle. The
same was denied by the trail court. A motion for reconsideration
of the said order was disallowed for the reason that complaint
having failed to raise the matter of subsidiary liability with the
appellate court, said court rendered its decision which has
become final and executory and the trial court has no power to
alter or modify such decision.
Issue: Whether or not the subsidiary liability of the owneroperator may be enforced in the same criminal proceeding
against the driver where the award was given or in a separate
civil action?
Held: compelling the owner-operator to pay on the basis of his
subsidiary liability does not constitute an amendment of the
judgment because in an action under Art, 103 of the Revised
Penal Code, once all the requisites as earlier discussed are met,
the employer becomes ipso facto subsidiary liable, without need
of a separate action. Such being the case, the subsidiary liability
can be enforced in the same case where the award was given,
and this does not constitute an act of amending the decision. It
becomes incumbent upon the court to grant a motion for
subsidiary writ of execution (but only after the employer has
been heard), upon conviction of the employee and after

execution is returned unsatisfied due to the employees


insolvency.
Wherefore, the order of the respondent court
disallowing the motion for subsidiary writ of execution is hereby
set aside, the Court a qou is directed to hear and decide in the
same proceeding the subsidiary liability of the alleged owneroperator of the passenger jeepney. Cost against private
respondent.
Ruling: Art. 103. Subsidiary civil liability of other persons. The
subsidiary liability established in the next preceding article shall
apply to employers, teacher, persons, and corporations engaged
in any kind of industry for felonies committed by their servants,
pupil, workmen, apprentices, or employees in t. In order that an
employer may be held subsidiarily liable for the employees
civil liability in the criminal action, it should be shown (1) that
the employer, etc. is engaged in any kind of industry, (2) that the
employee committed the offense in the discharge of his duties
and (3) that he is insolvent (Basa Marketing Corp. vs. Bolinao,
117 SCRA 156). The subsidiary liability of the employer,
however, arises only after conviction of the employee in the
criminal action. All these requisite present the employer
becomes ipso facto subsidiarily liable upon the employees
conviction and upon proof of the latters insolvency. Needles to
say, the case at bar satisfies all these requirements.

36

You might also like